Você está na página 1de 114

AFA

2024

AULA 07
Equações Algébricas
Prof. Victor So
Prof. Victor So

Sumário
APRESENTAÇÃO 4

1. EQUAÇÕES ALGÉBRICAS 5

1.1. TEOREMA FUNDAMENTAL DA ÁLGEBRA 5


1.1.1. TEOREMA DA DECOMPOSIÇÃO 5
1.1.2. COROLÁRIO 7

1.2. TEOREMA DA RAIZ COMPLEXA CONJUGADA 8

1.3. TEOREMA DAS RAÍZES IRRACIONAIS DA FORMA 𝒂 + 𝒃𝒄 11

1.4. TEOREMA DAS RAÍZES RACIONAIS 12

1.5. TEOREMA DE BOLZANO 15

2. RAIZ MÚLTIPLA E A DERIVADA POLINOMIAL 18

3. MÁXIMO DIVISOR COMUM 20

3.1. RAÍZES COMUNS 22

4. RELAÇÕES DE GIRARD 24

5. TRANSFORMAÇÕES 26

5.1. TRANSFORMADA ADITIVA 27


5.1.1. ALGORITMO DE HORNER-RUFFINI 27

5.2. TRANSFORMADA MULTIPLICATIVA 29

5.3. TRANSFORMADA INVERSA OU RECÍPROCA 29

6. EQUAÇÕES RECÍPROCAS 30

6.1. TEOREMA FUNDAMENTAL 30

6.2. RESOLUÇÃO DE UMA EQUAÇÃO RECÍPROCA 32


6.2.1. 1ª ESPÉCIE E GRAU PAR 32
6.2.2. 1ª ESPÉCIE E GRAU ÍMPAR 33
6.2.3. 2ª ESPÉCIE E GRAU PAR 33
6.2.4. 2ª ESPÉCIE E GRAU ÍMPAR 34

7. QUESTÕES NÍVEL 1 35

GABARITO 40

AULA 07 – EQUAÇÕES ALGÉBRICAS 2


Prof. Victor So

RESOLUÇÃO 41

8. QUESTÕES NÍVEL 2 52

GABARITO 59

RESOLUÇÃO 60

9. QUESTÕES NÍVEL 3 77

GABARITO 85

RESOLUÇÃO 85

10. CONSIDERAÇÕES FINAIS DA AULA 114

11. REFERÊNCIAS BIBLIOGRÁFICAS 114

AULA 07 – EQUAÇÕES ALGÉBRICAS 3


Prof. Victor So

APRESENTAÇÃO
Olá,
Chegamos à última aula de álgebra do curso. Nesta aula, veremos os teoremas envolvendo
raízes de equações algébricas polinomiais. São diversos teoremas e, para conseguir internalizar
todos, o recomendado é resolver muitos exercícios!
Se você for um aluno que já possui alguma base, você pode passar rapidamente pela teoria
ou, se preferir, ir direto para a lista de questões. Tente resolver todas e, sempre que tiver dúvidas,
não hesite em nos procurar no fórum de dúvidas.
Então, vamos à aula.
Bons estudos.

AULA 07 – EQUAÇÕES ALGÉBRICAS 4


Prof. Victor So

1. EQUAÇÕES ALGÉBRICAS
Estudamos na aula passada que, se um número 𝛼 ∈ ℂ é raiz do polinômio 𝑃(𝑥) = 𝑎𝑛 𝑥 𝑛 +
𝑎𝑛−1 𝑥 𝑛−1 + ⋯ + 𝑎1 𝑥 + 𝑎0 , então 𝑃(𝛼) = 0. Vimos pelo Teorema de D’Alembert que isso é
equivalente a dizer que 𝑃(𝑥) é divisível pelo fator 𝑥 − 𝛼, isto é,
𝑃 (𝛼 ) = 0 ⇔ 𝑃 (𝑥 ) ⋮ (𝑥 − 𝛼 )
No estudo das equações algébricas, queremos encontrar todos os números 𝛼 ∈ ℂ que
resultam 𝑃(𝛼) = 0, ou seja, todas as raízes da equação polinomial 𝑃 (𝑥) = 0. Podemos encontrar
essas raízes através da tentativa e erro e usando o dispositivo prático de Briot-Ruffini para reduzir
o grau do polinômio, mas isso pode ser muito trabalhoso quando o polinômio possui grau elevado,
como 𝜕𝑃 = 10. Felizmente, temos diversos teoremas que podem auxiliar-nos a encontrar as
raízes de um polinômio. Vamos estudar um dos mais importantes, o Teorema Fundamental da
Álgebra.

1.1. TEOREMA FUNDAMENTAL DA ÁLGEBRA

Todo polinômio de grau 𝒏 ≥ 𝟏, com coeficiente reais ou complexos, admite


pelo menos uma raiz complexa.

O Teorema Fundamental da Álgebra (T.F.A.) é um teorema que ajuda bastante na resolução


de questões sobre polinômios. Não veremos a demonstração desse teorema, pois foge ao escopo
do curso. Há diversos enunciados para esse teorema, porém, o mais conhecido é esse. Vejamos o
que podemos extrair dele.

1.1.1. TEOREMA DA DECOMPOSIÇÃO

Todo polinômio de grau 𝑛 ≥ 1, com coeficiente reais ou complexos, pode ser


decomposto em um produto de 𝑛 fatores do 1º grau.

Esse teorema diz que dado um polinômio 𝑃(𝑥) = 𝑎𝑛 𝑥 𝑛 + 𝑎𝑛−1 𝑥 𝑛−1 + ⋯ + 𝑎1 𝑥 + 𝑎0 , ele
também pode ser escrito da seguinte forma:
𝑃(𝑥) = 𝑎𝑛 (𝑥 − 𝛼1 )(𝑥 − 𝛼2 ) … (𝑥 − 𝛼𝑛 )
Em que 𝛼1 , 𝛼2 , … , 𝛼𝑛 são as raízes do polinômio 𝑃.

AULA 07 – EQUAÇÕES ALGÉBRICAS 5


Prof. Victor So

Demonstração
Seja o polinômio 𝑃 de grau 𝑛 ≥ 1 dado por
𝑃(𝑥) = 𝑎𝑛 𝑥 𝑛 + 𝑎𝑛−1 𝑥 𝑛−1 + ⋯ + 𝑎1 𝑥 + 𝑎0
Pelo T.F.A., podemos afirmar que 𝑃 admite pelo menos uma raiz complexa. Então, ∃𝛼1 ∈
ℂ tal que 𝑃(𝛼1 ) = 0. Pelo Teorema de D’Alembert, P é divisível por 𝑥 − 𝛼1 , desse modo:
𝑃(𝑥) ≡ (𝑥 − 𝛼1 )𝑄1 (𝑥), 𝜕𝑄1 = 𝑛 − 1
Se 𝑛 = 1, temos que 𝜕𝑄1 = 1 − 1 = 0 e, assim, o polinômio 𝑄1 é um polinômio constante,
logo:
𝑛 = 1 ⇒ 𝑃(𝑥) = 𝑎1 𝑥 + 𝑎0 ≡ (𝑥 − 𝛼1 )𝑄1
𝑎0
𝑎1 (𝑥 − ) ≡ (𝑥 − 𝛼1 )𝑄1 ∴ 𝑄1 ≡ 𝑎1
𝑎1
Se 𝑛 > 1, 𝑄1 não é um polinômio constante e possui grau 𝑛 − 1, então, podemos aplicar
o T.F.A. e afirmar que 𝑄1 possui uma raiz 𝛼2 ∈ ℂ tal que 𝑄1 (𝛼2 ) = 0, então:
𝑄1 (𝑥) ≡ (𝑥 − 𝛼2 )𝑄2 (𝑥), 𝜕𝑄2 = 𝑛 − 2
Seguindo da mesma forma para os polinômios 𝑄𝑖 , 𝑖 = {2, 3, 4, … , 𝑛}, resultantes, temos:
∃𝛼3 ∈ ℂ tal que 𝑄2 (𝛼3 ) = 0 ∴ 𝑄2 (𝑥) ≡ (𝑥 − 𝛼3 )𝑄3 (𝑥), 𝜕𝑄3 = 𝑛 − 3
∃𝛼4 ∈ ℂ tal que 𝑄3 (𝛼4 ) = 0 ∴ 𝑄3 (𝑥) ≡ (𝑥 − 𝛼4 )𝑄4 (𝑥), 𝜕𝑄4 = 𝑛 − 4

∃𝛼𝑛 ∈ ℂ tal que 𝑄𝑛−1 (𝛼𝑛 ) = 0 ∴ 𝑄𝑛−1 (𝑥) ≡ (𝑥 − 𝛼𝑛 )𝑄𝑛 (𝑥), 𝜕𝑄𝑛 = 𝑛 − 𝑛 = 0
Assim, temos que 𝜕𝑄𝑛 = 0 e pela identidade, podemos afirmar:
𝑃(𝑥) = 𝑎𝑛 𝑥 𝑛 + 𝑎𝑛−1 𝑥 𝑛−1 + ⋯ + 𝑎1 𝑥 + 𝑎0 ≡ (𝑥 − 𝛼1 )(𝑥 − 𝛼2 ) … (𝑥 − 𝛼𝑛 )𝑄𝑛
𝑎𝑛−1 𝑛−1 𝑎1 𝑎0
𝑃(𝑥) = 𝑎𝑛 (𝑥 𝑛 + 𝑥 + ⋯ + 𝑥 + ) ≡ (𝑥 − 𝛼1 )(𝑥 − 𝛼2 ) … (𝑥 − 𝛼𝑛 )𝑄𝑛
𝑎𝑛 𝑎𝑛 𝑎𝑛
∴ 𝑄𝑛 ≡ 𝑎𝑛
Portanto, 𝑃 pode ser escrito como
𝑃(𝑥) ≡ 𝑎𝑛 (𝑥 − 𝛼1 )(𝑥 − 𝛼2 ) … (𝑥 − 𝛼𝑛 )
Em que 𝛼1 , 𝛼2 , … , 𝛼𝑛 ∈ ℂ são raízes do polinômio.
Vamos demonstrar que o produto dos 𝑛 fatores é único.
Suponha que 𝑃 admita duas decomposições em 𝑛 fatores:
𝑃(𝑥) ≡ 𝑎𝑛 (⏟𝑥 − 𝛼1 )(𝑥 − 𝛼2 ) … (𝑥 − 𝛼𝑛 ) ≡ 𝑎𝑚 (⏟𝑥 − 𝛼′1 )(𝑥 − 𝛼′2 ) … (𝑥 − 𝛼′𝑛 )
𝑔𝑟𝑎𝑢 𝑛 𝑔𝑟𝑎𝑢 𝑛

Pela definição da identidade, devemos ter 𝑎𝑛 ≡ 𝑎𝑚 , logo:


(𝑥 − 𝛼1 )(𝑥 − 𝛼2 ) … (𝑥 − 𝛼𝑛 ) ≡ (𝑥 − 𝛼′1 )(𝑥 − 𝛼′2 ) … (𝑥 − 𝛼′𝑛 )

AULA 07 – EQUAÇÕES ALGÉBRICAS 6


Prof. Victor So

Para 𝑥 = 𝛼1 , o polinômio à esquerda zera, logo:


(𝛼1 − 𝛼1 )(𝛼1 − 𝛼2 ) … (𝛼1 − 𝛼𝑛 ) ≡ (𝛼1 − 𝛼′1 )(𝛼1 − 𝛼′2 ) … (𝛼1 − 𝛼′𝑛 )
0 ≡ (𝛼1 − 𝛼′1 )(𝛼1 − 𝛼′2 ) … (𝛼1 − 𝛼′𝑛 )
Assim, para a identidade ser satisfeita, devemos ter 𝛼𝑖′ = 𝛼1 , podemos supor, sem perda
de generalidade, 𝛼1 = 𝛼1′ e, assim,
(𝑥 − 𝛼2 )(𝑥 − 𝛼3 ) … (𝑥 − 𝛼𝑛 ) ≡ (𝑥 − 𝛼′2 )(𝑥 − 𝛼′3 ) … (𝑥 − 𝛼′𝑛 )
Novamente, agora para 𝑥 = 𝛼2 :
0 ≡ (𝛼2 − 𝛼 ′ 2 )(𝛼2 − 𝛼 ′ 3 ) … (𝛼2 − 𝛼 ′ 𝑛 ) ⇒ 𝛼2 = 𝛼2′
∴ (𝑥 − 𝛼3 )(𝑥 − 𝛼4 ) … (𝑥 − 𝛼𝑛 ) ≡ (𝑥 − 𝛼′3 )(𝑥 − 𝛼′4 ) … (𝑥 − 𝛼′𝑛 )
Seguindo de forma análoga para os outros fatores:
0 ≡ (𝛼3 − 𝛼′3 )(𝛼3 − 𝛼′4 ) … (𝛼3 − 𝛼′𝑛 ) ⇒ 𝛼3 = 𝛼3′

0 ≡ (𝛼𝑛 − 𝛼′𝑛 ) ⇒ 𝛼𝑛 = 𝛼𝑛′
Portanto, a decomposição de 𝑃 de grau 𝑛 em 𝑛 fatores é único.

1.1.2. COROLÁRIO

Todo polinômio de grau 𝒏 ≥ 𝟏, com coeficientes reais ou complexos, possui 𝒏, e


somente 𝒏, raízes complexas, podendo ser todas distintas ou não.

Esse é o resultado mais importante dos teoremas vistos. Dado um polinômio de grau 𝑛,
podemos afirmar que ele possui exatamente 𝑛 raízes complexas, sendo que elas não precisam ser
distintas entre si. Na ocorrência de haver raízes repetidas, temos que se 𝛼𝑖 é uma raiz repetida
𝑚𝑖 vezes, dizemos que a raiz 𝛼𝑖 possui multiplicidade 𝑚𝑖 , ou seja, dado um polinômio 𝑃 de grau
𝑛 e de raízes 𝛼1 , 𝛼2 , … , 𝛼𝑘 com multiplicidade 𝑚1 , 𝑚2 , … , 𝑚𝑘 , temos:
𝑃 (𝑥) ≡ 𝑎𝑛 (𝑥 − 𝛼1 )𝑚1 (𝑥 − 𝛼2 )𝑚2 … (𝑥 − 𝛼𝑘 )𝑚𝑘
Em que 𝑚1 + 𝑚2 + ⋯ + 𝑚𝑘 = 𝑛.
Assim, podemos afirmar:

Se 𝜶 é uma raiz de multiplicidade 𝒎 de um polinômio 𝑃 de grau 𝑛, então:


𝑃 (𝑥 ) ≡ (𝑥 − 𝛼 )𝑚 𝑄 (𝑥 ) 𝑒 𝑄 (𝛼 ) ≠ 0

AULA 07 – EQUAÇÕES ALGÉBRICAS 7


Prof. Victor So

Exemplos:
1.1.2.a) 𝑃(𝑥) = 4𝑥 10 + 5𝑥 6 − 78𝑥 2 + 100
Pelo corolário, temos que 𝑃 admite 10 raízes.
1.1.2.b) 𝑃(𝑥) = 10(𝑥 − 2)3 (𝑥 − 4)7
Nesse caso, temos que o polinômio 𝑃 possui 3 + 7 = 10 raízes, sendo a raiz 2 de
multiplicidade 3 e a raiz 4 de multiplicidade 7.

1.1.2.c) Sabendo que 3 é uma raiz de multiplicidade dupla, resolva a equação:


𝑥 4 − 6𝑥 3 + 5𝑥 2 + 24𝑥 − 36 = 0
Sabemos que 3 é uma raiz dupla, logo, podemos aplicar Briot-Ruffini duas vezes e fatorar
a expressão do quarto grau:

3 1 −6 5 24 −36

3 1 −3 −4 12 0

1 0 −4 0

Assim, a equação pode ser escrita como:


(𝑥 − 3)2 ( 𝑥 2 − 4) = 0
Basta resolver a equação quadrática para encontrar as outras raízes:
𝑥 2 − 4 = 0 ⇒ 𝑥 = ±2
Portanto, a solução é 𝑆 = {3; ±2}.

1.2. TEOREMA DA RAIZ COMPLEXA CONJUGADA


Uma propriedade que temos com os polinômios complexos é a seguinte:
Seja 𝑧 ∈ ℂ, 𝑃(𝑥) = 𝑎𝑛 𝑥 𝑛 + 𝑎(𝑛−1) 𝑥 𝑛−1 + ⋯ + 𝑎0 e 𝑎𝑛 , 𝑎𝑛−1 , … , 𝑎0 ∈ ℝ, sendo 𝑧̅ o
conjugado de 𝑧, então:
̅̅̅̅̅̅ = 𝑃(𝑧̅)
𝑃(𝑧)
Demonstração
̅̅̅̅̅̅
𝑃 (𝑧) = ̅̅̅̅̅̅̅̅̅̅̅̅̅̅̅̅̅̅̅̅̅̅̅̅̅̅̅̅̅̅̅̅̅̅̅̅ 𝑎𝑛 𝑧 𝑛 + ̅̅̅̅̅̅̅̅̅̅̅̅
𝑎𝑛 𝑧 𝑛 + 𝑎(𝑛−1) 𝑧 𝑛−1 + ⋯ + 𝑎0 = ̅̅̅̅̅̅ 𝑎𝑛−1 𝑧 𝑛−1 + ⋯ + ̅̅̅
𝑎0
Como os coeficientes do polinômio são todos reais, o conjugado deles não altera seu valor:
𝑎𝑛 𝑧 𝑛 + ̅̅̅̅̅̅̅̅̅̅̅̅
̅̅̅̅̅̅ 𝑎0 = 𝑎𝑛 𝑧̅̅̅
𝑎𝑛−1 𝑧 𝑛−1 + ⋯ + ̅̅̅ 𝑛 +𝑎 ̅̅̅̅̅̅
𝑛−1 𝑧
𝑛−1 + ⋯ + 𝑎
0

Das propriedades dos números complexos:

AULA 07 – EQUAÇÕES ALGÉBRICAS 8


Prof. Victor So

𝑧̅̅̅
𝑛 = 𝑧̅ 𝑛

Substituindo na equação, encontramos:


𝑎𝑛 𝑧̅̅̅
𝑛 +𝑎 ̅̅̅̅̅̅
𝑛−1 𝑧
𝑛−1 + ⋯ + 𝑎 = 𝑎 𝑧̅ 𝑛 + 𝑎
0 𝑛 𝑛−1 𝑧̅
𝑛−1
+ ⋯ + 𝑎0 = 𝑃(𝑧̅)
Vamos ao teorema:

Se uma equação polinomial de coeficientes reais admite 𝑎 + 𝑏𝑖 (𝑎 ∈ ℝ e 𝑏 ∈


∗)
ℝ como raiz, então 𝑎 − 𝑏𝑖 também será raiz da equação.

Atenção nesse teorema! Se uma equação polinomial possui todos os coeficientes reais e
uma de suas raízes é complexa, então, o conjugado dessa raiz também é raiz! Por exemplo, se a
questão afirma que 𝑖 é raiz do polinômio 𝑃 de coeficiente reais, então, podemos afirmar que −𝑖
também é raiz, ou seja, 𝑃(𝑖) = 𝑃(−𝑖) = 0.
Demonstração
𝑧 é raiz ⇒ 𝑃(𝑧) = 0
Aplicando o conjugado nos dois lados da equação:
̅̅̅̅̅̅ = 0̅
𝑃(𝑧)
Como 0 é real, temos 0̅ = 0, logo:
̅̅̅̅̅̅ = 0
𝑃(𝑧)
Usando o que acabamos de provar:
̅̅̅̅̅̅ = 𝑃(𝑧̅)
𝑃(𝑧)
Portanto:
𝑃(𝑧̅) = 0

Outro teorema que temos é o seguinte:

Se uma equação polinomial de coeficientes reais admite 𝑎 + 𝑏𝑖 (𝑎 ∈ ℝ e 𝛽 ∈


∗)
ℝ como raiz de multiplicidade 𝑚, então 𝑎 − 𝑏𝑖 também será raiz da equação com a
mesma multiplicidade.

Demonstração
Suponha que a equação polinomial de coeficientes reais 𝑃(𝑥) = 0 admite como raízes 𝑎 +
𝑏𝑖 de multiplicidade 𝑚 e 𝑎 − 𝑏𝑖 de multiplicidade 𝑚′ tal que 𝑚 > 𝑚′. Então, podemos escrever:

AULA 07 – EQUAÇÕES ALGÉBRICAS 9


Prof. Victor So

′ ′
𝑃(𝑥) ≡ [𝑥 − (𝑎 + 𝑏𝑖)]𝑚 [𝑥 − (𝑎 − 𝑏𝑖)]𝑚 𝑄 (𝑥)
Como 𝑚 > 𝑚′, temos que 𝑄 (𝑥) possui 𝑚 − 𝑚′ fatores 𝑥 − (𝑎 + 𝑏𝑖) e nenhum fator 𝑥 −
(𝑎 − 𝑏𝑖). Logo, 𝑄 (𝑥) possui coeficientes complexos.
Multiplicando os fatores dos divisores acima, temos:
′ ′
𝑃(𝑥) ≡ [(𝑥 − 𝑎) + 𝑏𝑖]𝑚 [(𝑥 − 𝑎) − 𝑏𝑖]𝑚 𝑄 (𝑥)

𝑃(𝑥) ≡ [(𝑥 − 𝑎)2 − (𝑏𝑖)2 ]𝑚 𝑄(𝑥)

𝑃(𝑥) ≡ [(𝑥 − 𝑎)2 + 𝑏2 ]𝑚 𝑄(𝑥)

Sabemos que 𝑃(𝑥) possui apenas coeficientes reais e o fator [(𝑥 − 𝑎)2 + 𝑏2 ]𝑚 também
possui apenas coeficientes reais, logo 𝑄(𝑥) deve possuir apenas coeficientes reais. Isso é um
absurdo, dado que 𝑄(𝑥) possui coeficientes complexos. Portanto, 𝑚 = 𝑚′.

1. Seja 𝑃(𝑧) = 𝑧 4 − 𝑧 3 − 5𝑧 2 − 𝑧 − 6. Sabe-se que 𝑖 é uma das raízes de 𝑃, calcule as


outras raízes.
Resolução:
Como os coeficientes de 𝑃 são todos reais, temos pelo teorema das raízes complexas
que −𝑖 também é raiz. Logo, aplicando o dispositivo prático de Briot-Ruffini para fatorar o
polinômio:
i 1 -1 -5 -1 -6
-i 1 -1+i -i-6 -6i 0
1 -1 -6 0
Assim, temos:
𝑝(𝑧) = (𝑧 − 𝑖)(𝑧 + 𝑖)(𝑧 2 − 𝑧 − 6) = 0
𝑧 2 − 𝑧 − 6 = 0 ⇒ 𝑧1 = 3 𝑒 𝑧2 = −2
As raízes são: ±𝑖, 3 e − 2.
Gabarito: ±𝒊, 𝟑 𝐞 − 𝟐.

2. Calcule as raízes do polinômio 𝑃(𝑧) = 1 + 𝑧 + 𝑧 2 + 𝑧 3 + 𝑧 4 .


Resolução:

AULA 07 – EQUAÇÕES ALGÉBRICAS 10


Prof. Victor So

O polinômio é a soma de uma PG de razão 𝑧, usando a fórmula da soma de uma PG


temos:
𝑎1 (𝑞𝑛 −1)
𝑆𝑛 =
𝑞−1
𝑧 5 −1
𝑃 (𝑧 ) = ,𝑧 ≠ 1
𝑧−1

Logo:
𝑧 5 − 1 = 0 ⇒ 𝑧 5 = 1 = 𝑐𝑖𝑠(2𝑘𝜋), 𝑘 = 0,1,2, …
Aplicando a fórmula de Moivre:
2𝑘𝜋
𝑧 = 𝑐𝑖𝑠 ( )
5

Como 𝑧 ≠ 1, temos como raízes:


2𝜋 4𝜋 6𝜋 8𝜋
𝑧1 = 𝑐𝑖𝑠 ( ) , 𝑧2 = 𝑐𝑖𝑠 ( ) , 𝑧3 = 𝑐𝑖𝑠 ( ) 𝑒 𝑧4 = 𝑐𝑖𝑠 ( )
5 5 5 5
𝟐𝝅 𝟒𝝅 𝟔𝝅 𝟖𝝅
Gabarito: 𝒛𝟏 = 𝒄𝒊𝒔 ( ) , 𝒛𝟐 = 𝒄𝒊𝒔 ( ) , 𝒛𝟑 = 𝒄𝒊𝒔 ( ) 𝒆 𝒛𝟒 = 𝒄𝒊𝒔 ( )
𝟓 𝟓 𝟓 𝟓

1.3. TEOREMA DAS RAÍZES IRRACIONAIS DA FORMA 𝒂 + 𝒃√𝒄

Se uma equação polinomial de coeficientes racionais admite 𝑎 + 𝑏√𝑐 como raiz, com
𝑎 ∈ ℚ, 𝑏 ∈ ℚ∗ e 𝑐 ∈ ℚ∗+ , então 𝑎 − 𝑏√𝑐 também será raiz da equação.

Esse teorema é parecido com o teorema da raiz complexa conjugada. Vamos demonstrá-
la.
Demonstração
Seja 𝑃 um polinômio de coeficientes racionais que admite 𝑎 + 𝑏√𝑐 como raiz, com 𝑎 ∈
ℚ, 𝑏 ∈ ℚ∗ e 𝑐 ∈ ℚ∗+ . Então, vamos dividir 𝑃 pelos fatores 𝑥 − (𝑎 + 𝑏√𝑐) e 𝑥 − (𝑎 − 𝑏√𝑐). Pela
definição de divisão:
𝑃(𝑥) ≡ [𝑥
⏟ − (𝑎 + 𝑏√𝑐)][𝑥 − (𝑎 − 𝑏√𝑐)] 𝑄(𝑥) + 𝑅(𝑥)
𝑔𝑟𝑎𝑢 2

Como o divisor possui grau 2, temos que 𝑅(𝑥) = 𝛼𝑥 + 𝛽.


Multiplicando os fatores, obtemos:
𝑃(𝑥) ≡ [(𝑥 − 𝑎) − 𝑏√𝑐][(𝑥 − 𝑎) + 𝑏√𝑐]𝑄(𝑥) + 𝛼𝑥 + 𝛽
2
𝑃(𝑥) ≡ [(𝑥 − 𝑎)2 − (𝑏√𝑐) ] 𝑄 (𝑥) + 𝛼𝑥 + 𝛽
𝑃(𝑥) ≡ [(𝑥 − 𝑎)2 − 𝑏2 𝑐 ]𝑄(𝑥) + 𝛼𝑥 + 𝛽

AULA 07 – EQUAÇÕES ALGÉBRICAS 11


Prof. Victor So

Como 𝑃(𝑥) e [(𝑥 − 𝑎)2 − 𝑏2 𝑐 ] possuem apenas coeficientes racionais, temos que 𝑄 (𝑥) e
𝛼𝑥 + 𝛽 também devem possuir apenas coeficientes racionais. Sabendo que 𝑎 + 𝑏√𝑐 é raiz,
temos:
𝑃(𝑎 + 𝑏√𝑐) = 0 ⇒ 𝑅(𝑎 + 𝑏√𝑐) = 0
𝛼(𝑎 + 𝑏√𝑐) + 𝛽 = 0
𝑎𝛼 + 𝛽 + 𝑏√𝑐𝛼 = 0
Para essa equação ser verdadeira, devemos ter:
𝑏√𝑐𝛼 = 0 𝑒 𝑎𝛼 + 𝛽 = 0
Como 𝑏√𝑐 ≠ 0, temos 𝛼 = 0, o que implica 𝛽 = 0. Logo, o resto da divisão de 𝑃 por
[(𝑥 − 𝑎) − 𝑏√𝑐][(𝑥 − 𝑎) + 𝑏√𝑐] é nulo.
𝑅 (𝑥 ) ≡ 0
Portanto, 𝑎 − 𝑏√𝑐 também é raiz de 𝑃.

Desse teorema, podemos enunciar:

Se uma equação polinomial de coeficientes racionais admite 𝑎 + 𝑏√𝑐 como raiz


de multiplicidade 𝑚, com 𝑎 ∈ ℚ, 𝑏 ∈ ℚ∗ e 𝑐 ∈ ℚ∗+ , então 𝑎 − 𝑏√𝑐 também será raiz da
equação de mesma multiplicidade.

A demonstração desse teorema é parecida com aquela realizada para a multiplicidade das
raízes complexas.

1.4. TEOREMA DAS RAÍZES RACIONAIS

Se a equação polinomial de coeficientes inteiros 𝑃(𝑥) = 𝑎𝑛 𝑥 𝑛 + 𝑎𝑛−1 𝑥 𝑛−1 + ⋯ +


𝑎1 𝑥 + 𝑎0 = 0 admite 𝑝/𝑞 (𝑝 ∈ ℤ, 𝑞 ∈ ℤ∗ e 𝑝 e 𝑞 primos entre si) como raiz, então 𝑝 é
divisor de 𝑎0 e 𝑞 é divisor de 𝑎𝑛 .

AULA 07 – EQUAÇÕES ALGÉBRICAS 12


Prof. Victor So

Esse teorema nos permite encontrar as raízes de um polinômio de coeficientes inteiros


através de um método mais refinado da tentativa e erro. Ele é baseado nos coeficientes do
polinômio. Vejamos sua demonstração.
Demonstração
Se 𝑝/𝑞 é raiz da equação polinomial de coeficientes inteiros 𝑃(𝑥) = 0, então:
𝑝 𝑝 𝑛 𝑝 𝑛−1 𝑝
𝑃 ( ) = 0 ⇒ 𝑎𝑛 ( ) + 𝑎𝑛−1 ( ) + ⋯ + 𝑎1 ( ) + 𝑎0 = 0
𝑞 𝑞 𝑞 𝑞
𝑎𝑛 𝑝𝑛 𝑎𝑛−1 𝑝𝑛−1 𝑎1 𝑝
𝑛
+ 𝑛−1
+ ⋯+ + 𝑎0 = 0
𝑞 𝑞 𝑞
Multiplicando a equação por 𝑞𝑛 :
𝑎𝑛 𝑝𝑛 + 𝑎𝑛−1 𝑝𝑛−1 𝑞 + ⋯ + 𝑎1 𝑝𝑞𝑛−1 + 𝑎0 𝑞𝑛 = 0
Assim, temos:
𝑎𝑛 𝑝𝑛 = −(𝑎𝑛−1 𝑝𝑛−1 𝑞 + ⋯ + 𝑎1 𝑝𝑞𝑛−1 + 𝑎0 𝑞𝑛 )
⇒ 𝑎𝑛 𝑝𝑛 = −𝑞 (⏟𝑎𝑛−1 𝑝𝑛−1 + ⋯ + 𝑎1 𝑝𝑞𝑛−2 + 𝑎0 𝑞𝑛−1 )
𝑘1 ∈ℤ

𝑎0 𝑞 𝑛 = − ⏟
(𝑎𝑛 𝑝𝑛 + 𝑎𝑛−1 𝑝𝑛−1 𝑞 + ⋯ + 𝑎1 𝑝𝑞𝑛−1 )

⇒ 𝑎0 𝑞𝑛 = −𝑝 (⏟𝑎𝑛 𝑝𝑛−1 + 𝑎𝑛−1 𝑝𝑛−2 𝑞 + ⋯ + 𝑎1 𝑞𝑛−1 )


𝑘2 ∈ℤ

Como os coeficientes 𝑎0 , 𝑎1 , 𝑎2 , … , 𝑎𝑛 , 𝑝 e 𝑞 são números inteiros, então, considerando


𝑝 ≠ 0:
𝑎𝑛 𝑝𝑛 = −𝑞𝑘1 , 𝑘1 ∈ ℤ
𝑎𝑛 𝑝𝑛
⇒ = −𝑘1 ∈ ℤ
𝑞
𝑎0 𝑞𝑛 = −𝑝𝑘2 , 𝑘2 ∈ ℤ
𝑎0 𝑞𝑛
⇒ = −𝑘2 ∈ ℤ
𝑝
Como 𝑝 e 𝑞 são primos entre si, temos que 𝑝𝑛 e 𝑞 são primos entre si e 𝑝 e 𝑞𝑛 são primos
entre si, logo:
𝑎𝑛 𝑝𝑛
∈ ℤ ⇒ 𝑞 𝑑𝑖𝑣𝑖𝑑𝑒 𝑎𝑛
𝑞
𝑎0 𝑞𝑛
∈ ℤ ⇒ 𝑝 𝑑𝑖𝑣𝑖𝑑𝑒 𝑎0
𝑝

AULA 07 – EQUAÇÕES ALGÉBRICAS 13


Prof. Victor So

Esse teorema que acabamos de provar diz que, se um polinômio de coeficientes inteiros
admite um número racional 𝑝/𝑞 como raiz, então 𝑝 é um fator do número 𝑎0 e 𝑞 é um fator do
número 𝑎𝑛 . Vejamos na prática como usamos esse teorema.
1.4.a) Encontre todas as raízes do polinômio 𝑝(𝑥) = 3𝑥 3 − 5𝑥 2 − 4𝑥 + 4.
Como o polinômio possui apenas coeficientes inteiros, podemos aplicar o teorema das
raízes racionais. Então, se 𝑝/𝑞 for raiz do polinômio, temos que 𝑝 divide 𝑎0 = 4 e 𝑞 divide 𝑎3 =
3. Assim, analisemos os divisores de 𝑎0 = 4 e 𝑎3 = 3.
Divisores de 𝑎0 = 4:
4 = 1 ⋅ 2 ⋅ 2 ⇒ 𝑑𝑖𝑣𝑖𝑠𝑜𝑟𝑒𝑠 (4) ∈ {1, 2, 4}

𝑝𝑜𝑠𝑠í𝑣𝑒𝑖𝑠 𝑣𝑎𝑙𝑜𝑟𝑒𝑠 𝑝𝑎𝑟𝑎 𝑝

Divisores de 𝑎3 = 3:
3 = 1 ⋅ 3 ⇒ 𝑑𝑖𝑣𝑖𝑠𝑜𝑟𝑒𝑠 (3) ∈ {1, 3}

𝑝𝑜𝑠𝑠í𝑣𝑒𝑖𝑠 𝑣𝑎𝑙𝑜𝑟𝑒𝑠 𝑝𝑎𝑟𝑎 𝑞

Dos possíveis valores de 𝑝 e 𝑞, devemos considerar a possibilidade de 𝑝/𝑞 ser um número


negativo. Desse modo, temos:
𝑝 1 2 4
∈ ± {1; 2; 4; ; ; }
𝑞 3 3 3
Esses são os candidatos às raízes, para saber quem é raiz, precisamos testar cada uma
delas. Temos 6 possibilidades para os positivos e 6 possibilidades para os negativos, totalizando
12 possibilidades. Fazendo os testes:
𝑝(1) = 3(1)3 − 5(1)2 − 4(1) + 4 = −2
𝑝(−1) = 3(−1)3 − 5(−1)2 − 4(−1) + 4 = 0 ⇒ −1 é 𝑟𝑎𝑖𝑧
𝑝(2) = 3(2)3 − 5(2)2 − 4(2) + 4 = 0 ⇒ 2 é 𝑟𝑎𝑖𝑧
𝑝(−2) = 3(−2)3 − 5(−2)2 − 4(−2) + 4 = −32
𝑝(4) = 3(4)3 − 5(4)2 − 4(4) + 4 = 100
𝑝(−4) = 3(−4)3 − 5(−4)2 − 4(−4) + 4 = −252
1 1 3 1 2 1 20
𝑝( ) = 3( ) − 5( ) − 4( ) + 4 =
3 3 3 3 9
1 1 3 1 2 1 14
𝑝 (− ) = 3 (− ) − 5 (− ) − 4 (− ) + 4 =
3 3 3 3 3
2 2 3 2 2 2 2
𝑝 ( ) = 3 ( ) − 5 ( ) − 4 ( ) + 4 = 0 ⇒ é 𝑟𝑎𝑖𝑧
3 3 3 3 3
2 2 3 2 2 2 32
𝑝 (− ) = 3 (− ) − 5 (− ) − 4 (− ) + 4 =
3 3 3 3 9

AULA 07 – EQUAÇÕES ALGÉBRICAS 14


Prof. Victor So

4 4 3 4 2 4 28
𝑝( ) = 3( ) − 5( ) − 4( ) + 4 = −
3 3 3 3 9
4 4 3 4 2 4 20
𝑝 (− ) = 3 (− ) − 5 (− ) − 4 (− ) + 4 = −
3 3 3 3 3
Portanto, as raízes de 𝑝 são os números −1, 2 e 2/3.

1.5. TEOREMA DE BOLZANO


O Teorema de Bolzano nos permite determinar o número de raízes reais em um
determinado intervalo real.
Enunciemos.

Se 𝑃(𝑥) = 0 é uma equação polinomial com coeficientes reais e ]𝑎, 𝑏[ é um intervalo


real aberto, então:
𝑷(𝒂) ⋅ 𝑷(𝒃) > 𝟎 implica que o polinômio 𝑃 possui um número par de raízes ou
nenhuma raiz no intervalo ]𝑎, 𝑏[.
𝑷(𝒂) ⋅ 𝑷(𝒃) < 𝟎 implica que o polinômio 𝑃 possui um número ímpar de raízes no
intervalo ]𝑎, 𝑏[.

Demonstração
Seja 𝑃(𝑥) = 0 uma equação polinomial de coeficientes reais tal que grau de 𝑃 é igual a 𝑛.
Como o polinômio possui grau 𝑛, podemos afirmar que 𝑃 admite 𝑛 raízes complexas. Indiquemos
as raízes reais por 𝛼1 , 𝛼2 , … , 𝛼𝑘 e as raízes complexas por 𝑧1 , 𝑧2 , … , 𝑧𝑚 . Pelo teorema da raiz
complexa conjugada e pelo fato de o polinômio possuir apenas coeficientes reais, podemos
afirmar que 𝑧1 , 𝑧2 , … , 𝑧𝑞 também são raízes. Logo, pelo teorema da decomposição, podemos
escrever:
𝑃(𝑥) = 𝑎𝑛 (𝑥 − 𝛼1 )(𝑥 − 𝛼2 ) … (𝑥 − 𝛼𝑘 )(𝑥 − 𝑧1 )(𝑥 − 𝑧1 ) … (𝑥 − 𝑧𝑚 )(𝑥 − 𝑧𝑚 )
Consideremos o produto dos fatores complexos como um polinômio 𝑄(𝑥), desse modo:
𝑄(𝑥) = (𝑥 − 𝑧1 )(𝑥 − 𝑧1 ) … (𝑥 − 𝑧𝑚 )(𝑥 − 𝑧𝑚 )
Vamos analisar o produto de cada par de fatores (𝑥 − 𝑧𝑖 )(𝑥 − 𝑧𝑖 ) e usar a forma algébrica
dos complexos:
𝑧 = 𝛼 + 𝛽𝑖; 𝛼 ∈ ℝ e 𝛽 ∈ ℝ∗
(𝑥 − 𝑧)(𝑥 − 𝑧) = [(𝑥 − (𝛼 + 𝛽𝑖)][(𝑥 − (𝛼 − 𝛽𝑖 )] = [(𝑥 − 𝛼 ) − 𝛽𝑖][(𝑥 − 𝛼) + 𝛽𝑖]
∴ (𝑥 − 𝑧)(𝑥 − 𝑧) = (𝑥 − 𝛼)2 + 𝛽2 > 0; ∀𝑥 ∈ ℝ

AULA 07 – EQUAÇÕES ALGÉBRICAS 15


Prof. Victor So

Assim, cada par de fatores de raízes complexas conjugadas gera uma função em 𝑥 sempre
maior que zero para qualquer valor de 𝑥. Portanto, podemos afirmar que 𝑄 (𝑥) > 0, ∀𝑥 ∈ ℝ.
∴ 𝑃(𝑥) = 𝑎𝑛 (𝑥 − 𝛼1 )(𝑥 − 𝛼2 ) … (𝑥 − 𝛼𝑘 )𝑄(𝑥); 𝑄(𝑥) > 0 ∀𝑥 ∈ ℝ
Agora, vamos analisar os valores de 𝑃 no intervalo aberto ]𝑎; 𝑏[.
Note que para 𝛼𝑖 , 𝑖 ∈ {1, 2, … , 𝑘 }, temos:
• 𝑎 < 𝛼𝑖 < 𝑏 (𝛼𝑖 𝑒𝑠𝑡á 𝑑𝑒𝑛𝑡𝑟𝑜 𝑑𝑜 𝑖𝑛𝑡𝑒𝑟𝑣𝑎𝑙𝑜 ]𝑎; 𝑏[ )
𝑎 − 𝛼𝑖 < 0
{ ⇒ (𝑎 − 𝛼𝑖 )(𝑏 − 𝛼𝑖 ) < 0
𝑏 − 𝛼𝑖 > 0

• 𝛼𝑖 < 𝑎 < 𝑏 ou 𝑎 < 𝑏 < 𝛼𝑖 (𝛼𝑖 𝑒𝑠𝑡á 𝑓𝑜𝑟𝑎 𝑑𝑜 𝑖𝑛𝑡𝑒𝑟𝑣𝑎𝑙𝑜 ]𝑎; 𝑏[ )


𝑎 − 𝛼𝑖 > 0
𝛼𝑖 < 𝑎 < 𝑏 ⇒ { ⇒ (𝑎 − 𝛼𝑖 )(𝑏 − 𝛼𝑖 ) > 0
𝑏 − 𝛼𝑖 > 0

𝑎 − 𝛼𝑖 < 0
𝑎 < 𝑏 < 𝛼𝑖 ⇒ { ⇒ (𝑎 − 𝛼𝑖 )(𝑏 − 𝛼𝑖 ) > 0
𝑏 − 𝛼𝑖 < 0

Analisemos o produto 𝑃(𝑎) ⋅ 𝑃(𝑏):


𝑃 (𝑎) = 𝑎𝑛 ⋅ 𝑄 (𝑎) ⋅ (𝑎 − 𝛼1 )(𝑎 − 𝛼2 ) … (𝑎 − 𝛼𝑘 )
𝑃 (𝑏) = 𝑎𝑛 ⋅ 𝑄(𝑏) ⋅ (𝑏 − 𝛼1 )(𝑏 − 𝛼2 ) … (𝑏 − 𝛼𝑘 )
⏟𝑛2 ⋅ ⏟
⇒ 𝑃 (𝑎 ) ⋅ 𝑃 (𝑏 ) = 𝑎 𝑄 (𝑎) ⋅ 𝑄 (𝑏) ⋅ (𝑎 − 𝛼1 )(𝑏 − 𝛼1 )(𝑎 − 𝛼2 )(𝑏 − 𝛼2 ) … (𝑎 − 𝛼𝑘 )(𝑏 − 𝛼𝑘 )
>0 >0

A equação acima nos permite inferir que o produto 𝑃(𝑎) ⋅ 𝑃 (𝑏) possui o mesmo sinal do
produto (𝑎 − 𝛼1 )(𝑏 − 𝛼1 )(𝑎 − 𝛼2 )(𝑏 − 𝛼2 ) … (𝑎 − 𝛼𝑘 )(𝑏 − 𝛼𝑘 ). Vimos que, se a raiz 𝛼𝑖 estiver
dentro do intervalo ]𝑎; 𝑏[, o produto (𝑎 − 𝛼𝑖 )(𝑏 − 𝛼𝑖 ) < 0, e se ela estiver fora do intervalo ]𝑎; 𝑏[
esse produto será positivo. Logo, devemos analisar apenas a quantidade de raízes dentro do
intervalo. Assim, temos as seguintes possibilidades:
• 𝑃 (𝑎 ) ⋅ 𝑃 (𝑏 ) > 0
Nesse caso, temos um número par de fatores negativos (𝑎 − 𝛼𝑖 )(𝑏 − 𝛼𝑖 ) e, portanto, um
número par de raízes no intervalo ]𝑎; 𝑏[.

AULA 07 – EQUAÇÕES ALGÉBRICAS 16


Prof. Victor So

• 𝑃 (𝑎 ) ⋅ 𝑃 (𝑏 ) < 0
Aqui, temos um número ímpar de fatores negativos (𝑎 − 𝛼𝑖 )(𝑏 − 𝛼𝑖 ) e,
consequentemente, um número ímpar de raízes no intervalo ]𝑎; 𝑏[.

Portanto, está provado o Teorema de Bolzano.


Vejamos um exemplo de aplicação para o teorema.

AULA 07 – EQUAÇÕES ALGÉBRICAS 17


Prof. Victor So

1.5.a) Determine os possíveis valores de 𝑎 ∈ ℝ, de modo que a equação 𝑎𝑥 4 + 3𝑥 2 − 2 =


0 tenha um número par de raízes no intervalo ]0; 3[.
Vamos aplicar o Teorema de Bolzano e calcular os valores de 𝑃(𝑥) = 𝑎𝑥 4 + 3𝑥 2 − 2 para
os extremos do intervalo ]0; 3[:
𝑃(0) = −2
𝑃(3) = 𝑎 ⋅ 34 + 3 ⋅ 32 − 2 = 81𝑎 + 25
Queremos um número par de raízes no intervalo, logo, devemos ter 𝑃(0) ⋅ 𝑃(3) > 0:
−2 ⋅ (81𝑎 + 25) > 0
81𝑎 + 25 < 0
25
∴𝑎<−
81

2. RAIZ MÚLTIPLA E A DERIVADA POLINOMIAL


Vimos que um polinômio 𝑃 possui raiz 𝛼 de multiplicidade 𝑚 se, e somente se, puder ser
escrito da seguinte forma:
𝑃 (𝑥 ) = (𝑥 − 𝛼 )𝑚 ⋅ 𝑄 (𝑥 ) ; 𝑄 (𝛼 ) ≠ 0
Há uma ferramenta muito poderosa que nos permite analisar a multiplicidade da raiz de
um polinômio. Ela envolve a derivada da função polinomial. Enunciemos o teorema.

O polinômio 𝑃(𝑥) = 𝑎𝑛 𝑥 𝑛 + 𝑎𝑛−1 𝑥 𝑛−1 + ⋯ + 𝑎1 𝑥 + 𝑎0 é divisível por (𝑥 − 𝛼)𝑚 tal


que 𝑚 ≤ 𝑛 se, e somente se, 𝑃(𝛼) = 𝑃′ (𝛼) = 𝑃′′ (𝛼) = ⋯ = 𝑃𝑚−1 (𝛼) = 0.

Esse teorema afirma que se um polinômio admite uma raiz com multiplicidade 𝑚, então,
as derivadas de ordem 𝑘 desse polinômio, com 𝑘 ∈ {1, 2, 3, … , 𝑚 − 1}, também possuem a
mesma raiz.
Demonstração
Para demonstrar esse teorema, lembremos da derivada do produto de funções e da
derivada de um monômio:
𝑓 (𝑥 ) = 𝑔 (𝑥 ) ⋅ ℎ (𝑥 ) ⇒ 𝑓 ′ (𝑥 ) = 𝑔 ′ (𝑥 ) ⋅ ℎ (𝑥 ) + 𝑔 ( 𝑥 ) ⋅ ℎ ′ (𝑥 )
𝐹 (𝑥) = 𝑎𝑛 𝑥 𝑛 ⇒ 𝐹 ′ (𝑥) = 𝑛𝑎𝑛 𝑥 𝑛−1
Assim, tomemos o polinômio 𝑃:
𝑃 (𝑥 ) = (𝑥 − 𝛼 ) 𝑚 ⋅ 𝑄 (𝑥 )
⇒ 𝑃′ (𝑥) = 𝑚(𝑥 − 𝛼)𝑚−1 ⋅ 𝑄(𝑥) + (𝑥 − 𝛼)𝑚 ⋅ 𝑄′ (𝑥)

AULA 07 – EQUAÇÕES ALGÉBRICAS 18


Prof. Victor So

⇒ 𝑃′ (𝑥) = (𝑥 − 𝛼)𝑚−1 [𝑚 ⋅ 𝑄(𝑥) + (𝑥 − 𝛼) ⋅ 𝑄′ (𝑥)]


Como 𝛼 é uma raiz de multiplicidade 𝑚 ≠ 0, temos que 𝑚 ⋅ 𝑄(𝛼) ≠ 0. Desse modo,
temos:
(𝛼 − 𝛼 ) ⋅ 𝑄 ′ (𝛼 ) = 𝑚 ⋅ 𝑄 (𝛼 )
𝑚 ⋅ 𝑄 (𝛼 ) + ⏟

≠0 =0

Portanto, 𝛼 é raiz de multiplicidade 𝑚 − 1 do polinômio 𝑃′ (𝑥).


Usando o mesmo raciocínio, conseguimos provar que se 𝛼 é raiz de multiplicidade 𝑚,
então:
𝑃(𝛼) = 𝑃′ (𝛼) = 𝑃′′ (𝛼) = ⋯ = 𝑃𝑚−1 (𝛼) = 0

Exemplo
2.a) Sabendo que a equação 𝑥 4 − 5𝑥 3 + 6𝑥 2 + 4𝑥 − 8 = 0 possui uma raiz de
multiplicidade 3, determine as raízes da equação.
Seja 𝑎, a raiz de multiplicidade 3. Então, para 𝑃(𝑥) = 𝑥 4 − 5𝑥 3 + 6𝑥 2 + 4𝑥 − 8, devemos
ter:
𝑃(𝑎) = 𝑃′ (𝑎) = 𝑃′′ (𝑎) = 0
Calculando as derivadas:
𝑃′ (𝑥) = 4𝑥 3 − 15𝑥 2 + 12𝑥 + 4
𝑃′′ (𝑥) = 12𝑥 2 − 30𝑥 + 12
Como 𝑎 é raiz de multiplicidade 3, temos:
𝑃′′ (𝑎) = 0 ⇒ 12𝑎2 − 30𝑎 + 12 = 0
15 ± √225 − 144 15 ± 9 1
𝑎= = = 2 𝑜𝑢
12 12 2
Encontramos duas possíveis raízes, uma delas é a raiz da equação polinomial 𝑃(𝑥) = 0.
Vamos testar os valores em 𝑃′ (𝑥) = 0:
1 1 3 1 2 1 27
𝑃′ ( ) = 4 ( ) − 15 ( ) + 12 ( ) + 4 =
2 2 2 2 4
′( ) 3 2
𝑃 2 = 4(2) − 15(2) + 12(2) + 4 = 0
Assim, verificamos que 2 é a raiz de multiplicidade 3 da equação, pois 𝑃′′ (2) = 𝑃′ (2) = 0.
Podemos testar a raiz no polinômio 𝑃:
𝑃 (2) = (2)4 − 5(2)3 + 6(2)2 + 4(2) − 8 = 0
Para determinar a última raiz, podemos aplicar o dispositivo prático de Briot-Ruffini:

AULA 07 – EQUAÇÕES ALGÉBRICAS 19


Prof. Victor So

2 1 −5 6 4 −8

2 1 −3 0 4 0

2 1 −1 −2 0

1 1 0

Pelo diagrama acima, temos:


𝑃 (𝑥 ) ≡ (𝑥 − 2)3 (𝑥 + 1)
Portanto, a última raiz é 𝑥 = −1.

3. MÁXIMO DIVISOR COMUM

O máximo divisor comum (𝑀𝐷𝐶) entre os polinômios 𝐹 (𝑥) e 𝐺 (𝑥) é um


polinômio 𝐻 (𝑥) tal que 𝐻 é o divisor de maior grau de 𝐹 e 𝐺.

Exemplo
3.a) Se 𝐹 (𝑥) = (𝑥 + 2)3 (𝑥 − 3)2 (𝑥 − 5)5 e 𝐺 (𝑥) = (𝑥 + 2)(𝑥 − 3)3 (𝑥 + 10), então
𝑀𝐷𝐶(𝐹, 𝐺) = (𝑥 + 2)(𝑥 − 3)2 .
3.b) Se 𝐹 (𝑥) = (𝑥 + 2)2 (𝑥 − 1) e 𝐺 (𝑥) = (𝑥 + 5)𝑥, então 𝑀𝐷𝐶 (𝐹, 𝐺 ) = 1, ou seja, 𝐹 e
𝐺 são primos entre si.
Para determinar o MDC, podemos usar a divisão euclidiana entre polinômios.
Antes de proceder, devemos saber que se 𝐹 (𝑥) ≡ 𝐺 (𝑥)𝑄 (𝑥) + 𝑅(𝑥), então:
𝑀𝐷𝐶 (𝐹, 𝐺 ) = 𝑀𝐷𝐶 (𝐺, 𝑅)
Demonstração
Se 𝑀𝐷𝐶 (𝐹, 𝐺 ) = 𝐻, então 𝐻 é divisor de 𝐹 e 𝐺, logo:
𝐹 (𝑥) = 𝐻 (𝑥)𝑄1 (𝑥) (𝑖)
𝐺 (𝑥) = 𝐻 (𝑥)𝑄2 (𝑥) (𝑖𝑖)
Se 𝑅 é o resto da divisão de 𝐹 por 𝐺, então:
𝐹 (𝑥) = 𝐺 (𝑥)𝑄 (𝑥) + 𝑅 (𝑥) ⇒ 𝑅(𝑥) = 𝐹 (𝑥) − 𝐺 (𝑥)𝑄(𝑥) (𝑖𝑖𝑖)
Substituindo (𝑖) e (𝑖𝑖) em (𝑖𝑖𝑖):

AULA 07 – EQUAÇÕES ALGÉBRICAS 20


Prof. Victor So

𝑅(𝑥) = 𝐻 (𝑥)𝑄1 (𝑥) − 𝐻 (𝑥)𝑄2 (𝑥)𝑄(𝑥) = (𝑄1 (𝑥) − 𝑄2 (𝑥)𝑄(𝑥))𝐻 (𝑥)


Assim, 𝐻 é divisor de 𝑅.
Se 𝑀𝐷𝐶 (𝐺, 𝑅) = 𝐻′, então 𝐻′ é divisor de 𝐺 e 𝑅. Como 𝐻 também é divisor de 𝐺 e 𝑅,
temos que 𝐻 é divisor de 𝐻′.
Pela definição de 𝐻′ ser divisor de 𝐺 e 𝑅, temos:
𝐺 (𝑥) = 𝐻 ′ (𝑥)𝑄3 (𝑥)
𝑅(𝑥) = 𝐻 ′ (𝑥)𝑄4 (𝑥)
Da relação 𝐹 (𝑥) = 𝐺 (𝑥)𝑄(𝑥) + 𝑅 (𝑥), temos:
𝐹 (𝑥) = 𝐻 ′ (𝑥)𝑄3 (𝑥)𝑄 (𝑥) + 𝐻 ′ (𝑥)𝑄4 (𝑥) = (𝑄3 (𝑥)𝑄(𝑥) + 𝑄4 (𝑥))𝐻 ′ (𝑥)
Ou seja, 𝐻′ é divisor de 𝐹. Como 𝐻′ também é divisor de 𝐺, temos que 𝐻′ é divisor de 𝐻.
Portanto, 𝐻′ é divisor de 𝐻 e 𝐻 é divisor de 𝐻′, logo, 𝐻 = 𝐻′.
∴ 𝑀𝐷𝐶 (𝐹, 𝐺 ) = 𝑀𝐷𝐶 (𝐺, 𝑅)
Vejamos, na prática, a aplicação da divisão euclidiana pelas divisões sucessivas, usando a
propriedade 𝑀𝐷𝐶 (𝐹, 𝐺 ) = 𝑀𝐷𝐶 (𝐺, 𝑅).
3.c) Determine o máximo divisor comum entre 𝐹 (𝑥) = 𝑥 4 − 5𝑥 2 + 4 e 𝐺 (𝑥) = 𝑥 3 − 7𝑥 −
6.
Vamos usar o método das chaves e dividir o polinômio de maior grau 𝐹 por 𝐺:

𝑥4 −5x 2 +4 𝑥 3 − 7𝑥 − 6

−𝑥 4 +7x 2 +6𝑥 𝑥

2x 2 +6𝑥 +4

Agora, dividimos o divisor 𝑥 3 − 7𝑥 − 6 pelo resto 2𝑥 2 + 6𝑥 + 4. Devemos repetir esse


processo até encontrar um resto nulo, ou seja, tomamos o divisor e dividimos pelo resto não nulo
e, assim, sucessivamente. Nessa divisão, podemos evidenciar o 2 do divisor 2𝑥 2 + 6𝑥 + 4 =
2(𝑥 2 + 3𝑥 + 2) e dividir 𝑥 3 − 7𝑥 − 6 por 𝑥 2 + 3𝑥 + 2 para simplificar as contas.

𝑥3 −7x −6 𝑥 2 + 3𝑥 + 2

−𝑥 3 −3𝑥 2 −2x 𝑥−3

−3𝑥 2 −9x −6
+3𝑥 2 +9x +6
0

AULA 07 – EQUAÇÕES ALGÉBRICAS 21


Prof. Victor So

Encontramos um resto nulo, logo, o máximo divisor comum entre 𝐹 e 𝐺 é o divisor da


divisão que resultou em resto nulo. Portanto:
𝑀𝐷𝐶 (𝑥 4 − 5𝑥 2 + 4; 𝑥 3 − 7𝑥 − 6) = 𝑥 2 + 3𝑥 + 2
No caso em que o resto da divisão euclidiana de 𝐹 por 𝐺 for uma constante diferente de
zero, podemos afirmar que 𝑀𝐷𝐶 (𝐹, 𝐺 ) = 1, isto é, 𝐹 e 𝐺 são primos entre si.

3.1. RAÍZES COMUNS


Note que quando 𝐹 e 𝐺 não são primos, temos que o 𝑀𝐷𝐶 (𝐹, 𝐺 ) terá fatores comuns dos
polinômios 𝐹 e 𝐺. Assim, podemos afirmar que as raízes desses fatores são as raízes comuns de
𝐹 e 𝐺, ou seja, as raízes comuns de 𝐹 e 𝐺 são também raízes do 𝑀𝐷𝐶 (𝐹, 𝐺 ).

𝛼 é uma raiz comum dos polinômios 𝐹 e 𝐺 se e somente se 𝛼 é raiz do 𝑀𝐷𝐶 (𝐹, 𝐺 ).

Vamos tomar os polinômios do exemplo anterior e encontrar as raízes comuns entre eles.
3.1.a) Encontre as raízes comuns entre 𝐹 (𝑥) = 𝑥 4 − 5𝑥 2 + 4 e 𝐺 (𝑥) = 𝑥 3 − 7𝑥 − 6.
Como vimos, o máximo divisor comum entre 𝐹 e 𝐺 é
𝑀𝐷𝐶 (𝐹, 𝐺 ) = 𝑀𝐷𝐶 (𝑥 4 − 5𝑥 2 + 4; 𝑥 3 − 7𝑥 − 6) = 𝑥 2 + 3𝑥 + 2
Se 𝛼 é uma raiz de 𝐹 e 𝐺, então ele deve ser raiz do 𝑀𝐷𝐶 (𝐹, 𝐺 ). Assim, basta encontrar as
raízes da equação 𝑥 2 + 3𝑥 + 2 = 0:
𝑥 2 + 3𝑥 + 2 = 0
−3 ± √1
𝑥= ⇒ 𝑥 = −2 𝑜𝑢 𝑥 = −1
2
Assim, as raízes comuns entre 𝐹 e 𝐺 são os números −2 e −1.

Há outro modo de encontrar as raízes comuns entre dois polinômios. Vejamos outro
exemplo.

3.1.b) Encontre as raízes comuns entre 𝑥 4 − 2𝑥 3 − 7𝑥 2 + 8𝑥 + 12 = 0 e 𝑥 4 − 4𝑥 3 −


13𝑥 2 + 4𝑥 + 12 = 0.
Suponha 𝛼 ∈ ℂ uma raiz comum das equações. Substituindo esse número nas equações e
subtraindo:

AULA 07 – EQUAÇÕES ALGÉBRICAS 22


Prof. Victor So

𝛼 4 − 2𝛼 3 − 7𝛼 2 + 8𝛼 + 12 = 0
−𝛼 4 + 4𝛼 3 + 13𝛼 2 − 4𝛼 − 12 = 0
2𝛼 3 + 6𝛼 2 + 4𝛼 = 0
As candidatas às raízes comuns são também raízes de 2𝛼 3 + 6𝛼 2 + 4𝛼 = 0.
2𝛼 3 + 6𝛼 2 + 4𝛼 = 0
2𝛼(𝛼 2 + 3𝛼 + 2) = 0
2𝛼(𝛼 + 2)(𝛼 + 1) = 0
𝑟𝑎í𝑧𝑒𝑠: 0; −1; −2
Para saber quais são as raízes comuns, podemos substituir esses valores em cada equação
e verificar qual satisfaz a igualdade ou aplicar Briot-Ruffini e verificar qual resulta em resto nulo.
Vamos usar o segundo método. Perceba que 0 não é raiz de ambas equações, pois o coeficiente
independente deles é 12. Logo, basta testar −1 e −2. Fazendo a divisão das expressões de cada
equação por Briot-Ruffini:

𝑥 4 − 2𝑥 3 − 7𝑥 2 + 8𝑥 + 12 = 0

−1 1 −2 −7 8 12

−2 1 −3 −4 12 0

1 −5 6 0

⇒ (𝑥 + 1)(𝑥 + 2)(𝑥 2 − 5𝑥 + 6) = 0
𝑥 4 − 4𝑥 3 − 13𝑥 2 + 4𝑥 + 12 = 0

−1 1 −4 −13 4 12

−2 1 −5 −8 12 0

1 −7 6 0

⇒ (𝑥 + 1)(𝑥 + 2)(𝑥 2 − 7𝑥 + 6) = 0
Portanto, como as divisões são exatas, temos que −1 e −2 são as raízes comuns das
equações.

AULA 07 – EQUAÇÕES ALGÉBRICAS 23


Prof. Victor So

4. RELAÇÕES DE GIRARD

As relações de Girard nos permitem relacionar as raízes de uma equação polinomial com
seus coeficientes. Elas nos ajudam bastante na resolução de questões envolvendo polinômios.
Vejamos do que se trata.
Considere o polinômio 𝑃(𝑥) = 𝑎𝑥 2 + 𝑏𝑥 + 𝑐, com 𝑎 ≠ 0, tal que suas raízes sejam 𝛼1 e
𝛼2 . Então, podemos escrever a seguinte identidade:
𝑎𝑥 2 + 𝑏𝑥 + 𝑐 ≡ 𝑎(𝑥 − 𝛼1 )(𝑥 − 𝛼2 )
𝑎𝑥 2 + 𝑏𝑥 + 𝑐 ≡ 𝑎[𝑥 2 − (𝛼1 + 𝛼2 )𝑥 + 𝛼1 𝛼2 ]
𝑎𝑥 2 + 𝑏𝑥 + 𝑐 ≡ 𝑎𝑥 2 − 𝑎(𝛼1 + 𝛼2 )𝑥 + 𝑎𝛼1 𝛼2
Para que a identidade seja satisfeita, devemos ter:
𝑏
𝑏 = −𝑎(𝛼1 + 𝛼2 ) 𝛼1 + 𝛼 2 = −
{ ⇒{ 𝑎
𝑐 = 𝑎𝛼1 𝛼2 𝑐
𝛼1 𝛼2 =
𝑎
Essas são as relações de Girard para uma equação algébrica do segundo grau.
Note que a soma das raízes é igual à razão −𝑏/𝑎 e o produto delas é igual à 𝑐/𝑎.
Vejamos o caso de um polinômio do terceiro grau.
Considere o polinômio 𝑃(𝑥) = 𝑎𝑥 3 + 𝑏𝑥 2 + 𝑐𝑥 + 𝑑, com 𝑎 ≠ 0, tal que suas raízes sejam
𝛼1 , 𝛼2 e 𝛼3 . Então, escrevendo 𝑃 como produto de fatores, temos:
𝑎𝑥 3 + 𝑏𝑥 2 + 𝑐𝑥 + 𝑑 ≡ 𝑎(𝑥 − 𝛼1 )(𝑥 − 𝛼2 )(𝑥 − 𝛼3 )
Desenvolvendo o membro à direita:
𝑎𝑥 3 + 𝑏𝑥 2 + 𝑐𝑥 + 𝑑 ≡ 𝑎(𝑥 2 − (𝛼1 + 𝛼2 )𝑥 + 𝛼1 𝛼2 )(𝑥 − 𝛼3 )
𝑎𝑥 3 + 𝑏𝑥 2 + 𝑐𝑥 + 𝑑 ≡ 𝑎[𝑥 3 − (𝛼1 + 𝛼2 )𝑥 2 + 𝛼1 𝛼2 𝑥 − 𝛼3 𝑥 2 + 𝛼3 (𝛼1 + 𝛼2 )𝑥 − 𝛼3 𝛼1 𝛼2 ]
𝑎𝑥 3 + 𝑏𝑥 2 + 𝑐𝑥 + 𝑑 ≡ 𝑎[𝑥 3 − (𝛼1 + 𝛼2 + 𝛼3 )𝑥 2 + (𝛼1 𝛼2 + 𝛼1 𝛼3 + 𝛼2 𝛼3 )𝑥 − 𝛼3 𝛼1 𝛼2 ]
Assim, pela identidade polinomial, temos:

AULA 07 – EQUAÇÕES ALGÉBRICAS 24


Prof. Victor So

𝑏
𝛼1 + 𝛼2 + 𝛼3 = −
𝑎
𝑐
𝛼1 𝛼2 + 𝛼1 𝛼3 + 𝛼2 𝛼3 =
𝑎
𝑑
{ 𝛼1 𝛼2 𝛼3 = −
𝑎
Essas são as relações de Girard para uma equação do terceiro grau. É possível generalizar
o resultado para uma equação algébrica de grau 𝑛.
Seja 𝑃(𝑥) = 𝑎𝑛 𝑥 𝑛 + 𝑎𝑛−1 𝑥 𝑛−1 + 𝑎𝑛−2 𝑥 𝑛−2 + ⋯ 𝑎𝑛−𝑘 𝑥 𝑘 + ⋯ + 𝑎1 𝑥 + 𝑎0 cujas raízes são
𝛼1 , 𝛼2 , … , 𝛼𝑛 , as relações de Girard são dadas por:
𝑎𝑛−1
𝛼1 + 𝛼2 + ⋯ + 𝛼𝑛 = − (𝑠𝑜𝑚𝑎 𝑑𝑎𝑠 𝑟𝑎í𝑧𝑒𝑠)
𝑎𝑛
𝑎𝑛−2
𝛼1 𝛼2 + 𝛼1 𝛼3 + ⋯ + 𝛼𝑛−1 𝛼𝑛 = (𝑠𝑜𝑚𝑎 𝑑𝑜 𝑝𝑟𝑜𝑑𝑢𝑡𝑜 𝑑𝑎𝑠 𝑟𝑎í𝑧𝑒𝑠 𝑑𝑜𝑖𝑠 𝑎 𝑑𝑜𝑖𝑠)
𝑎𝑛

𝑎 𝑛−𝑘
𝛼1 𝛼2 … 𝛼𝑘 + ⋯ = (−1)𝑘 (𝑠𝑜𝑚𝑎 𝑑𝑜 𝑝𝑟𝑜𝑑𝑢𝑡𝑜 𝑑𝑎𝑠 𝑟𝑎í𝑧𝑒𝑠 𝑘 𝑎 𝑘 )
𝑎𝑛

𝑛
𝑎0
𝛼1 𝛼2 … 𝛼𝑛 = (−1) (𝑝𝑟𝑜𝑑𝑢𝑡𝑜 𝑑𝑎𝑠 𝑛 𝑟𝑎í𝑧𝑒𝑠)
{ 𝑎𝑛
A demonstração dessas relações pode ser feita por indução finita.
Vejamos alguns exemplos de aplicação.
4.a) Sabendo que as raízes da equação 𝑥 3 − 21𝑥 2 + 126𝑥 − 216 = 0 estão em
progressão geométrica, determine-as.
Resolução:
Sejam 𝑥1 , 𝑥2 , 𝑥3 as raízes da equação dada. Como estão em PG, temos:
(𝑥1 , 𝑥2 , 𝑥3 ) = (𝑎, 𝑎𝑞, 𝑎𝑞2 )
Pelas relações de Girard:
𝑥 3 − 21𝑥 2 + 126𝑥 − 216 = 0
−21
𝑎 + 𝑎𝑞 + 𝑎𝑞2 = − ⇒ 𝑎(1 + 𝑞 + 𝑞2 ) = 21 (𝐼)
1
−216
𝑎 ⋅ 𝑎𝑞 ⋅ 𝑎𝑞2 = − = 216 ⇒ 𝑎3 𝑞3 = 216 ⇒ 𝑎𝑞 = 6 (𝐼𝐼)
1
Fazendo a divisão de (𝐼) por (𝐼𝐼):
𝑎(1 + 𝑞 + 𝑞2 ) 21 7
= = ⇒ 2(1 + 𝑞 + 𝑞2 ) = 7𝑞 ⇒ 2𝑞2 − 5𝑞 + 2 = 0
𝑎𝑞 6 2
Raízes:

AULA 07 – EQUAÇÕES ALGÉBRICAS 25


Prof. Victor So

5 ± √9 1
𝑞= = 2 𝑜𝑢
4 2
Podemos escolher qualquer uma das raízes, pois o resultado das raízes será o mesmo.
Logo, para 𝑞 = 2, temos:
𝑎⋅2=6⇒𝑎 =3
Portanto, as raízes são:
𝑎 = 3; 𝑎𝑞 = 6; 𝑎𝑞2 = 12
4.b) Determine a soma do cubo raízes da equação 𝑥 3 + 𝑥 − 1 = 0.
Resolução:
Sejam 𝛼, 𝛽, 𝛾 as raízes da equação. Como elas são raízes, temos:
𝛼3 + 𝛼 − 1 = 0
{𝛽 3 + 𝛽 − 1 = 0
𝛾3 + 𝛾 − 1 = 0
Somando as três equações:
𝛼 3 + 𝛽3 + 𝛾 3 + 𝛼 + 𝛽 + 𝛾 − 3 = 0
⇒ 𝛼 3 + 𝛽 3 + 𝛾 3 = 3 − (𝛼 + 𝛽 + 𝛾 )
Pelas relações de Girard, podemos obter a soma das raízes:
𝛼+𝛽+𝛾 =0
Logo:
𝛼 3 + 𝛽3 + 𝛾 3 = 3

5. TRANSFORMAÇÕES
Ao longo do curso, resolvemos muitas equações algébricas e, em algumas delas, aplicamos
a transformação algébrica para simplificar a resolução. Um exemplo é a equação 𝑥 4 − 5𝑥 2 + 6 =
0. Nós sabemos como encontrar as raízes de uma equação quadrática e, observando a equação,
vemos que ela lembra muito uma equação desse tipo. Se fizermos 𝑦 = 𝑥 2 , obtemos:
𝑥 4 − 5𝑥 2 + 6 = 0 ⇒ 𝑦 2 − 5𝑦 + 6 = 0
E resolvendo a equação em 𝑦, encontramos as raízes 𝑦1 = 3 ou 𝑦2 = 2.
Para encontrar as soluções em 𝑥, usamos a relação que estabelecemos 𝑦 = 𝑥 2 :
𝑦1 = 𝑥12 ⇒ 𝑥12 = 3 ⇒ 𝑥1 = ±√3
𝑦2 = 𝑥22 ⇒ 𝑥22 = 2 ⇒ 𝑥2 = ±√2

AULA 07 – EQUAÇÕES ALGÉBRICAS 26


Prof. Victor So

O que fizemos foi mudar a aparência da equação inicial para recair em um problema
conhecido. Então, transformar uma equação algébrica 𝑃1 (𝑥) = 0 em uma outra 𝑃2 (𝑦) =
0 significa obter uma lei de transformação 𝑦 = 𝑓 (𝑥) tal que as raízes das equações estejam
relacionadas.
Definimos a equação original 𝑃1 (𝑥) = 0 como equação primitiva e a equação resultante
𝑃2 (𝑦) = 0 como equação transformada. Vamos estudar os principais tipos de transformações e
algumas propriedades relacionadas a elas.

5.1. TRANSFORMADA ADITIVA


A transformação aditiva é uma relação de transformação do tipo:
𝑦 =𝑥+𝑎
Em que 𝑎 ∈ ℂ é uma constante.
Nessa transformação, a equação transformada 𝑃2 (𝑦) = 0 terá as raízes de 𝑃1 (𝑥) = 0
acrescidas de uma constante 𝑎.
Exemplo:
6.1.a) 𝑃1 (𝑥) = 𝑥 3 + 2𝑥 2 − 3𝑥 − 4 = 0
Façamos a transformação 𝑦 = 𝑥 + 1.
Dessa relação, temos:
𝑥 =𝑦−1
Substituindo em 𝑃1 :
𝑃1 (𝑥) = 𝑃1 (𝑦 − 1) = (𝑦 − 1)3 + 2(𝑦 − 1)2 − 3(𝑦 − 1) − 4 = 0
Assim, obtemos a equação transformada na variável 𝑦:
𝑃2 (𝑦) = 𝑦 3 − 𝑦 2 − 4𝑦 = 0
Que pode ser escrita na variável 𝑥:
𝑃2 (𝑥 + 1) = (𝑥 + 1)3 − (𝑥 + 1)2 − 4(𝑥 + 1) = 0
É possível obter esta transformação diretamente pelo algoritmo de Horner-Ruffini, este é
parecido com o algoritmo de Briot-Ruffini. Vejamos.

5.1.1. ALGORITMO DE HORNER-RUFFINI


Antes de aprendermos o algoritmo, relembremos o conceito de divisões sucessivas.
Dado um polinômio 𝑃1 (𝑥) = 𝑎𝑛 𝑥 𝑛 + 𝑎𝑛−1 𝑥 𝑛−1 + ⋯ + 𝑎1 𝑥 + 𝑎0 , vamos analisar as
divisões sucessivas deste polinômio pelo divisor 𝑥 + 𝑎. Pela definição de divisão:

AULA 07 – EQUAÇÕES ALGÉBRICAS 27


Prof. Victor So

𝑃1 (𝑥) = (𝑥 + 𝑎)𝑄1 (𝑥) + 𝑅1


O quociente da divisão é de grau 𝑛 − 1 e o resto é uma constante.
Procedendo com a divisão e dividindo 𝑄1 por 𝑥 + 𝑎, obtemos 𝑄2 tal que 𝜕𝑄2 = 𝑛 − 2:
𝑄1 (𝑥) = (𝑥 + 𝑎)𝑄2 (𝑥) + 𝑅2
Substituindo essa identidade em 𝑃1 :
𝑃1 (𝑥) = (𝑥 + 𝑎)[(𝑥 + 𝑎)𝑄2 (𝑥) + 𝑅2 ] + 𝑅1
𝑃1 (𝑥) = 𝑄2 (𝑥)(𝑥 + 𝑎)2 + 𝑅2 (𝑥 + 𝑎) + 𝑅1
Continuando com a divisão sucessiva, obtemos da divisão de 𝑄2 por 𝑥 + 𝑎:
𝑄2 (𝑥) = (𝑥 + 𝑎)𝑄3 (𝑥) + 𝑅3
Substituindo em 𝑃1 :
𝑃1 (𝑥) = [(𝑥 + 𝑎)𝑄3 (𝑥) + 𝑅3 ](𝑥 + 𝑎)2 + 𝑅2 (𝑥 + 𝑎) + 𝑅1
𝑃1 (𝑥) = 𝑄3 (𝑥)(𝑥 + 𝑎)3 + 𝑅3 (𝑥 + 𝑎)2 + 𝑅2 (𝑥 + 𝑎) + 𝑅1
Assim, fazendo as divisões sucessivas e as respectivas substituições, obtemos:
𝑃1 (𝑥) = 𝑄𝑛 (𝑥 + 𝑎)𝑛 + 𝑅𝑛 (𝑥 + 𝑎)𝑛−1 + ⋯ + 𝑅2 (𝑥 + 𝑎) + 𝑅1
Com base nisso, temos o dispositivo prático de Horner-Ruffini:

−𝑎 𝑃1

−𝑎 𝑄1 𝑅1

−𝑎 𝑄2 𝑅2

𝑄𝑛 𝑅𝑛

Perceba que esse algoritmo é baseado nas divisões sucessivas. Para cada linha, obteremos
um resto 𝑅𝑖 e estes serão os coeficientes dos termos (𝑥 + 𝑎)𝑖−1 . O coeficiente do termo (𝑥 + 𝑎)𝑛
será o quociente da última divisão. Note que serão 𝑛 restos resultantes de 𝑛 divisões sucessivas.
Vejamos um exemplo.
6.1.1.a) Façamos a transformação do polinômio do exemplo anterior 𝑥 3 + 2𝑥 2 − 3𝑥 − 4
em potências crescentes de 𝑥 + 1.
Como o polinômio possui grau 3, devemos fazer 3 divisões sucessivas. Pelo algoritmo de
Horner-Ruffini:

AULA 07 – EQUAÇÕES ALGÉBRICAS 28


Prof. Victor So

−1 1 2 −3 −4

−1 1 1 −4 0 = 𝑅1

−1 1 0 −4 = 𝑅2

1 = 𝑄3 −1 = 𝑅3

𝑃1 (𝑥) = 𝑄3 (𝑥 + 1)3 + 𝑅3 (𝑥 + 1)2 + 𝑅2 (𝑥 + 1) + 𝑅1


𝑃1 (𝑥) = (𝑥 + 1)3 − (𝑥 + 1)2 − 4(𝑥 + 1)

5.2. TRANSFORMADA MULTIPLICATIVA


A transformação multiplicativa é uma relação de transformação do tipo:
𝑦 = 𝑘𝑥; 𝑘 ≠ 0
Em que 𝑘 ∈ ℂ é uma constante.
Nessa transformação, a equação transformada 𝑃2 (𝑦) = 0 terá as raízes de 𝑃1 (𝑥) = 0
multiplicadas por uma constante 𝑘.
Exemplo.
6.2.a) Obtenha a equação cujas raízes são os triplos das raízes de 𝑥 3 − 6𝑥 2 + 11𝑥 − 6 =
0.
Queremos uma transformação do tipo 𝑦 = 3𝑥. Fazendo a substituição 𝑥 = 𝑦/3:
𝑦 3 𝑦 2 𝑦
( ) − 6 ( ) + 11 ( ) − 6 = 0
3 3 3
3 2
𝑦 2𝑦 11𝑦
− + −6=0
27 3 3
Eliminando os denominadores, obtemos a equação pedida:
𝑦 3 − 18𝑦 2 + 99𝑦 − 162 = 0

5.3. TRANSFORMADA INVERSA OU RECÍPROCA


A transformação recíproca é uma relação de transformação do tipo:
1
𝑦= ;𝑥 ≠ 0
𝑥

AULA 07 – EQUAÇÕES ALGÉBRICAS 29


Prof. Victor So

Nessa transformação, as raízes da equação resultante serão os inversos das raízes da


equação original.
Exemplo:
6.3.a) Obtenha uma equação cujas raízes são inversas das raízes de 𝑥 3 − 2𝑥 2 + 𝑥 − 5 =
0.
Queremos uma transformação do tipo 𝑦 = 1/𝑥, fazendo a substituição:
1 3 1 2 1
( ) − 2( ) + ( ) − 5 = 0
𝑦 𝑦 𝑦
1 2 1
3
− 2+ −5=0
𝑦 𝑦 𝑦
⇒ −5𝑦 3 + 𝑦 2 − 2𝑦 + 1 = 0
Perceba que em uma transformação recíproca, mudamos a ordem dos coeficientes e
substituímos 𝑥 por 𝑦.

6. EQUAÇÕES RECÍPROCAS
Dizemos que uma equação polinomial 𝑃(𝑥) = 0 é recíproca se, e somente se, a sua
1
transformada recíproca 𝑃 ( ) = 0 for equivalente à equação original.
𝑥

Exemplo:
6.a) 𝑃(𝑥) = 𝑥 4 − 2𝑥 3 + 𝑥 2 − 2𝑥 + 1 = 0
1 1 4 1 3 1 2 1
𝑃( ) = ( ) − 2( ) + ( ) − 2( ) + 1 = 0
𝑥 𝑥 𝑥 𝑥 𝑥
1 1 2 1 2
𝑃( ) = 4 − 3 + 2 − +1 = 0
𝑥 𝑥 𝑥 𝑥 𝑥
Multiplicando a equação por 𝑥 4 :
1
𝑃 ( ) = 𝑥 4 − 2𝑥 3 + 𝑥 2 − 2𝑥 + 1 = 0
𝑥
1
𝑃(𝑥) = 0 e 𝑃 ( ) = 0 são equações equivalentes, logo 𝑃(𝑥) = 0 é uma equação recíproca.
𝑥

6.1. TEOREMA FUNDAMENTAL


Se a equação recíproca admite 𝛼 ∈ ℂ∗ como raiz de multiplicidade 𝑚, então 1/𝛼 também
será raiz com a mesma multiplicidade.

AULA 07 – EQUAÇÕES ALGÉBRICAS 30


Prof. Victor So

Exemplo:
6.1.a) 𝑃(𝑥) = 2𝑥 3 − 3𝑥 2 − 3𝑥 + 2 é uma equação que admite 2 como raiz. Como ela é
recíproca, a outra raiz da equação é 1/2.
1
6.1.b) 𝑃(𝑥) = (𝑥 − 3) (𝑥 − ) (𝑥 − 1) é uma equação recíproca que admite 3 e 1/3 como
3
raízes.
Agora que sabemos o que é uma equação recíproca, vamos aprender a reconhecer quando
uma equação é recíproca.
Seja a equação polinomial dada por
𝑃(𝑥) = 𝑎𝑛 𝑥 𝑛 + 𝑎𝑛−1 𝑥 𝑛−1 + ⋯ + 𝑎2 𝑥 2 + 𝑎1 𝑥 + 𝑎0
A sua transformada recíproca é
1
𝑃 ( ) = 𝑎𝑛 + 𝑎𝑛−1 𝑥 + ⋯ + 𝑎2 𝑥 𝑛−2 + 𝑎1 𝑥 𝑛−1 + 𝑎0 𝑥 𝑛
𝑥
1
Se 𝑃(𝑥) = 0 é uma equação recíproca, então 𝑃(𝑥) = 0 e 𝑃 ( ) = 0 são equações
𝑥
equivalentes, logo, os coeficientes dessas equações são proporcionais. Sendo 𝑘 a constante de
proporcionalidade, então, igualando-se os coeficientes das respectivas equações:
𝑎𝑛 = 𝑘𝑎0
𝑎𝑛−1 = 𝑘𝑎1
𝑎𝑛−2 = 𝑘𝑎2

𝑎𝑛−𝑚 = 𝑘𝑎𝑚

𝑎𝑚 = 𝑘𝑎𝑛−𝑚

{ 𝑎0 = 𝑘𝑎𝑛
Tomando-se, sem perda de generalidade, 𝑎𝑚 = 𝑘𝑎𝑛−𝑚 e 𝑎𝑛−𝑚 = 𝑘𝑎𝑚 , com 0 ≤ 𝑚 ≤ 𝑛,
temos:
𝑎𝑚 = 𝑘 (𝑘𝑎𝑚 ) ⇒ 𝑎𝑚 = 𝑘 2 𝑎𝑚 ⇒ 𝑘 2 = 1
Assim, obtemos 𝑘 = 1 ou 𝑘 = −1.
Essas são as únicas possibilidades de constantes para as equações recíprocas.
Para 𝒌 = 𝟏, temos que os coeficientes dos termos equidistantes dos extremos da
equação recíproca 𝑃(𝑥) = 0 são iguais:

AULA 07 – EQUAÇÕES ALGÉBRICAS 31


Prof. Victor So

Essa equação é chamada de equação recíproca de primeira espécie.


Para 𝒌 = −𝟏, temos que os coeficientes dos termos equidistantes dos extremos da
equação recíproca 𝑃(𝑥) = 0 são opostos ou simétricos (mesmo módulo mas com sinais
trocados):

Essa equação é chamada de equação recíproca de segunda espécie.

6.2. RESOLUÇÃO DE UMA EQUAÇÃO RECÍPROCA


Até agora vimos a definição de equação recíproca e como classificá-la. Vamos aprender a
resolver cada tipo de equação recíproca que pode ser cobrada na prova. Sem perda de
generalidade, veremos como resolver as equações para graus menores, a resolução para graus
maiores usará a mesma ideia.

6.2.1. 1ª ESPÉCIE E GRAU PAR


Seja a equação 𝑎𝑥 4 + 𝑏𝑥 3 + 𝑐𝑥 2 + 𝑏𝑥 + 𝑎 = 0. Para resolver essa equação, dividimos a
equação por 𝑥 2 :
𝑏 𝑎
𝑎𝑥 2 + 𝑏𝑥 + 𝑐 + + =0
𝑥 𝑥2
Associamos o coeficiente dos termos equidistantes:
1 1
𝑎 (𝑥 2 + ) + 𝑏 (𝑥 + )+𝑐 =0
𝑥2 𝑥
Fazemos a transformação:
1 𝑒𝑙𝑒𝑣𝑎𝑛𝑑𝑜 𝑎𝑜 𝑞𝑢𝑎𝑑𝑟𝑎𝑑𝑜 2𝑥 1 1
𝑥+ =𝑦⇒ 𝑥2 + + 2 = 𝑦2 ⇒ 𝑥2 + 2 = 𝑦2 − 2
𝑥 𝑥 𝑥 𝑥

AULA 07 – EQUAÇÕES ALGÉBRICAS 32


Prof. Victor So

E obtemos uma equação do segundo grau em 𝑦:


𝑎(𝑦 2 − 2) + 𝑏𝑦 + 𝑐 = 0
𝑎𝑦 2 + 𝑏𝑦 + 𝑐 − 2𝑎 = 0
Essa equação resultará em duas raízes 𝑦1 e 𝑦2 . Para cada uma dessas raízes, encontramos
uma equação em 𝑥:
1
𝑦1 = 𝑥 + ⇒ 𝑥 2 − 𝑦1 𝑥 + 1 = 0
𝑥
1
𝑦2 = 𝑥 + ⇒ 𝑥 2 − 𝑦2 𝑥 + 1 = 0
𝑥
Resolvendo essas equações, encontramos a solução da equação.

6.2.2. 1ª ESPÉCIE E GRAU ÍMPAR


Seja a equação 𝑎𝑥 5 + 𝑏𝑥 4 + 𝑐𝑥 3 + 𝑐𝑥 2 + 𝑏𝑥 + 𝑎 = 0. Note que −1 é raiz dessa equação:
𝑎(−1)5 + 𝑏(−1)4 + 𝑐 (−1)3 + 𝑐 (−1)2 + 𝑏(−1) + 𝑎
= −𝑎 + 𝑏 − 𝑐 + 𝑐 − 𝑏 + 𝑎 = 0
Assim, podemos aplicar o dispositivo prático de Briot-Ruffini para simplificar a equação:

−1 𝑎 𝑏 𝑐 𝑐 𝑏 𝑎

𝑎 𝑏−𝑎 𝑎−𝑏+𝑐 𝑏−𝑎 𝑎 0

Obtemos a seguinte equação:


[𝑎𝑥 4 + (𝑏 − 𝑎)𝑥 3 + (𝑎 − 𝑏 + 𝑐 )𝑥 2 + (𝑏 − 𝑎)𝑥 + 𝑎] = 0
(𝑥 + 1) ⏟
1ª 𝑒𝑠𝑝é𝑐𝑖𝑒 𝑑𝑒 𝑔𝑟𝑎𝑢 𝑝𝑎𝑟

𝑎𝑥 4 + (𝑏 − 𝑎)𝑥 3 + (𝑎 − 𝑏 + 𝑐 )𝑥 2 + (𝑏 − 𝑎)𝑥 + 𝑎 = 0 é uma equação recíproca de 1ª


espécie de grau par e já aprendemos como resolver esse tipo de equação.
Portanto, quando encontrarmos uma equação recíproca de 1ª espécie de grau ímpar,
temos que −1 será raiz da equação e, assim, podemos fatorá-lo usando o algoritmo de Briot-
Ruffini. A equação resultante terá como fator uma equação recíproca de grau par, cuja solução é
conhecida.

6.2.3. 2ª ESPÉCIE E GRAU PAR


Seja a equação 𝑎𝑥 6 + 𝑏𝑥 5 + 𝑐𝑥 4 − 𝑐𝑥 2 − 𝑏𝑥 − 𝑎 = 0. Essa é uma equação recíproca de
2ª espécie de grau par. Note que ela possui o termo central nulo (coeficiente de 𝑥 3 é zero). Nesse
caso, 1 e −1 são raízes. Assim, podemos usar o algoritmo de Briot-Ruffini para fatorá-lo.

AULA 07 – EQUAÇÕES ALGÉBRICAS 33


Prof. Victor So

−1 𝑎 𝑏 𝑐 0 −𝑐 −𝑏 −𝑎

1 𝑎 𝑏−𝑎 𝑎−𝑏+𝑐 −𝑎 + 𝑏 − 𝑐 𝑎−𝑏 −𝑎 0

𝑎 𝑏 𝑎+𝑐 𝑏 𝑎 0

Obtemos a seguinte equação:


[𝑎𝑥 4 + 𝑏𝑥 3 + (𝑎 + 𝑐 )𝑥 2 + 𝑏𝑥 + 𝑎] = 0
(𝑥 + 1)(𝑥 − 1) ⏟
1ª 𝑒𝑠𝑝é𝑐𝑖𝑒 𝑑𝑒 𝑔𝑟𝑎𝑢 𝑝𝑎𝑟

Assim, basta resolver a equação recíproca 1ª espécie de grau par para encontrar as outras
raízes.

6.2.4. 2ª ESPÉCIE E GRAU ÍMPAR


Seja a equação 𝑎𝑥 5 + 𝑏𝑥 4 + 𝑐𝑥 3 − 𝑐𝑥 2 − 𝑏𝑥 − 𝑎 = 0. Perceba que 1 é raiz da equação,
pois a soma dos coeficientes é zero:
𝑎+𝑏+𝑐−𝑐−𝑏−𝑎 =0
Assim, podemos aplicar Briot-Ruffini:

1 𝑎 𝑏 𝑐 −𝑐 −𝑏 −𝑎

𝑎 𝑎+𝑏 𝑎+𝑏+𝑐 𝑎+𝑏 𝑎 0

Obtemos a seguinte equação:


(𝑥 − 1) [⏟𝑎𝑥 4 + (𝑎 + 𝑏)𝑥 3 + (𝑎 + 𝑏 + 𝑐 )𝑥 2 + (𝑎 + 𝑏)𝑥 + 𝑎] = 0
1ª 𝑒𝑠𝑝é𝑐𝑖𝑒 𝑑𝑒 𝑔𝑟𝑎𝑢 𝑝𝑎𝑟

Para encontrar as outras raízes, basta resolver a equação de 1ª espécie de grau par.

AULA 07 – EQUAÇÕES ALGÉBRICAS 34


Prof. Victor So

7. QUESTÕES NÍVEL 1
1. (EEAR/2016)

Dada a equação 𝟑𝒙𝟑 + 𝟐𝒙𝟐 − 𝒙 + 𝟑 = 𝟎 e sabendo que 𝒂, 𝒃 e 𝒄 são raízes dessa equação, o valor
do produto 𝒂 ⋅ 𝒃 ⋅ 𝒄 é

a) 𝟏

b) −𝟏
𝟏
c) 𝟑
𝟏
d) − 𝟑

2. (EEAR/2015)

Seja a equação 𝒙𝟑 − 𝟓𝒙𝟐 + 𝟕𝒙 − 𝟑 = 𝟎. Usando as relações de Girard, pode-se encontrar como


soma das raízes o valor

a) 12

b) 7

c) 5
d) 2

3. (EEAR/2012)
Seja a equação polinomial 𝟐𝒙𝟑 + 𝟒𝒙𝟐 − 𝟐𝒙 + 𝟒 = 𝟎. Se 𝑺 e 𝑷 são, respectivamente, a soma e o
produto de suas raízes, então

a) 𝑺 = 𝑷

b) 𝑺 = 𝟐𝑷

c) 𝑺 = 𝟐 e 𝑷 = −𝟒

d) 𝑺 = −𝟐 e 𝑷 = 𝟒

4. (EEAR/2000)

AULA 07 – EQUAÇÕES ALGÉBRICAS 35


Prof. Victor So

Dada a equação 𝒙𝟑 − 𝟏𝟎𝒙𝟐 − 𝟐𝒙 + 𝟐𝟎 = 𝟎 e sendo 𝒂, 𝒃 e 𝒄 as suas raízes, o valor da soma 𝒂𝟐 𝒃𝒄 +


𝒂𝒃𝟐 𝒄 + 𝒂𝒃𝒄𝟐 é:

a) 𝟐𝟎𝟎

b) −𝟐𝟎𝟎

c) 𝟒𝟎𝟎

d) −𝟒𝟎𝟎

5. (ESA/2006)

Seja 𝒙𝟐 + (𝒒 − 𝟑)𝒙 − 𝒒 − 𝟐 = 𝟎. O valor de 𝒒 que torna mínima a soma dos quadrados das raízes
da equação é:

a) 4

b) -2

c) -4

d) 2

e) 0

6. (ESA/2006)

A soma dos inversos das raízes da equação do 2º grau:

𝒙𝟐 − 𝟐(𝜶 + 𝟏)𝒙 + (𝜶 + 𝟑) = 𝟎
é igual a 𝟒. Se nesta equação 𝜶 é constante, podemos afirmar que 𝜶𝟐 é:

a) 16

b) 1

c) 25

d) 9

e) 4

7. (EsPCEx/2011)

As medidas em centímetros das arestas de um bloco retangular são as raízes da equação polinomial
𝒙𝟑 − 𝟏𝟒𝒙𝟐 + 𝟔𝟒𝒙 − 𝟗𝟔 = 𝟎. Denominando-se 𝒓, 𝒔 e 𝒕 essas medidas, se for construído um novo

AULA 07 – EQUAÇÕES ALGÉBRICAS 36


Prof. Victor So

bloco retangular, com arestas medindo (𝒓 − 𝟏), (𝒔 − 𝟏) e (𝒕 − 𝟏), ou seja, cada aresta medindo
𝟏 𝒄𝒎 a menos que a do bloco anterior, a medida do volume desse novo bloco será

a) 𝟑𝟔 𝒄𝒎𝟑

b) 𝟒𝟓 𝒄𝒎𝟑

c) 𝟓𝟒 𝒄𝒎𝟑

d) 𝟔𝟎 𝒄𝒎𝟑

e) 𝟖𝟎 𝒄𝒎𝟑

8. (EEAR/2011)

Seja 𝒓 a maior raiz da equação 𝒙(𝒙 + 𝟐)(𝒙 − 𝟏)𝟑 = 𝟎. Se 𝒎 é a multiplicidade de 𝒓, então 𝒓 ⋅ 𝒎 é


igual a

a) 6

b) 5

c) 4

d) 3

9. (EEAR/2011)

Uma equação polinomial de coeficientes reais admite como raízes os números −𝟐, 𝟎, 𝟐 e 𝟏 + 𝒊. O
menor grau que essa equação pode ter é

a) 6

b) 5

c) 4

d) 3

10. (EEAR/2014)

A equação (𝒙𝟐 + 𝟑)(𝒙 − 𝟐)(𝒙 + 𝟏) = 𝟎 tem raízes reais.

a) 3

b) 2
c) 1

AULA 07 – EQUAÇÕES ALGÉBRICAS 37


Prof. Victor So

d) 0

11. (EEAR/2018)

Se os números 𝟐, 𝟓, 𝟏 + 𝒊 e 𝟑 − 𝟓𝒊 são raízes de uma equação polinomial de grau 𝟔, a soma das


outras duas raízes dessa equação é

a) 𝟒 + 𝟒𝒊

b) 𝟒 + 𝟑𝒊

c) 𝟑 + 𝟒𝒊

d) 𝟑 + 𝟑𝒊

12. (EEAR/2019)
Seja a equação polinomial 𝒙𝟑 + 𝒃𝒙𝟐 + 𝒄𝒙 + 𝟏𝟖 = 𝟎. Se −𝟐 e 𝟑 são suas raízes, sendo que a raiz 𝟑
tem multiplicidade 2, o valor de 𝒃 é

a) 8

b) 6

c) -3

d) -4

13. (EEAR/2003)

Dentro do conjunto dos números complexos, a equação 𝒙𝟒 − 𝒙𝟐 − 𝟐 = 𝟎 tem como soluções

a) ±𝟐 e ±𝒊

b) ±√𝟐 e ±𝒊

c) ±𝟏 e 𝒊√𝟐

d) ±𝟏 e ±𝒊

14. (EEAR/2010)

Se a maior das raízes da equação 𝒙𝟑 − 𝟔𝒙𝟐 + 𝟏𝟏𝒙 − 𝟔 = 𝟎 é igual à soma das outras duas, então
seu valor é divisor de

a) 10
b) 16

AULA 07 – EQUAÇÕES ALGÉBRICAS 38


Prof. Victor So

c) 18

d) 20

15. (EEAR/2006)

A equação, cujas raízes são −√𝟐, +√𝟐, −√𝟓 e +√𝟓, é 𝒙𝟒 + 𝒂𝒙𝟐 + 𝒃 = 𝟎. O valor de |𝒂 + 𝒃| é

a) 2

b) 3

c) 4

d) 5

16. (EEAR/2001)

Uma das raízes da equação 𝒙𝟑 − 𝟏𝟐𝒙𝟐 + 𝟒𝟒𝒙 − 𝟒𝟖 = 𝟎 é a soma das outras duas. A maior raiz
dessa equação é

a) 7

b) 6

c) 4

d) 2

17. (EEAR/2003)
𝟑𝟓
Se a diferença entre os quadrados das raízes da equação 𝟑𝒙𝟐 − 𝟕𝒙 + 𝒄 = 𝟎 é , então o valor de 𝒄
𝟗
é
𝟐
a) − 𝟑

b) −𝟐
𝟐
c) 𝟑

d) 𝟐

18. (ESA/2017)

Se 𝟐 + 𝟑𝒊 é raiz de uma equação algébrica 𝑷(𝒙) = 𝟎, de coeficientes reais, então podemos afirmar
que:

AULA 07 – EQUAÇÕES ALGÉBRICAS 39


Prof. Victor So

a) 𝟑 − 𝟐𝒊 também é raiz da mesma equação.

b) 𝟐 − 𝟑𝒊 também é raiz da mesma equação.

c) 𝟐 também é raiz da mesma equação.

d) −𝟑𝒊 também é raiz da mesma equação.

e) 𝟑 + 𝟐𝒊 também é raiz da mesma equação.

19. (EsPCEx/2011)

Seja a função complexa 𝑷(𝒙) = 𝟐𝒙𝟑 − 𝟗𝒙𝟐 + 𝟏𝟒𝒙 − 𝟓. Sabendo-se que 𝟐 + 𝒊 é raiz de 𝑷, o intervalo
𝑰 de números reais que faz 𝑷(𝒙) < 𝟎, para todo 𝒙 ∈ 𝑰 é
𝟏
a) ] − ∞, [
𝟐

b) ]𝟎, 𝟏[
𝟏
c) ] 𝟒 , 𝟐[

d) ]𝟎, +∞[
𝟏 𝟑
e) ] − 𝟒 , [
𝟒

GABARITO
1. b
2. c
3. a
4. b
5. d
6. c
7. b
8. d
9. b
10. b
11. a
12. d
13. b
14. c
15. b
16. b
17. d
18. b

AULA 07 – EQUAÇÕES ALGÉBRICAS 40


Prof. Victor So

19. a

RESOLUÇÃO
1. (EEAR/2016)

Dada a equação 𝟑𝒙𝟑 + 𝟐𝒙𝟐 − 𝒙 + 𝟑 = 𝟎 e sabendo que 𝒂, 𝒃 e 𝒄 são raízes dessa equação, o valor
do produto 𝒂 ⋅ 𝒃 ⋅ 𝒄 é

a) 𝟏

b) −𝟏
𝟏
c) 𝟑
𝟏
d) − 𝟑

Comentários
Utilizando as Relações de Girard, o produto das raízes da equação 𝟑𝑥 3 + 2𝑥 2 − 𝑥 + 𝟑 =
0 é dado por
𝟑
𝑎⋅𝑏⋅𝑐 =− = −1
𝟑
Gabarito: “b”.
2. (EEAR/2015)

Seja a equação 𝒙𝟑 − 𝟓𝒙𝟐 + 𝟕𝒙 − 𝟑 = 𝟎. Usando as relações de Girard, pode-se encontrar como


soma das raízes o valor
a) 12

b) 7

c) 5

d) 2
Comentários
Utilizando as Relações de Girard, a soma das raízes da equação 𝟏𝑥 3 − 𝟓𝑥 2 + 7𝑥 − 3 =
0 é dada por
(−𝟓)
𝑎+𝑏+𝑐 =− =5
𝟏

Gabarito: “c”.
3. (EEAR/2012)

AULA 07 – EQUAÇÕES ALGÉBRICAS 41


Prof. Victor So

Seja a equação polinomial 𝟐𝒙𝟑 + 𝟒𝒙𝟐 − 𝟐𝒙 + 𝟒 = 𝟎. Se 𝑺 e 𝑷 são, respectivamente, a soma e o


produto de suas raízes, então

a) 𝑺 = 𝑷

b) 𝑺 = 𝟐𝑷

c) 𝑺 = 𝟐 e 𝑷 = −𝟒

d) 𝑺 = −𝟐 e 𝑷 = 𝟒
Comentários
Utilizando as Relações de Girard, a soma das raízes da equação 𝟐𝑥 3 + 𝟒𝑥 2 − 2𝑥 + 4 =
0 é dada por
𝟒
𝑆 =𝑎+𝑏+𝑐 =− = −2
𝟐
E, o produto das raízes da equação 𝟐𝑥 3 + 4𝑥 2 − 2𝑥 + 𝟒 = 0 é dado por
𝟒
𝑃 =𝑎⋅𝑏⋅𝑐 =− = −2
𝟐
Logo, 𝑆 = 𝑃
Gabarito: “a”.
4. (EEAR/2000)

Dada a equação 𝒙𝟑 − 𝟏𝟎𝒙𝟐 − 𝟐𝒙 + 𝟐𝟎 = 𝟎 e sendo 𝒂, 𝒃 e 𝒄 as suas raízes, o valor da soma 𝒂𝟐 𝒃𝒄 +


𝒂𝒃𝟐 𝒄 + 𝒂𝒃𝒄𝟐 é:

a) 𝟐𝟎𝟎

b) −𝟐𝟎𝟎

c) 𝟒𝟎𝟎

d) −𝟒𝟎𝟎
Comentários
𝑎2 𝑏𝑐 + 𝑎𝑏2 𝑐 + 𝑎𝑏𝑐 2 = (𝑎 ⋅ 𝑏 ⋅ 𝑐 ) ⋅ (𝑎 + 𝑏 + 𝑐 )
Utilizando as Relações de Girard, a soma das raízes da equação 𝟏𝑥 3 − 𝟏𝟎𝑥 2 − 2𝑥 + 20 =
0 é dada por
(−𝟏𝟎)
𝑎+𝑏+𝑐 =− = 10
𝟏
E, o produto das raízes da equação 𝟏𝑥 3 − 10𝑥 2 − 2𝑥 + 𝟐𝟎 = 0 é dado por
𝟐𝟎
𝑎⋅𝑏⋅𝑐 =− = −20
𝟏

AULA 07 – EQUAÇÕES ALGÉBRICAS 42


Prof. Victor So

Portanto,
(𝑎 ⋅ 𝑏 ⋅ 𝑐 ) ⋅ (𝑎 + 𝑏 + 𝑐 ) = (−20) ⋅ (10) = −200
𝑎2 𝑏𝑐 + 𝑎𝑏2 𝑐 + 𝑎𝑏𝑐 2 = −200
Gabarito: “b”.
5. (ESA/2006)

Seja 𝒙𝟐 + (𝒒 − 𝟑)𝒙 − 𝒒 − 𝟐 = 𝟎. O valor de 𝒒 que torna mínima a soma dos quadrados das raízes
da equação é:

a) 4

b) -2

c) -4

d) 2

e) 0
Comentários
Utilizando as Relações De Girard. Sejam 𝑎 e 𝑏 raízes do polinômio:
𝑞−3
𝑎+𝑏 =− =3−𝑞
1
−𝑞 − 2
𝑎⋅𝑏 = = −2 − 𝑞
1
Sendo assim, vejamos agora o valor de 𝑎2 + 𝑏2
(𝑎 + 𝑏)2 = 𝑎2 + 2𝑎𝑏 + 𝑏2
⇒ 𝑎 2 + 𝑏 2 = ( 𝑎 + 𝑏 ) 2 − 2 ⋅ (𝑎 ⋅ 𝑏 )
𝑎2 + 𝑏2 = (3 − 𝑞)2 − 2 ⋅ (−2 − 𝑞)
= 9 − 6𝑞 + 𝑞2 + 4 + 2𝑞 =
⇒ 𝑎2 + 𝑏2 = 𝑓 (𝑞) = 𝑞2 − 4𝑞 + 13
Devemos encontrar o valor de 𝑞 tal que 𝑓(𝑞) seja mínimo. O valor da abcissa do vértice da
parábola é dado por:
𝑏
𝑞𝑚𝑖𝑛 = −
2𝑎
−4
𝑞𝑚𝑖𝑛 = − =2
2⋅1
𝑞𝑚𝑖𝑛 = 2
Gabarito: “d”.
6. (ESA/2006)

AULA 07 – EQUAÇÕES ALGÉBRICAS 43


Prof. Victor So

A soma dos inversos das raízes da equação do 2º grau:

𝒙𝟐 − 𝟐(𝜶 + 𝟏)𝒙 + (𝜶 + 𝟑) = 𝟎
é igual a 𝟒. Se nesta equação 𝜶 é constante, podemos afirmar que 𝜶𝟐 é:

a) 16

b) 1

c) 25

d) 9

e) 4
Comentários
Utilizando as Relações De Girard. Sejam 𝑎 e 𝑏 raízes do polinômio:
−2(𝛼 + 1)
𝑎+𝑏 =− = 2(𝛼 + 1)
1
(𝛼 + 3)
𝑎⋅𝑏 = = (𝛼 + 3)
1
Sendo assim:
1 1 𝑏 + 𝑎 2(𝛼 + 1)
+ = = =4
𝑎 𝑏 𝑎𝑏 (𝛼 + 3)
2𝛼 + 2 = 4𝛼 + 12
2𝛼 + 10 = 0
𝛼 = −5
Portanto,
𝛼 2 = (−5)2 = 25
Gabarito: “c”.
7. (EsPCEx/2011)

As medidas em centímetros das arestas de um bloco retangular são as raízes da equação polinomial
𝒙𝟑 − 𝟏𝟒𝒙𝟐 + 𝟔𝟒𝒙 − 𝟗𝟔 = 𝟎. Denominando-se 𝒓, 𝒔 e 𝒕 essas medidas, se for construído um novo
bloco retangular, com arestas medindo (𝒓 − 𝟏), (𝒔 − 𝟏) e (𝒕 − 𝟏), ou seja, cada aresta medindo
𝟏 𝒄𝒎 a menos que a do bloco anterior, a medida do volume desse novo bloco será

a) 𝟑𝟔 𝒄𝒎𝟑

b) 𝟒𝟓 𝒄𝒎𝟑
c) 𝟓𝟒 𝒄𝒎𝟑

AULA 07 – EQUAÇÕES ALGÉBRICAS 44


Prof. Victor So

d) 𝟔𝟎 𝒄𝒎𝟑

e) 𝟖𝟎 𝒄𝒎𝟑
Comentários
Desenvolvendo o produto, obtemos:
𝑉 = (𝑟 − 1) ⋅ ( 𝑠 − 1) ⋅ (𝑡 − 1) =
𝑉 = (𝑟 ⋅ 𝑠 ⋅ 𝑡) − (𝑟𝑠 + 𝑟𝑡 + 𝑠𝑡 ) + (𝑟 + 𝑠 + 𝑡 ) − 1
Aplicando as Relações de Girard no polinômio, obtemos que:
−14
𝑟+𝑠+𝑡 =− = 14
1
64
𝑟𝑠 + 𝑟𝑡 + 𝑠𝑡 = = 64
1
−96
{ 𝑟𝑠𝑡 = − 1 = 96
Portanto:
𝑉 = (96) − (64) + (14) − 1 = 45
𝑉 = 45 𝑐𝑚3
Gabarito: “b”.
8. (EEAR/2011)

Seja 𝒓 a maior raiz da equação 𝒙(𝒙 + 𝟐)(𝒙 − 𝟏)𝟑 = 𝟎. Se 𝒎 é a multiplicidade de 𝒓, então 𝒓 ⋅ 𝒎 é


igual a

a) 6

b) 5

c) 4

d) 3
Comentários
O polinômio 𝑥 (𝑥 + 2)(𝑥 − 1)3 possui raízes:
0 𝑚𝑢𝑙𝑡𝑖𝑝𝑙𝑖𝑐𝑖𝑑𝑎𝑑𝑒 1
{−2 𝑚𝑢𝑙𝑡𝑖𝑝𝑙𝑖𝑐𝑖𝑑𝑎𝑑𝑒 1
1 𝑚𝑢𝑙𝑡𝑖𝑝𝑙𝑖𝑐𝑖𝑑𝑎𝑑𝑒 3
A maior das raízes é 𝑟 = 1, logo 𝑚 = 3
𝑟⋅𝑚 =1⋅3=3
Gabarito: “d”.
9. (EEAR/2011)

AULA 07 – EQUAÇÕES ALGÉBRICAS 45


Prof. Victor So

Uma equação polinomial de coeficientes reais admite como raízes os números −𝟐, 𝟎, 𝟐 e 𝟏 + 𝒊. O
menor grau que essa equação pode ter é

a) 6

b) 5

c) 4

d) 3
Comentários
Primeiramente, sendo os coeficientes reais, as raízes complexas veem aos pares. Sendo
assim, se 1 + 𝑖 é raiz, então (̅̅̅̅̅̅̅̅̅
1 + 𝑖) = 1 − 𝑖 também é raiz.
Portanto, temos um total de 5 raízes para o polinômio, o grau mínimo deste polinômio
deve ser 5.
Gabarito: “b”.
10. (EEAR/2014)

A equação (𝒙𝟐 + 𝟑)(𝒙 − 𝟐)(𝒙 + 𝟏) = 𝟎 tem raízes reais.

a) 3

b) 2

c) 1

d) 0
Comentários
Perceba que (𝑥 2 + 3) não fornece raízes reais, pois:
(𝑥 2 + 3)(𝑥 − 2)(𝑥 + 1) = 0
⇒ (𝑥 2 + 3) = 0 𝑜𝑢 𝑥 = 2 𝑜𝑢 𝑥 = −1
𝑥2 + 3 = 0
𝑥 2 = −3
𝑥 = ±√3𝑖
Dentre as raízes apresentadas, apenas 2 delas são reais.
Gabarito: “b”.
11. (EEAR/2018)

Se os números 𝟐, 𝟓, 𝟏 + 𝒊 e 𝟑 − 𝟓𝒊 são raízes de uma equação polinomial de grau 𝟔, a soma das


outras duas raízes dessa equação é
a) 𝟒 + 𝟒𝒊

AULA 07 – EQUAÇÕES ALGÉBRICAS 46


Prof. Victor So

b) 𝟒 + 𝟑𝒊

c) 𝟑 + 𝟒𝒊

d) 𝟑 + 𝟑𝒊
Comentários
Considere que os coeficientes do polinômio são números reais. Lembre-se que as raízes
complexas vêm aos pares. Se um complexo é raiz, o seu conjugado também é.
(̅̅̅̅̅̅̅̅̅
1 + 𝑖) = 1 − 𝑖
⇒{ 𝑠ã𝑜 𝑟𝑎í𝑧𝑒𝑠
(̅̅̅̅̅̅̅̅̅̅̅
3 − 5𝑖) = 3 + 5𝑖
Portanto,
(1 − 𝑖) + (3 + 5𝑖) = 4 + 4𝑖
Gabarito: “a”.
12. (EEAR/2019)

Seja a equação polinomial 𝒙𝟑 + 𝒃𝒙𝟐 + 𝒄𝒙 + 𝟏𝟖 = 𝟎. Se −𝟐 e 𝟑 são suas raízes, sendo que a raiz 𝟑
tem multiplicidade 2, o valor de 𝒃 é

a) 8

b) 6

c) -3

d) -4
Comentários
Utilizaremos os conceitos de igualdade de polinômios:
𝑥 3 + 𝑏𝑥 2 + 𝑐𝑥 + 18 = (𝑥 + 2) ⋅ (𝑥 − 3)2 = 0
= (𝑥 + 2) ⋅ (𝑥 2 − 6𝑥 + 9) =
⇒ 𝑥 3 + 𝑏𝑥 2 + 𝑐𝑥 + 18 = 𝑥 3 − 4𝑥 2 − 3𝑥 + 18
𝑏 = −4
⇒ {
𝑐 = −3
Gabarito: “d”.
13. (EEAR/2003)

Dentro do conjunto dos números complexos, a equação 𝒙𝟒 − 𝒙𝟐 − 𝟐 = 𝟎 tem como soluções

a) ±𝟐 e ±𝒊

b) ±√𝟐 e ±𝒊
c) ±𝟏 e 𝒊√𝟐

AULA 07 – EQUAÇÕES ALGÉBRICAS 47


Prof. Victor So

d) ±𝟏 e ±𝒊
Comentários
Considere 𝑦 = 𝑥 2
𝑥4 − 𝑥2 − 2 = 𝑦2 − 𝑦 − 2 = 0
Resolvendo a equação de 2º grau em 𝑦, obtemos:
𝑦1 = 2
{
𝑦2 = −1
2
⇒ { 𝑥2 = 2 ⇒ {𝑥 = ±√2
𝑥 = −1 𝑥 = ±𝑖
Portanto, as raízes são ±√2 e ±𝑖.
Gabarito: “b”.
14. (EEAR/2010)

Se a maior das raízes da equação 𝒙𝟑 − 𝟔𝒙𝟐 + 𝟏𝟏𝒙 − 𝟔 = 𝟎 é igual à soma das outras duas, então
seu valor é divisor de

a) 10

b) 16

c) 18

d) 20
Comentários
Sejam as raízes 𝑎, 𝑏 e 𝑐, tal que 𝑐 = 𝑎 + 𝑏
Pelas Relações de Girard:
−6
𝑎+𝑏+𝑐 =− =6
1
𝑎 + 𝑏 + 𝑎 + 𝑏 = 2(𝑎 + 𝑏 ) = 6
2⋅𝑐 =6 ⇒ 𝑐 =3
Logo, a maior raiz vale 3 e, analisando as alternativas, obtemos que 3 é divisor de 18 (item
c)
Gabarito: “c”.
15. (EEAR/2006)

A equação, cujas raízes são −√𝟐, +√𝟐, −√𝟓 e +√𝟓, é 𝒙𝟒 + 𝒂𝒙𝟐 + 𝒃 = 𝟎. O valor de |𝒂 + 𝒃| é

a) 2
b) 3

AULA 07 – EQUAÇÕES ALGÉBRICAS 48


Prof. Victor So

c) 4

d) 5
Comentários
𝑥 4 + 𝑎𝑥 2 + 𝑏 = (𝑥 + √2) ⋅ (𝑥 − √2) ⋅ (𝑥 + √5) ⋅ (𝑥 − √5) =
= (𝑥 2 − 2) ⋅ (𝑥 2 − 5) = 𝑥 4 − 7𝑥 2 + 10
⇒ 𝑥 4 + 𝑎𝑥 2 + 𝑏 = 𝑥 4 − 7𝑥 2 + 10
𝑎 = −7
⇒ {
𝑏 = 10
Então,
|𝑎 + 𝑏| = |−7 + 10| = |3| = 3
Gabarito: “b”.
16. (EEAR/2001)

Uma das raízes da equação 𝒙𝟑 − 𝟏𝟐𝒙𝟐 + 𝟒𝟒𝒙 − 𝟒𝟖 = 𝟎 é a soma das outras duas. A maior raiz
dessa equação é

a) 7

b) 6

c) 4

d) 2
Comentários
Sejam as raízes 𝑎, 𝑏 e 𝑐, tal que 𝑐 = 𝑎 + 𝑏
Pelas Relações de Girard:
−12
𝑎+𝑏+𝑐 =− = 12
1
𝑎 + 𝑏 + 𝑎 + 𝑏 = 2(𝑎 + 𝑏) = 12
2 ⋅ 𝑐 = 12 ⇒ 𝑐 = 6
⇒ 𝑎 + 𝑏 = 6 (𝑒𝑞. 1)
Também das relações de Girard:
−48
𝑎⋅𝑏⋅𝑐 =− = 48
1
𝑎 ⋅ 𝑏 ⋅ 6 = 48
⇒ 𝑎 ⋅ 𝑏 = 8 (𝑒𝑞. 2)
Isolando 𝑏 em 𝑒𝑞. 1 e substituindo em 𝑒𝑞. 2, obtemos:

AULA 07 – EQUAÇÕES ALGÉBRICAS 49


Prof. Victor So

𝑎 ⋅ (6 − 𝑎 ) = 8
𝑎2 − 6𝑎 + 8 = 0
Resolvendo a equação de 2º grau em 𝑎, obtemos:
𝑎1 = 2 𝑏1 = 6 − 2 = 4
{ ⇒ {
𝑎2 = 4 𝑏2 = 6 − 4 = 2
Posto isso, as raízes do polinômio são 6, 4 e 2. Portanto, a maior raiz vale 6.
Gabarito: “b”.
17. (EEAR/2003)
𝟑𝟓
Se a diferença entre os quadrados das raízes da equação 𝟑𝒙𝟐 − 𝟕𝒙 + 𝒄 = 𝟎 é , então o valor de 𝒄
𝟗
é
𝟐
a) − 𝟑

b) −𝟐
𝟐
c) 𝟑

d) 𝟐
Comentários
Sejam 𝑎 e 𝑏 raízes, com 𝑏 > 𝑎:
35
𝑏2 − 𝑎2 =
9
35
(𝑏 + 𝑎 ) ⋅ (𝑏 − 𝑎 ) =
9
Mas, decorre das Relações De Girard:
−7 7
𝑎+𝑏 =− =
3 3
𝑐
𝑎⋅𝑏 =
3
Então,
7 35 5
( ) ⋅ (𝑏 − 𝑎 ) = ⇒ 𝑏−𝑎 =
3 9 3
Sendo, assim, temos o seguinte sistema:
7
𝑏+𝑎 =
{ 3
5
𝑏−𝑎 =
3

AULA 07 – EQUAÇÕES ALGÉBRICAS 50


Prof. Victor So

1
Resolvendo o sistema, obtemos 𝑎 = e 𝑏 = 2. Mas sabemos que,
3
𝑐
𝑎⋅𝑏 =
3
1
𝑐 = 3 ⋅ ( ) ⋅ (2) = 2
3
Gabarito: “d”.
18. (ESA/2017)

Se 𝟐 + 𝟑𝒊 é raiz de uma equação algébrica 𝑷(𝒙) = 𝟎, de coeficientes reais, então podemos afirmar
que:

a) 𝟑 − 𝟐𝒊 também é raiz da mesma equação.

b) 𝟐 − 𝟑𝒊 também é raiz da mesma equação.

c) 𝟐 também é raiz da mesma equação.

d) −𝟑𝒊 também é raiz da mesma equação.

e) 𝟑 + 𝟐𝒊 também é raiz da mesma equação.


Comentários
Sendo, a equação algébrica de coeficientes reais, as raízes complexas, vem aos pares, ou
seja, se 𝑧 é raiz 𝑧̅ também é. Sendo assim, se 2 + 3𝑖 é raiz, ̅̅̅̅̅̅̅̅
2 + 3𝑖 = 2 − 3𝑖 também é raiz.
Gabarito: “b”.
19. (EsPCEx/2011)

Seja a função complexa 𝑷(𝒙) = 𝟐𝒙𝟑 − 𝟗𝒙𝟐 + 𝟏𝟒𝒙 − 𝟓. Sabendo-se que 𝟐 + 𝒊 é raiz de 𝑷, o intervalo
𝑰 de números reais que faz 𝑷(𝒙) < 𝟎, para todo 𝒙 ∈ 𝑰 é
𝟏
a) ] − ∞, [
𝟐

b) ]𝟎, 𝟏[
𝟏
c) ] 𝟒 , 𝟐[

d) ]𝟎, +∞[
𝟏 𝟑
e) ] − 𝟒 , [
𝟒

Comentários
Sendo os coeficientes de 𝑃(𝑥) reais, temos que se 2 + 𝑖 é raiz, então 2 − 𝑖 também é raiz.
Portanto, 𝑃(𝑥) é divisível por (𝑥 − 2 − 𝑖) e por (𝑥 − 2 + 𝑖), logo, 𝑃(𝑥) é divisível por
(𝑥 − 2 − 𝑖) ⋅ (𝑥 − 2 + 𝑖) = ((𝑥 − 2)2 − (𝑖)2 ) =
= ((𝑥 2 − 4𝑥 + 4) + 1) = 𝑥 2 − 4𝑥 + 5

AULA 07 – EQUAÇÕES ALGÉBRICAS 51


Prof. Victor So

𝑃(𝑥) é divisível por 𝑥 2 − 4𝑥 + 5. Sendo assim, aplicaremos a divisão algébrica de


polinômios.

Logo,
𝑃(𝑥) = 2𝑥 3 − 9𝑥 2 + 14𝑥 − 5 = (𝑥 2 − 4𝑥 + 5) ⋅ (2𝑥 − 1) =
1
𝑃(𝑥) = 2(𝑥 2 − 4𝑥 + 5) ⋅ (𝑥 − )
2
1
Ou seja, a única raiz real de 𝑃(𝑥) é 𝑥 = , fazendo o estudo de sinal para 𝑃(𝑥) < 0,
2
obtemos:

1 1
Logo, 𝑃(𝑥) < 0 ⇒ 𝑥 < ∴ 𝑥 ∈ ] − ∞, [
2 2

Gabarito: “a”.

8. QUESTÕES NÍVEL 2
20. (AFA/2020)

Considere os polinômios na variável 𝒙:

𝑨(𝒙) = 𝒙𝟑 + (𝟑𝒎𝟑 − 𝟒𝒎)𝒙𝟐 − 𝟐, sendo 𝒎 ∈ ℚ; e

𝑩(𝒙) = 𝒙𝟐 − 𝟐𝒙 + 𝟏

AULA 07 – EQUAÇÕES ALGÉBRICAS 52


Prof. Victor So

Os gráficos de 𝑨(𝒙) e 𝑩(𝒙) possuem apenas um ponto comum sobre o eixo das abcissas.

É correto afirmar que:

a) o produto e a soma das raízes imaginárias de 𝑨(𝒙) são números conjugados.

b) os afixos das raízes de 𝑨(𝒙) formam um triângulo equilátero.

c) as raízes de 𝑨(𝒙) possuem argumentos que NÃO formam uma Progressão Aritmética.

d) todas as raízes de 𝑨(𝒙) possuem o mesmo módulo.

21. (AFA/2019)
𝒂
Considere 𝒂 ∈ ℝ e os polinômios 𝑷(𝒙) = 𝟐 𝒙𝟔 − 𝟐𝟔𝒙𝟑 − 𝟐𝟕 e 𝑨(𝒙) = 𝟐𝒙𝟐 + 𝟒𝒙 + 𝒂, tais que seus
gráficos se intersectam em um único ponto de ordenada nula.

Sabendo também que, graficamente, 𝑨(𝒙) tangencia o eixo ⃡𝑶𝒙 analise as afirmativas abaixo e
escreva V para verdadeira e F para falsa.
⃡ em dois pontos.
( ) O gráfico de 𝑷(𝒙) corta o eixo 𝑶𝒙

( ) Os afixos das raízes de 𝑷(𝒙) que possuem menor módulo formam um triângulo cujo perímetro
mede 𝟑√𝟑 unidades de comprimento.

( ) A soma das raízes imaginárias de 𝑷(𝒙) é igual a −𝟐.

A sequência correta é

a) V-V-V
b) V-F-F

c) F-V-F

d) F-V-V

22. (AFA/2017)

O polinômio 𝑷(𝒙) = 𝒙𝟑 + 𝒎𝒙𝟐 + 𝒏𝒙 + 𝟏𝟐 é tal que 𝑷(𝒙) = 𝟎 admite as raízes 𝒙𝟏 , 𝒙𝟐 , 𝒆 𝒙𝟑 . Se 𝒙𝟏 ⋅


𝒙𝟐 = −𝟑 e 𝒙𝟐 + 𝒙𝟑 = 𝟓, então é correto afirmar que

a) 𝑷(𝒎) = 𝟎

b) 𝒎 − 𝒏 = −𝟏𝟑

c) 𝒎 ⋅ 𝒏 = 𝟐𝟎
d) 𝒏 − 𝟐𝒎 = −𝟕

AULA 07 – EQUAÇÕES ALGÉBRICAS 53


Prof. Victor So

23. (AFA/2014)
𝒎 𝒑 𝒒
A equação 𝒙𝟑 − 𝟒𝒙𝟐 + 𝟓𝒙 + 𝟑 = 𝟎 possui as raízes 𝒎, 𝒑 e 𝒒. O valor da expressão 𝒑𝒒 + 𝒎𝒒 + 𝒎𝒑 é

a) −𝟐

𝒃) −3

𝒄) 𝟐
d) 3

24. (AFA/2013)

As raízes da equação algébrica 𝟐𝒙𝟑 − 𝒂𝒙𝟐 + 𝒃𝒙 + 𝟓𝟒 = 𝟎 formam uma progressão geométrica.


𝒂
Se 𝒂, 𝒃 ∈ ℝ, 𝒃 ≠ 𝟎, então 𝒃 é igual a
𝟐
a) 𝟑

b) 𝟑
𝟑
c) − 𝟐
𝟏
d) − 𝟑

25. (AFA/2012)

Sejam (𝟏, 𝒂𝟐 , 𝒂𝟑 , 𝒂𝟒 ) e (𝟏, 𝒃𝟐 , 𝒃𝟑 , 𝒃𝟒 ) uma progressão aritmética e uma progressão geométrica,


respectivamente, ambas com a mesma soma dos termos e ambas crescentes. Se a razão 𝒓 da
progressão aritmética é o dobro da razão 𝒒 da progressão geométrica, então, o produto 𝒓 ⋅ 𝒒 é igual
a

a) 14

b) 18

c) 21

d) 24

26. (EFOMM/2017)

Considere a equação 𝒙𝟒 − 𝟐𝒂𝒙𝟑 + 𝟗𝒂𝒙𝟐 − 𝟔𝒂𝒙 + 𝟗𝒂 = 𝟎. Sabendo que 𝒂 é raiz dupla dessa
equação e não é nulo, determine o valor de 𝒂.

AULA 07 – EQUAÇÕES ALGÉBRICAS 54


Prof. Victor So

a) 𝒂 = −𝟏

b) 𝒂 = 𝟏

c) 𝒂 = 𝟐

d) 𝒂 = 𝟑

e) 𝒂 = 𝟒

27. (EFOMM/2017)

Sobre uma equação linear de grau 𝒏 é INCORRETO afirmar que

a) terá 𝒏 raízes complexas.

b) se 𝒏 for ímpar, sempre terá, ao menos, uma raiz real.

c) se um número complexo 𝒛 = 𝒂 + 𝒃𝒊, 𝒃 ≠ 𝟎 for raiz, então seu conjugado também o será.

d) a equação não pode ter raízes repetidas.

e) uma equação acima de grau 𝟒 pode ter todas as raízes reais.

28. (EFOMM/2016)

Seja o polinômio 𝒑(𝒙) = 𝒙𝟔 − 𝟐𝟔𝒙𝟒 − 𝟑𝟐𝒙𝟑 − 𝟏𝟒𝟕𝒙𝟐 − 𝟗𝟔𝒙 − 𝟏𝟖𝟎.

A respeito das raízes da equação 𝒑(𝒙) = 𝟎, podemos afirmar que

a) todas as raízes são reais.

b) somente duas raízes são reais, sendo ela distintas.

c) somente duas raízes são reais, sendo elas iguais.

d) somente quatro raízes são reais, sendo todas elas distintas.

e) nenhuma raiz é real.

29. (EFOMM/2016)

A solução do sistema:
𝒙+𝒚+𝒛+𝒘=𝟕
𝒙𝒚 + 𝒙𝒛 + 𝒙𝒘 + 𝒚𝒛 + 𝒚𝒘 + 𝒛𝒘 = 𝟒
{
𝒘𝒚𝒛 + 𝒙𝒚𝒘 + 𝒙𝒛𝒘 + 𝒚𝒛𝒘 = 𝟔
𝒘𝒚𝒛𝒘 = 𝟏
pode ser representada pelas raízes do polinômio:

AULA 07 – EQUAÇÕES ALGÉBRICAS 55


Prof. Victor So

a) 𝒙𝟑 + 𝟔𝒙𝟐 + 𝟒𝒙 + 𝟕

b) 𝒙𝟑 + 𝟔𝒙𝟐 + 𝟒𝒙 − 𝟕

c) 𝟐𝒙𝟒 − 𝟏𝟒𝒙𝟑 + 𝟖𝒙𝟐 − 𝟏𝟐𝒙 + 𝟐

d) 𝟕𝒙𝟒 − 𝟒𝒙𝟑 + 𝟔𝒙𝟐 + 𝒙

e) 𝒙𝟒 + 𝟕𝒙𝟑 + 𝟒𝒙𝟐 + 𝟔𝒙

30. (EFOMM/2015)

Assinale a alternativa que apresenta o polinômio 𝑷 de grau mínimo, com coeficiente reais, de modo
que 𝑷(𝒊) = 𝟐 e 𝑷(𝟏 + 𝒊) = 𝟎.

a) 𝟏/𝟓 (𝒙𝟐 − 𝟐𝒙 + 𝟐)

b) 𝟐/𝟓 (𝒙𝟐 − 𝟐𝒙 + 𝟐)

c) 𝟐/𝟓 (𝒙𝟐 − 𝟐𝒙 + 𝟑)

d) 𝟏/𝟓 (𝒙𝟐 − 𝟐𝒙𝟐 + 𝟐)

e) 𝟐/𝟑 (𝒙𝟐 − 𝟐𝒙 + 𝟑)

31. (EFOMM/2013)

𝑷(𝒙) é um polinômio de coeficientes reais e menor grau com as propriedades abaixo:


• os números 𝒓𝟏 = 𝟏, 𝒓𝟐 = 𝒊 e 𝒓𝟑 = 𝟏 − 𝒊 são raízes da equação 𝑷(𝒙) = 𝟎;
• 𝑷(𝟎) = −𝟒.

Então, 𝑷(−𝟏) é igual a:

a) 𝟒.
b) −𝟐.

c) −𝟏𝟎.

d) 𝟏𝟎.

e) −𝟒𝟎.

32. (EFOMM/2009)

Após a determinação dos valores numéricos: 𝒑(−𝟏), 𝒑(𝟎) e 𝒑(𝟏), verifica-se que o polinômio
𝒑(𝒙) = 𝒙𝟑 + 𝒙𝟐 − 𝒙 − 𝟎, 𝟓 tem

AULA 07 – EQUAÇÕES ALGÉBRICAS 56


Prof. Victor So

a) apenas uma raiz real.

b) apenas duas raízes reais.

c) três raízes reais, todas de mesmo sinal.

d) três raízes reais, duas positivas e uma negativa.

e) três raízes reais, duas negativas e uma positiva.

33. (EFOMM/2005)

Determine as raízes na equação 𝒙𝟑 − 𝟗𝒙𝟐 + 𝟐𝟔𝒙 − 𝟐𝟒 = 𝟎, sabendo que elas estão em P.A.

a) 𝑺 = {𝟏, 𝟐, 𝟑}

b) 𝑺 = {𝟏, 𝟑, 𝟓}

c) 𝑺 = {𝟐, 𝟒, 𝟔}

d) 𝑺 = {𝟐, 𝟑, 𝟒}

e) 𝑺 = {𝟑, 𝟓, 𝟕}

34. (EFOMM/2005)

Determine as raízes na equação 𝒙𝟑 − 𝟏𝟒𝒙𝟐 + 𝟓𝟔𝒙 − 𝟔𝟒 = 𝟎, sabendo que elas estão em P.G.

a) 𝑺 = {𝟏, 𝟐, 𝟒}

b) 𝑺 = {𝟐, 𝟑, 𝟒}

c) 𝑺 = {𝟐, 𝟑, 𝟔}

d) 𝑺 = {𝟐, 𝟒, 𝟔}

e) 𝑺 = {𝟐, 𝟒, 𝟖}

35. (Escola Naval/2014)

Considere 𝑷(𝒙) = (𝒎 − 𝟒)(𝒎𝟐 + 𝟒)𝒙𝟓 + 𝒙𝟐 + 𝒌𝒙 + 𝟏 um polinômio na variável real 𝒙, em que 𝒎


e 𝒌 são constantes reais. Quais os valores das constantes 𝒎 e 𝒌 para que 𝑷(𝒙) não admita raiz real?

a) 𝒎 = 𝟒 e −𝟐 < 𝒌 < 𝟐

b) 𝒎 = −𝟒 e 𝒌 > 𝟐

c) 𝒎 = −𝟐 e −𝟐 < 𝒌 < 𝟐
d) 𝒎 = 𝟒 e |𝒌| > 𝟐

AULA 07 – EQUAÇÕES ALGÉBRICAS 57


Prof. Victor So

e) 𝒎 = −𝟐 e 𝒌 > −𝟐

36. (Escola Naval/2013)

Sabendo que 𝒊√𝟑 é uma das raízes da equação 𝒙𝟒 + 𝒙𝟑 + 𝟐𝒙𝟐 + 𝟑𝒙 − 𝟑 = 𝟎, a soma de todas as
raízes desta equação é

a) −𝟐𝒊√𝟑

b) 𝟒𝒊√𝟑

c) 𝟎

d) −𝟏

e) −𝟐

37. (EN/2013)
Sejam 𝐹 (𝑥) = 𝑥 3 + 𝑎𝑥 + 𝑏 e 𝐺 (𝑥) = 2𝑥 2 + 2𝑥 − 6 dois polinômios na variável real 𝑥, com
𝐹(𝑥)
𝑎 𝑒 𝑏 números reais. Qual valor de (𝑎 + 𝑏) para que a divisão seja exata?
𝐺(𝑥)

a) −2
b) −1
c) 0
d) 1
e) 2

38. (EN/2022)

Suponha que o conjunto solução da equação 𝟓𝒙𝟑 − 𝟒𝒙𝟐 + 𝟕𝒙 − 𝟐 = 𝟎 é {𝒙𝟏 , 𝒙𝟐 , 𝒙𝟑 }. Se a equação


polinomial 𝑷(𝒙) = 𝟎 apresenta {𝟓𝒙𝟏 , 𝟓𝒙𝟐 , 𝟓𝒙𝟑 } como conjunto solução, assinale a opção que
apresenta a soma dos coeficientes de 𝑷(𝒙).
𝟏𝟖
a) − 𝟐𝟓
𝟏𝟗
b) − 𝟏𝟓
𝟐𝟏
c) − 𝟐𝟎
𝟏𝟕
d) − 𝟑𝟓
𝟏𝟕
e) − 𝟐𝟎

AULA 07 – EQUAÇÕES ALGÉBRICAS 58


Prof. Victor So

39. (EN/2022)

Considere o sistema abaixo:


𝒚𝟐 + 𝒖𝟐 + 𝒗𝟐 + 𝒘𝟐 = 𝟒𝒙 − 𝟏
𝒙𝟐 + 𝒖𝟐 + 𝒗𝟐 + 𝒘𝟐 = 𝟒𝒚 − 𝟏
𝒙𝟐 + 𝒚𝟐 + 𝒗𝟐 + 𝒘𝟐 = 𝟒𝒖 − 𝟏
𝒙𝟐 + 𝒚𝟐 + 𝒖𝟐 + 𝒘𝟐 = 𝟒𝒗 − 𝟏
{𝒙𝟐 + 𝒚𝟐 + 𝒖𝟐 + 𝒗𝟐 = 𝟒𝒘 − 𝟏
Se 𝒙 = 𝒂, 𝒚 = 𝒃, 𝒖 = 𝒄, 𝒗 = 𝒅 e 𝒘 = 𝒆 constituem a solução do sistema, assinale a opção que
apresenta a soma 𝒂 + 𝒃 + 𝒄 + 𝒅 + 𝒆.
𝟓
a) 𝟐
𝟐
b) 𝟕
𝟏
c) 𝟕
𝟏
d) 𝟒
𝟐
e) 𝟑

GABARITO
20. c
21. a
22. d
23. a
24. d
25. b
26. d
27. Anulada
28. b
29. c
30. Anulada
31. e
32. e
33. d
34. e
35. a
36. d
37. b
38. a

AULA 07 – EQUAÇÕES ALGÉBRICAS 59


Prof. Victor So

39. a

RESOLUÇÃO
20. (AFA/2020)

Considere os polinômios na variável 𝒙:

𝑨(𝒙) = 𝒙𝟑 + (𝟑𝒎𝟑 − 𝟒𝒎)𝒙𝟐 − 𝟐, sendo 𝒎 ∈ ℚ; e

𝑩(𝒙) = 𝒙𝟐 − 𝟐𝒙 + 𝟏
Os gráficos de 𝑨(𝒙) e 𝑩(𝒙) possuem apenas um ponto comum sobre o eixo das abcissas.

É correto afirmar que:

a) o produto e a soma das raízes imaginárias de 𝑨(𝒙) são números conjugados.


b) os afixos das raízes de 𝑨(𝒙) formam um triângulo equilátero.

c) as raízes de 𝑨(𝒙) possuem argumentos que NÃO formam uma Progressão Aritmética.

d) todas as raízes de 𝑨(𝒙) possuem o mesmo módulo.


Comentários
Se 𝐴 𝑒 𝐵 possuem apenas um ponto em comum sobre o eixo das abcissas, significa que
esse ponto é do tipo (𝑥, 0) e, portanto, é raiz de ambos polinômios. Mas a raiz de 𝐵 é única:
𝐵(𝑥) = 0 ⇒ 𝑥 2 − 2𝑥 + 1 = 0 ⇒ (𝑥 − 1)2 = 0 ⇒ 𝑥 = 1
Portanto, 𝑥 = 1 é raiz de 𝐴(𝑥) = 0 também. Substituindo 𝑥 = 1 em A:
13 + (3𝑚2 − 4𝑚)12 − 2 = 0 ⇒ 3𝑚2 − 4𝑚 − 1 = 0 ⇒ 3𝑚2 − 4𝑚 = 1
⇒ 𝐴(𝑥) = 𝑥 3 + 𝑥 2 − 2 = 𝑥 3 − 𝑥 2 + 2𝑥 2 − 2 = 𝑥 2 (𝑥 − 1) + 2(𝑥 2 − 1)
𝐴(𝑥) = 𝑥 2 (𝑥 − 1) + 2(𝑥 − 1)(𝑥 + 1) = (𝑥 − 1)(𝑥 2 + 2𝑥 + 2)
Assim, as outras raízes de 𝐴(𝑥) são as raízes de 𝑥 2 + 2𝑥 + 2:
𝑥 2 + 2𝑥 + 2 = 0 ⇒ 𝑥 2 + 2𝑥 + 1 = −1 ⇒ (𝑥 + 1)2 = 𝑖 2 ⇒ 𝑥 + 1 = ±𝑖
⇒ 𝑥 = −1 + 𝑖 𝑜𝑢 𝑥 = −1 − 𝑖
3𝜋 5𝜋
Portanto, vemos que os argumentos das raízes são: 0(𝑥 = 1), (𝑥 = −1 + 𝑖) 𝑒 (𝑥 =
4 4
−1 − 𝑖). Eles não formam uma progressão aritmética. As outras estão erradas pois o produto e a
soma das raízes imaginárias de A são números inteiros; os afixos das raízes de 𝐴 formam a reta
𝑥 = 1; os módulos distintos entre as raízes são 1 𝑒 √2.

Gabarito: “c”.

AULA 07 – EQUAÇÕES ALGÉBRICAS 60


Prof. Victor So

21. (AFA/2019)
𝒂
Considere 𝒂 ∈ ℝ e os polinômios 𝑷(𝒙) = 𝟐 𝒙𝟔 − 𝟐𝟔𝒙𝟑 − 𝟐𝟕 e 𝑨(𝒙) = 𝟐𝒙𝟐 + 𝟒𝒙 + 𝒂, tais que seus
gráficos se intersectam em um único ponto de ordenada nula.

Sabendo também que, graficamente, 𝑨(𝒙) tangencia o eixo ⃡𝑶𝒙 analise as afirmativas abaixo e
escreva V para verdadeira e F para falsa.
⃡ em dois pontos.
( ) O gráfico de 𝑷(𝒙) corta o eixo 𝑶𝒙

( ) Os afixos das raízes de 𝑷(𝒙) que possuem menor módulo formam um triângulo cujo perímetro
mede 𝟑√𝟑 unidades de comprimento.

( ) A soma das raízes imaginárias de 𝑷(𝒙) é igual a −𝟐.

A sequência correta é

a) V-V-V

b) V-F-F

c) F-V-F

d) F-V-V
Comentários
Se 𝐴 𝑒 𝑃 tem um ponto em comum de ordenada nula, significa que esse ponto é (𝑥0 , 0).
Veja que, então esse ponto é raiz dos dois polinômios.
Mais que isso, como 𝐴(𝑥) graficamente tangencia o eixo 𝑂𝑋, isto é só, o toca em um único
ponto, então esse ponto só pode ser (𝑥0 , 0) e, portanto, 𝑥0 é raiz dupla de 𝐴(𝑥) = 0.
𝐴(𝑥0 ) = 0 ⇒ 2𝑥02 + 4𝑥0 + 𝑎 = 0
Como possui raiz dupla, entãp Δ = 0:
Δ = 42 − 4 ⋅ 2 ⋅ 𝑎 = 0 ⇒ 16 − 8𝑎 = 0 ⇒ 𝑎 = 2
−4
⇒ 𝑥0 = = −1
2⋅2
Assim, 𝑃(𝑥) = 𝑥 6 − 26𝑥 3 − 27. Chamando 𝑥 3 = 𝑦:
𝑦 2 − 26𝑦 − 27 = 0 ⇒ 𝑦 2 + 𝑦 − 27𝑦 − 27 = 0 ⇒ 𝑦(𝑦 + 1) − 27(𝑦 + 1) = 0
⇒ (𝑦 + 1)(𝑦 − 27) = 0 ⇒ (𝑥 3 + 1)(𝑥 3 − 27) = 0
Portanto, as raízes de 𝑃(𝑥) são tais que:
𝑥 3 = −1 𝑜𝑢 𝑥 3 = 27
Assim, raízes reais são apenas 𝑥 = −1 ou 𝑥 = 3 e as demais são raízes complexas (duas
são raízes cúbicas de -1 e outras duas são raízes cúbicas de 3). Portanto a primeira afirmação é
verdadeira, pois 𝑃(𝑥) tem duas raízes reais.

AULA 07 – EQUAÇÕES ALGÉBRICAS 61


Prof. Victor So

𝑖𝜋 𝑖5𝜋
3
As raízes de menor módulo são as 𝑥 = √−1 = 𝑒 3 𝑜𝑢 𝑒 𝑖𝜋 𝑜𝑢 𝑒 3 . Elas de fato formam um
triângulo equilátero no plano de Argand-Gauss. O módulo delas é igual à distância do incentro ao
vértice, que é dois terços da altura de um triângulo equilátero de lado 𝑙:
2 𝑙√3 𝑙√3
1= ⋅( )⇒1= ⇒ 𝑙 = √3
3 2 3
Portanto, se o lado mede √3, então o perímetro realmente mede 3𝑙 = 3√3. Assim, a
segunda informação é verdadeira.
Para saber a soma das raízes imaginárias de 𝑃(𝑥) vamos à sua fatoração já feita:
𝑃(𝑥) = (𝑥 3 + 1)(𝑥 3 − 27) = (𝑥 + 1)(𝑥 2 − 𝑥 + 1)(𝑥 − 3)(𝑥 2 + 3𝑥 + 9)
Assim, veja que a soma das raízes de (𝑥 2 − 𝑥 + 1) é 1, e a soma das raízes de 𝑥 2 + 3𝑥 + 9
é −3. Portanto, a soma de todas as raízes complexas é 1 − 3 = −2. Logo, todas as afirmativas
são verdadeiras.

Gabarito: “a”.
22. (AFA/2017)

O polinômio 𝑷(𝒙) = 𝒙𝟑 + 𝒎𝒙𝟐 + 𝒏𝒙 + 𝟏𝟐 é tal que 𝑷(𝒙) = 𝟎 admite as raízes 𝒙𝟏 , 𝒙𝟐 , 𝒆 𝒙𝟑 . Se 𝒙𝟏 ⋅


𝒙𝟐 = −𝟑 e 𝒙𝟐 + 𝒙𝟑 = 𝟓, então é correto afirmar que

a) 𝑷(𝒎) = 𝟎

b) 𝒎 − 𝒏 = −𝟏𝟑

c) 𝒎 ⋅ 𝒏 = 𝟐𝟎
d) 𝒏 − 𝟐𝒎 = −𝟕
Comentários
Sabemos pelas Relações de Girard que:
12
𝑥1 𝑥2 𝑥 3 = − ⇒ −3 ⋅ 𝑥3 = −12 ⇒ 𝑥3 = 4
1
É dado que 𝑥2 + 𝑥3 = 5 ⇒ 𝑥2 + 4 = 5 ⇒ 𝑥2 = 1
⇒ 𝑥1 𝑥2 = −3 ⇒ 𝑥1 = −3
Ainda, por Girard:
𝑚
𝑥1 + 𝑥2 + 𝑥3 = − ⇒ −𝑚 = 2 ⇒ 𝑚 = −2
1
𝑛
𝑥1 𝑥2 + 𝑥1 𝑥3 + 𝑥2 𝑥3 =
⇒ 𝑛 = −3 − 12 + 4 ⇒ 𝑛 = −11
1
Portanto, 𝑚 − 2𝑚 = −11 − 2(−2) = −11 + 4 = −7.

AULA 07 – EQUAÇÕES ALGÉBRICAS 62


Prof. Victor So

Gabarito: “d”.
23. (AFA/2014)
𝒎 𝒑 𝒒
A equação 𝒙𝟑 − 𝟒𝒙𝟐 + 𝟓𝒙 + 𝟑 = 𝟎 possui as raízes 𝒎, 𝒑 e 𝒒. O valor da expressão 𝒑𝒒 + 𝒎𝒒 + 𝒎𝒑 é

a) −𝟐

𝒃) −3

𝒄) 𝟐
d) 3
Comentários
Antes de sair procurando as raízes da equação, vamos analisar a expressão pedida: dadas
as raízes 𝑚, 𝑝 𝑒 𝑞:
𝑚 𝑝 𝑞 𝑚2 + 𝑝2 + 𝑞²
+ + =
𝑝𝑞 𝑚𝑞 𝑚𝑝 𝑚𝑝𝑞
Lembrando que:
(𝑚 + 𝑝 + 𝑞)2 = 𝑚2 + 𝑝2 + 𝑞2 + 2(𝑚𝑝 + 𝑚𝑞 + 𝑝𝑞)
⇒ 𝑚2 + 𝑝2 + 𝑞2 = (𝑚 + 𝑝 + 𝑞)2 − 2(𝑚𝑝 + 𝑚𝑞 + 𝑝𝑞)
Portanto, a expressão pedida fica:
𝑚 𝑝 𝑞 𝑚2 + 𝑝2 + 𝑞² (𝑚 + 𝑝 + 𝑞)2 − 2(𝑚𝑝 + 𝑚𝑞 + 𝑝𝑞)
+ + = =
𝑝𝑞 𝑚𝑞 𝑚𝑝 𝑚𝑝𝑞 𝑚𝑝𝑞
Podemos achar cada termo da expressão acima pelas relações de Girard:
𝑏 −4
𝑚+𝑝+𝑞 =− =− ⇒ 𝑚+𝑝+𝑞 =4
𝑎 1
𝑐 5
𝑚𝑝 + 𝑚𝑞 + 𝑝𝑞 = = ⇒ 𝑚𝑝 + 𝑚𝑞 + 𝑝𝑞 = 5
𝑎 1
𝑑 3
𝑚𝑝𝑞 = − = − ⇒ 𝑚𝑝𝑞 = −3
𝑎 1
Assim, a expressão pedida é:
𝑚 𝑝 𝑞 (𝑚 + 𝑝 + 𝑞)2 − 2(𝑚𝑝 + 𝑚𝑞 + 𝑝𝑞) 42 − 2 ⋅ 5 16 − 10 6
+ + = = = =−
𝑝𝑞 𝑚𝑞 𝑚𝑝 𝑚𝑝𝑞 −3 −3 3
𝑚 𝑝 𝑞
⇒ + + = −2
𝑝𝑞 𝑚𝑞 𝑚𝑝
Gabarito: “a”.
24. (AFA/2013)

AULA 07 – EQUAÇÕES ALGÉBRICAS 63


Prof. Victor So

As raízes da equação algébrica 𝟐𝒙𝟑 − 𝒂𝒙𝟐 + 𝒃𝒙 + 𝟓𝟒 = 𝟎 formam uma progressão geométrica.


𝒂
Se 𝒂, 𝒃 ∈ ℝ, 𝒃 ≠ 𝟎, então 𝒃 é igual a
𝟐
a) 𝟑

b) 𝟑
𝟑
c) − 𝟐
𝟏
d) − 𝟑

Comentários
Se as raízes formam uma progressão geométrica, então elas podem ser representadas por:
𝑥, 𝑥𝑞, 𝑥𝑞2 , em que 𝑞 é a razão dessa PG. Pelas relações de Girard, o produto das raízes é:
𝑑 54 3
𝑥 ⋅ 𝑥𝑞 ⋅ 𝑥𝑞2 = − =− ⇒ 𝑥 3 𝑞3 = −27 ⇒ 𝑥𝑞 = √−27 = −3
𝑎 2
Ainda, por Girard:
𝑎
𝑥 + 𝑥𝑞 + 𝑥𝑞2 =
2
𝑏
𝑥 2 𝑞 + 𝑥 2 𝑞2 + 𝑥 2 𝑞3 =
2
Dividindo ambas equações acima:
𝑥 (1 + 𝑞 + 𝑞 2 ) 𝑎 𝑎 1 1
2 2
= ⇒ = =−
𝑥 𝑞 (1 + 𝑞 + 𝑞 ) 𝑏 𝑏 𝑥𝑞 3
Gabarito: “d”.
25. (AFA/2012)

Sejam (𝟏, 𝒂𝟐 , 𝒂𝟑 , 𝒂𝟒 ) e (𝟏, 𝒃𝟐 , 𝒃𝟑 , 𝒃𝟒 ) uma progressão aritmética e uma progressão geométrica,


respectivamente, ambas com a mesma soma dos termos e ambas crescentes. Se a razão 𝒓 da
progressão aritmética é o dobro da razão 𝒒 da progressão geométrica, então, o produto 𝒓 ⋅ 𝒒 é igual
a

a) 14

b) 18

c) 21

d) 24
Comentários
Do enunciado, temos 𝑟 = 2𝑞. Como a soma dos termos das duas sequências é igual, temos:

AULA 07 – EQUAÇÕES ALGÉBRICAS 64


Prof. Victor So

(1 + 𝑎4 )4
𝑃𝐴 → 𝑆𝑃𝐴 = = 2(1 + (1 + 3𝑟)) = 4 + 6𝑟 = 4 + 12𝑞
2
𝑎1 (𝑞4 − 1) (𝑞 − 1)(𝑞 + 1)(𝑞2 + 1)
𝑃𝐺 → 𝑆𝑃𝐺 = =
𝑞−1 𝑞−1
A fatoração acima foi feita usando diferença de quadrados.
Sendo ambas crescentes, temos 𝑞 ≠ 1, logo:
𝑆𝑃𝐺 = (𝑞 + 1)(𝑞2 + 1)
Para 𝑆𝑃𝐴 = 𝑆𝑃𝐺 :
4 + 12𝑞 = (𝑞 + 1)(𝑞2 + 1)
4 + 12𝑞 = 𝑞3 + 𝑞 + 𝑞2 + 1
𝑞3 + 𝑞2 − 11𝑞 − 3 = 0
Veja que, por inspeção, 𝑞 = 3 é razão da equação acima. Fatorando em busca do fator 𝑞 −
3:
𝑞3 − 3𝑞2 + 4𝑞2 − 12𝑞 + 𝑞 − 3 = 0 ⇒ 𝑞2 (𝑞 − 3) + 4𝑞(𝑞 − 3) + (𝑞 − 3) = 0
⇒ (𝑞 − 3)(𝑞2 + 4𝑞 + 1) = 0
Se resolvermos 𝑞2 + 4𝑞 + 1 = 0 ⇒ 𝑞2 + 4𝑞 + 4 = 3 ⇒ (𝑞 + 2)2 = 3 ⇒ 𝑞 = −2 ± √3
Veja que há apenas um valor de 𝑞 > 0 ⇒ 𝑞 = 3. Assim, 𝑟 = 2𝑞 = 6. Logo:
𝑟𝑞 = 6 ⋅ 3 = 18

Gabarito: “b”.
26. (EFOMM/2017)

Considere a equação 𝒙𝟒 − 𝟐𝒂𝒙𝟑 + 𝟗𝒂𝒙𝟐 − 𝟔𝒂𝒙 + 𝟗𝒂 = 𝟎. Sabendo que 𝒂 é raiz dupla dessa
equação e não é nulo, determine o valor de 𝒂.

a) 𝒂 = −𝟏

b) 𝒂 = 𝟏

c) 𝒂 = 𝟐

d) 𝒂 = 𝟑

e) 𝒂 = 𝟒
Comentários
Se 𝑎 é raiz dupla da equação acima, se as outras duas equações são 𝑚 𝑒 𝑛, então pelas
Relações de Girard:

AULA 07 – EQUAÇÕES ALGÉBRICAS 65


Prof. Victor So

−2𝑎
𝑎+𝑎+𝑚+𝑛 =− = 2𝑎 ⇒ 𝑚 + 𝑛 = 0 ⇒ 𝑛 = −𝑚
1
Assim, as raízes são 𝑎, 𝑎, 𝑚 𝑒 − 𝑚.
Ainda, por Girard, o produto das raízes:
−𝑚2 𝑎2 = 9𝑎 ⇒ −𝑚2 𝑎 = 9
O produto das raízes, 3 a 3 delas:
−6𝑎
𝑎2 𝑚 − 𝑎2 𝑚 − 2𝑚2 𝑎 = − = 6𝑎 ⇒ −2𝑚2 𝑎 = 6𝑎 ⇒ 6𝑎 = 18 ⇒ 𝑎 = 3
1
Gabarito: “d”.
27. (EFOMM/2017)

Sobre uma equação linear de grau 𝒏 é INCORRETO afirmar que

a) terá 𝒏 raízes complexas.

b) se 𝒏 for ímpar, sempre terá, ao menos, uma raiz real.

c) se um número complexo 𝒛 = 𝒂 + 𝒃𝒊, 𝒃 ≠ 𝟎 for raiz, então seu conjugado também o será.

d) a equação não pode ter raízes repetidas.

e) uma equação acima de grau 𝟒 pode ter todas as raízes reais.


Comentários
a) Verdadeira. O teorema fundamental da álgebra garante que isso.
b) Falso. Um contra exemplo é 𝑥 − 𝑖 = 0. 𝑛 = 1, ímpar, e a única raiz é imaginário puro.
𝜋 𝜋 𝜋
c) Falso. Um contra exemplo é 𝑥 2 − 𝑖 = 0 ⇒ 𝑥 2 = 𝑖 = 𝑒 𝑖 2 ⇒ 𝑥 = ± (cos + 𝑖 sen ).
4 4
Essas raízes não são conjugadas. Essa propriedade só é válida quando os coeficientes
são reais.
d) Obviamente uma equação pode ter raiz repetida, basta tomar por exemplo (𝑥 − 1)2 =
0, que é de grau 2 e tem duas raízes idênticas a 1.
e) Verdadeiro. Tome (𝑥 − 1)5 = 0. De quinto grau e todas as raízes reais.
Portanto, há 3 afirmações falsas. A questão foi anulada.
Gabarito: “Anulada”.
28. (EFOMM/2016)

Seja o polinômio 𝒑(𝒙) = 𝒙𝟔 − 𝟐𝟔𝒙𝟒 − 𝟑𝟐𝒙𝟑 − 𝟏𝟒𝟕𝒙𝟐 − 𝟗𝟔𝒙 − 𝟏𝟖𝟎.

A respeito das raízes da equação 𝒑(𝒙) = 𝟎, podemos afirmar que

a) todas as raízes são reais.


b) somente duas raízes são reais, sendo ela distintas.

AULA 07 – EQUAÇÕES ALGÉBRICAS 66


Prof. Victor So

c) somente duas raízes são reais, sendo elas iguais.

d) somente quatro raízes são reais, sendo todas elas distintas.

e) nenhuma raiz é real.


Comentários
Precisamos achar as raízes:
𝑝(𝑥) = 𝑥 6 − 26𝑥 4 − 32𝑥 3 − 147𝑥 2 − 96𝑥 − 180
Por inspeção, vemos que 𝑥 = −5 é raiz. Fatorando buscando o fator 𝑥 + 5:
𝑝(𝑥) = 𝑥 4 (𝑥 2 − 25) − 𝑥 4 − 5𝑥 3 − 27𝑥 3 − 135𝑥 2 − 12𝑥 2 − 60𝑥 − 36𝑥 − 180
⇒ 𝑝(𝑥) = 𝑥 4 (𝑥 − 5)(𝑥 + 5) − 𝑥 3 (𝑥 + 5) − 27𝑥 2 (𝑥 + 5) − 12𝑥 (𝑥 + 5) − 36(𝑥 + 5)
⇒ 𝑝(𝑥) = (𝑥 + 5)(𝑥 5 − 5𝑥 4 − 𝑥 3 − 27𝑥 2 − 12𝑥 − 36)
Por inspeção, veja que 6 é raiz do polinômio de grau 5 acima: Daí, fatorando buscando o
termo 𝑥 − 6:
𝑝(𝑥) = (𝑥 + 5)(𝑥 5 − 5𝑥 4 − 𝑥 3 − 27𝑥 2 − 12𝑥 − 36)
𝑝(𝑥) = (𝑥 + 5)(𝑥 5 − 6𝑥 4 + 𝑥 4 − 6𝑥 3 + 5𝑥 3 − 30𝑥 2 + 3𝑥 2 − 18𝑥 + 6𝑥 − 36)
𝑝(𝑥) = (𝑥 + 5)(𝑥 4 (𝑥 − 6) + 𝑥 3 (𝑥 − 6) + 5𝑥 2 (𝑥 − 6) + 3𝑥 (𝑥 − 6) + 6(𝑥 − 6))
𝑝(𝑥) = (𝑥 + 5)(𝑥 − 6)(𝑥 4 + 𝑥 3 + 5𝑥 2 + 3𝑥 + 6)
⇒ 𝑝(𝑥) = (𝑥 + 5)(𝑥 − 6)(𝑥 4 + 𝑥 3 + 5𝑥 2 + 3𝑥 + 6)
Analisando o polinômio de quarto grau acima, vamos mostrar que só possui raízes
complexas:
𝑥 4 + 𝑥 3 + 5𝑥 2 + 3𝑥 + 6 = (𝑥 4 + 5𝑥 2 + 6) + 𝑥 (𝑥 2 + 3)
Sabemos que as raízes de 𝑦 2 + 5𝑦 + 6 são −2 𝑜𝑢 − 3. Assim, se 𝑥 2 = −2 ou 𝑥 2 = −3, o
termo 𝑥 4 + 5𝑥 2 + 6 vai dar 0. Perceba que, apenas para 𝑥 2 = −3, tanto o termo 𝑥 4 + 5𝑥 2 + 6
quanto 𝑥(𝑥 2 + 3) vão zerar. Portanto, fatoraremos o polinômio do quarto grau buscando o termo
𝑥 2 + 3:
(𝑥 2 + 3)(𝑥 2 + 2) + 𝑥 (𝑥 2 + 3) = 0 ⇒ (𝑥 2 + 3)(𝑥 2 + 2 + 𝑥)
Claramente, as raízes dos fatores (𝑥 2 + 3) e (𝑥 2 + 𝑥 + 2) acima são complexas, pois o
delta é negativo. Portanto, 𝑝(𝑥) só possui duas raízes reais distintas.
Gabarito: “b”.
29. (EFOMM/2016)

A solução do sistema:
𝒙+𝒚+𝒛+𝒘=𝟕
𝒙𝒚 + 𝒙𝒛 + 𝒙𝒘 + 𝒚𝒛 + 𝒚𝒘 + 𝒛𝒘 = 𝟒
{
𝒘𝒚𝒛 + 𝒙𝒚𝒘 + 𝒙𝒛𝒘 + 𝒚𝒛𝒘 = 𝟔
𝒘𝒚𝒛𝒘 = 𝟏

AULA 07 – EQUAÇÕES ALGÉBRICAS 67


Prof. Victor So

pode ser representada pelas raízes do polinômio:

a) 𝒙𝟑 + 𝟔𝒙𝟐 + 𝟒𝒙 + 𝟕

b) 𝒙𝟑 + 𝟔𝒙𝟐 + 𝟒𝒙 − 𝟕

c) 𝟐𝒙𝟒 − 𝟏𝟒𝒙𝟑 + 𝟖𝒙𝟐 − 𝟏𝟐𝒙 + 𝟐

d) 𝟕𝒙𝟒 − 𝟒𝒙𝟑 + 𝟔𝒙𝟐 + 𝒙

e) 𝒙𝟒 + 𝟕𝒙𝟑 + 𝟒𝒙𝟐 + 𝟔𝒙
Comentários
Pelas relações de Girard, se 𝑥, 𝑦, 𝑧 𝑒 𝑤 são raízes da equação que buscamos, então ela
obedece às relações de Girard. A equação que buscamos é do quarto grau (4 raízes), da forma:
𝑎𝑥 4 + 𝑏𝑥 3 + 𝑐𝑥 2 + 𝑑𝑥 + 𝑒 = 0
É nos dado a soma das raízes, o produto das raízes duas a duas, três a três e o produto de
todas elas. Assim, por Girard, sabemos que:
𝑏
𝑥+𝑦+𝑧+𝑤 = 7=−
𝑎
Veja que a única alternativa que satisfaz isso é a letra 𝑐. Portanto, já dava para marcar até
aqui. Para motivos de verificação, vejamos as outras propriedades:
𝑐
𝑥𝑦 + 𝑥𝑧 + 𝑥𝑤 + 𝑦𝑧 + 𝑦𝑤 + 𝑤𝑧 = 4 =
𝑎
𝑑
𝑤𝑦𝑧 + 𝑥𝑦𝑤 + 𝑥𝑧𝑤 + 𝑦𝑧𝑤 = 6 = −
𝑎
𝑒
𝑤𝑦𝑧𝑤 = 1 =
𝑎
Veja que, para a letra 𝑐 todas essas condições são satisfeitas.
Gabarito: “c”.
30. (EFOMM/2015)

Assinale a alternativa que apresenta o polinômio 𝑷 de grau mínimo, com coeficiente reais, de modo
que 𝑷(𝒊) = 𝟐 e 𝑷(𝟏 + 𝒊) = 𝟎.

a) 𝟏/𝟓 (𝒙𝟐 − 𝟐𝒙 + 𝟐)

b) 𝟐/𝟓 (𝒙𝟐 − 𝟐𝒙 + 𝟐)

c) 𝟐/𝟓 (𝒙𝟐 − 𝟐𝒙 + 𝟑)

d) 𝟏/𝟓 (𝒙𝟐 − 𝟐𝒙𝟐 + 𝟐)


e) 𝟐/𝟑 (𝒙𝟐 − 𝟐𝒙 + 𝟑)

AULA 07 – EQUAÇÕES ALGÉBRICAS 68


Prof. Victor So

Comentários
Veja que 1 + 𝑖 é raiz desse polinômio. Entretanto, sabemos que, se o polinômio é de
coeficientes reais, e um número complexo 𝑧 for raiz dele, então o conjugado de 𝑧 também é raiz
desse mesmo polinômio. Portanto 1 − 𝑖 também é raiz de 𝑃(𝑥).
Pelo princípio fundamental da álgebra, escrevemos 𝑃, por não sabermos seu grau, como:
𝑃(𝑥) = 𝑞 (𝑥)(𝑥 − 1 − 𝑖)(𝑥 − 1 + 𝑖) = (𝑥 2 − 2𝑥 + 2)𝑞 (𝑥)
É dito que:
𝑃(𝑖) = 2 ⇒ 2 = (𝑖 2 − 2𝑖 + 2)𝑞 (𝑥) ⇒ (1 − 2𝑖)𝑞(𝑖) = 2
2 2 + 4𝑖 2
⇒ 𝑞 (𝑖 ) = = = (1 + 2𝑖)
1 − 2𝑖 1+4 5
Assim, para que 𝑃(𝑥) tenha o menor grau de coeficientes reais, devemos ter 𝑞 (𝑥) = 𝑎𝑥 +
𝑏, com 𝑎, 𝑏 ∈ ℝ:
2 4 4 2
𝑞(𝑥) = 𝑎𝑥 + 𝑏 ⇒ 𝑞(𝑖) = 𝑎𝑖 + 𝑏 = + 𝑖 ⇒ 𝑎 = ,𝑏 =
5 5 5 5
Portanto:
4𝑥 + 2
𝑃(𝑥) = 𝑞(𝑥)(𝑥 2 − 2𝑥 + 2) = ( ) (𝑥 2 − 2𝑥 + 2)
5
Veja que não temos alternativas do 3º grau e, portanto, a questão não possui alternativa
correta e foi anulada no ano de aplicação.
Gabarito: Anulada
31. (EFOMM/2013)

𝑷(𝒙) é um polinômio de coeficientes reais e menor grau com as propriedades abaixo:


• os números 𝒓𝟏 = 𝟏, 𝒓𝟐 = 𝒊 e 𝒓𝟑 = 𝟏 − 𝒊 são raízes da equação 𝑷(𝒙) = 𝟎;
• 𝑷(𝟎) = −𝟒.

Então, 𝑷(−𝟏) é igual a:

a) 𝟒.

b) −𝟐.

c) −𝟏𝟎.

d) 𝟏𝟎.

e) −𝟒𝟎.
Comentários
Sabemos que, se um polinômio é de coeficientes reais, e um número complexo 𝑧 for raiz
dele, então o conjugado de 𝑧 também é raiz desse mesmo polinômio. Assim:

AULA 07 – EQUAÇÕES ALGÉBRICAS 69


Prof. Victor So

𝑃(𝑖) = 0 ⇒ 𝑃(−𝑖) = 0
𝑃 (1 − 𝑖 ) = 0 ⇒ 𝑃 (1 + 𝑖 ) = 0
Desse modo, somado com a raiz 𝑟1 = 1, temos pelo menos 5 raízes em 𝑃(𝑥). Assim, como
queremos 𝑃(𝑥) de menor grau, basta tomarmos seu grau igual a 5:
𝑃(𝑥) = 𝑎(𝑥 − 1)(𝑥 + 𝑖)(𝑥 − 𝑖)(𝑥 − 1 + 𝑖)(𝑥 − 1 − 𝑖)
Ainda, queremos:
𝑃(0) = −4 ⇒ 𝑎(−1)(𝑖)(−𝑖)(−1 + 𝑖)(−1 − 𝑖) = −2𝑎 ⇒ −2𝑎 = 4 ⇒ 𝑎 = 2
Assim:
𝑃(−1) = 2(−1 − 1)(−1 + 𝑖)(−1 − 𝑖)(−2 + 𝑖)(−2 − 𝑖) = −4 ⋅ 2 ⋅ 5 = −40
Gabarito: “e”.
32. (EFOMM/2009)

Após a determinação dos valores numéricos: 𝒑(−𝟏), 𝒑(𝟎) e 𝒑(𝟏), verifica-se que o polinômio
𝒑(𝒙) = 𝒙𝟑 + 𝒙𝟐 − 𝒙 − 𝟎, 𝟓 tem

a) apenas uma raiz real.

b) apenas duas raízes reais.

c) três raízes reais, todas de mesmo sinal.

d) três raízes reais, duas positivas e uma negativa.

e) três raízes reais, duas negativas e uma positiva.


Comentários
Determinando os valores numéricos citados:
1
𝑝(−1) =
2
1
𝑝 (0 ) = −
2
1
𝑝 ( 1) =
2
Assim, vemos que a função polinomial muda de sinal entre o intervalo (−1,0), e em
seguida muda novamente no intervalo (0,1). Isso significa que ela tem duas raízes reais nesses
intervalos, uma em cada (uma positiva e outra negativa), pelo menos.
Entretanto, sabemos que ela possui grau 3, e o teorema fundamental da álgebra garante
que essa equação possui três raízes complexas.
Outro fato é que 𝑝(𝑥) = 𝑥 3 + 𝑥 2 − 𝑥 − 0,5 possui apenas coeficientes reais, então
qualquer raiz não real deveria ter sua conjugada como raiz também. Já sabemos que duas raízes
são reais. Portanto, não pode existir uma terceira raiz não real, pois se existisse, sua conjugada

AULA 07 – EQUAÇÕES ALGÉBRICAS 70


Prof. Victor So

também seria raiz, e o polinômio passaria a ter 4 raízes, o que é um absurdo. Assim, a terceira raiz
também tem que ser, necessariamente, real.
Para saber onde se localiza essa terceira raiz basta lembrarmos que:
lim 𝑥 3 + 𝑥 2 − 𝑥 − 0,5 = −∞
𝑥→−∞

Isto, é, a função tende para valores negativos quando 𝑥 fica muito negativo. Mas 𝑓 (−1) =
1
> 0! Isso implica que a função muda novamente de sinal para um 𝑥0 < −1, de modo que
2
𝑓 (𝑥0 ) = 0. Portanto essa terceira raiz deve ser menor que -1 e, portanto, é negativa.
Assim, 𝑝(𝑥) tem três raízes reais, duas negativas e uma positiva.
Gabarito: “e”.
33. (EFOMM/2005)

Determine as raízes na equação 𝒙𝟑 − 𝟗𝒙𝟐 + 𝟐𝟔𝒙 − 𝟐𝟒 = 𝟎, sabendo que elas estão em P.A.

a) 𝑺 = {𝟏, 𝟐, 𝟑}

b) 𝑺 = {𝟏, 𝟑, 𝟓}

c) 𝑺 = {𝟐, 𝟒, 𝟔}

d) 𝑺 = {𝟐, 𝟑, 𝟒}

e) 𝑺 = {𝟑, 𝟓, 𝟕}
Comentários
Sejam as raízes em PA 𝑎 − 𝑟, 𝑎 e 𝑎 + 𝑟. Por Girard, a soma das raízes é:
−9
(𝑎 − 𝑟 ) + 𝑎 + ( 𝑎 + 𝑟 ) = − =9
1
⇒ 3𝑎 = 9 ⇒ 𝑎 = 3
O produto das raízes, por Girard, é:
−24
(𝑎 − 𝑟)(𝑎 + 𝑟)𝑎 = − = 24
1
⇒ 3(3 + 𝑟)(3 − 𝑟) = 24 ⇒ 9 − 𝑟 2 = 8 ⇒ 𝑟 2 = 1 ⇒ 𝑟 = ±1
Portanto, se 𝑟 = 1, as raízes são: {2,3,4}. Se 𝑟 = −1 as raízes são {4, 3, 2}. Como são o
mesmo conjunto, então as raízes sempre vão ser {2,3.4}
Gabarito: “d”.
34. (EFOMM/2005)

Determine as raízes na equação 𝒙𝟑 − 𝟏𝟒𝒙𝟐 + 𝟓𝟔𝒙 − 𝟔𝟒 = 𝟎, sabendo que elas estão em P.G.

a) 𝑺 = {𝟏, 𝟐, 𝟒}
b) 𝑺 = {𝟐, 𝟑, 𝟒}

AULA 07 – EQUAÇÕES ALGÉBRICAS 71


Prof. Victor So

c) 𝑺 = {𝟐, 𝟑, 𝟔}

d) 𝑺 = {𝟐, 𝟒, 𝟔}

e) 𝑺 = {𝟐, 𝟒, 𝟖}
Comentários
As raízes estão em PG e são 𝑎, 𝑎𝑞, 𝑎𝑞2 . Pelas relações de Girard, o produto delas é:
−64
𝑎 ⋅ 𝑎𝑞 ⋅ 𝑎𝑞2 = − ⇒ 𝑎3 𝑞3 = 64 = 43 ⇒ 𝑎𝑞 = 4
1
Pela soma das raízes:
−14
𝑎 + 𝑎𝑞 + 𝑎𝑞2 = − = 14 ⇒ 𝑎 + 4 + 4𝑞 = 14 ⇒ 𝑎 + 4𝑞 = 10
1
Multiplicando a equação acima por 𝑞:
25 9
⇒ 𝑎𝑞 + 4𝑞2 = 10𝑞 ⇒ 4𝑞2 − 10𝑞 + 4 = 0 ⇒ 4𝑞2 − 10𝑞 + =
4 4
5 2 9 5 3 5 3
⇒ (2𝑞 − ) = ⇒ 2𝑞 − = ± ⇒ 2𝑞 = ± ⇒ 2𝑞 = 4 𝑜𝑢 2𝑞 = 1
2 4 2 2 2 2
1
⇒ 𝑞 = 2 𝑜𝑢 𝑞 =
2
1
Se 𝑞 = 2, as raízes são: {2, 4, 8}. Se 𝑞 = as raízes são {8, 4, 2}. Assim, formam o mesmo
2
conjunto de raízes. Portanto, para qualquer desses valores de 𝑞, as raízes são {2,4,8}.
Gabarito: “e”.
35. (Escola Naval/2014)

Considere 𝑷(𝒙) = (𝒎 − 𝟒)(𝒎𝟐 + 𝟒)𝒙𝟓 + 𝒙𝟐 + 𝒌𝒙 + 𝟏 um polinômio na variável real 𝒙, em que 𝒎


e 𝒌 são constantes reais. Quais os valores das constantes 𝒎 e 𝒌 para que 𝑷(𝒙) não admita raiz real?

a) 𝒎 = 𝟒 e −𝟐 < 𝒌 < 𝟐

b) 𝒎 = −𝟒 e 𝒌 > 𝟐

c) 𝒎 = −𝟐 e −𝟐 < 𝒌 < 𝟐

d) 𝒎 = 𝟒 e |𝒌| > 𝟐

e) 𝒎 = −𝟐 e 𝒌 > −𝟐
Comentários
Se 𝑚 e 𝑘 são constantes reais, então o polinômio dado possui coeficientes reais. Portanto,
o número de raízes complexas dessa equação só pode ser par, pois dado um complexo sendo raiz,
seu conjugado também deve ser.

AULA 07 – EQUAÇÕES ALGÉBRICAS 72


Prof. Victor So

Queremos que todas as raízes do polinômio sejam complexas não reais, então o grau do
polinômio precisa ser par (para que haja um número par de raízes complexas não reais). Desse
modo, o coeficiente que multiplica 𝑥 5 deve ser nulo:
(𝑚 − 4)(𝑚2 + 4) = 0 ⇒ 𝑚 = 4
Veja que não há solução real para 𝑚2 + 4 = 0. Dessa maneira, o polinômio até então é da
forma:
𝑃(𝑥) = 𝑥 2 + 𝑘𝑥 + 1
Para que não possua raízes reais, o delta da função tem que ser negativo:
Δ = 𝑘 2 − 4 < 0 ⇒ 𝑘 2 < 4 ⇒ −2 < 𝑘 < 2
Gabarito: “a”.
36. (Escola Naval/2013)

Sabendo que 𝒊√𝟑 é uma das raízes da equação 𝒙𝟒 + 𝒙𝟑 + 𝟐𝒙𝟐 + 𝟑𝒙 − 𝟑 = 𝟎, a soma de todas as
raízes desta equação é

a) −𝟐𝒊√𝟑

b) 𝟒𝒊√𝟑

c) 𝟎

d) −𝟏

e) −𝟐
Comentários
Pelas relações de Girard, a soma das raízes de uma equação do tipo:
𝑎𝑛 𝑥 𝑛 + 𝑎𝑛−1 𝑥 𝑛−1 + 𝑎𝑛−2 𝑥 𝑛−2 + ⋯ + 𝑎2 𝑥 2 + 𝑎1 𝑥 + 𝑎0
É igual a:
𝑎𝑛−1
𝑠𝑜𝑚𝑎 = −
𝑎𝑛
Assim, para a equação:
𝑥 4 + 𝑥 3 + 2𝑥 2 + 3𝑥 − 3 = 0
A soma de todas as suas raízes é:
1
𝑠𝑜𝑚𝑎 = − = −1
1
Gabarito: “d”.
37. (EN/2013)

AULA 07 – EQUAÇÕES ALGÉBRICAS 73


Prof. Victor So

Sejam 𝑭(𝒙) = 𝒙𝟑 + 𝒂𝒙 + 𝒃 e 𝑮(𝒙) = 𝟐𝒙𝟐 + 𝟐𝒙 − 𝟔 dois polinômios na variável real 𝒙, com 𝒂 𝒆 𝒃


𝑭(𝒙)
números reais. Qual valor de (𝒂 + 𝒃) para que a divisão 𝑮(𝒙) seja exata?

a) −𝟐

b) −𝟏

c) 0

d) 1

e) 2
Comentários
Para que a divisão seja exata, devemos ter que as raízes de 𝐺(𝑥) também são raízes de
𝐹(𝑥).
Note que as duas raízes de 𝐺 (𝑥) são reais, pois:
Δ = 4 − 4 ∙ (−6) ∙ 2 > 0
Além disso, temos que 𝐹(𝑥) possui grau 3, logo, se ele possui duas raízes reais, a terceira
deve ser real, pois as raízes complexas vêm aos pares. Disso, seja 𝛼 a raiz real de 𝐹(𝑥) que não é
raiz de 𝐺(𝑥).
2
Sabemos, por Girard em 𝐺(𝑥), que a soma de suas raízes é dada por − = −1. Aplicando
2
Girard para a soma das raízes em 𝐹(𝑥), temos:
𝛼−1=0⇒𝛼 =1
Como 1 é raiz de 𝐹(𝑥):
𝐹 (1) = 1 + 𝑎 + 𝑏 = 0 ⇒ 𝑎 + 𝑏 = −1
Gabarito: “b”.
38. (EN/2022)

Suponha que o conjunto solução da equação 𝟓𝒙𝟑 − 𝟒𝒙𝟐 + 𝟕𝒙 − 𝟐 = 𝟎 é {𝒙𝟏 , 𝒙𝟐 , 𝒙𝟑 }. Se a equação


polinomial 𝑷(𝒙) = 𝟎 apresenta {𝟓𝒙𝟏 , 𝟓𝒙𝟐 , 𝟓𝒙𝟑 } como conjunto solução, assinale a opção que
apresenta a soma dos coeficientes de 𝑷(𝒙).
𝟏𝟖
a) − 𝟐𝟓
𝟏𝟗
b) − 𝟏𝟓
𝟐𝟏
c) − 𝟐𝟎
𝟏𝟕
d) − 𝟑𝟓
𝟏𝟕
e) − 𝟐𝟎

AULA 07 – EQUAÇÕES ALGÉBRICAS 74


Prof. Victor So

Comentários
Note que a equação 𝑃(𝑥) = 0 possui as mesmas raízes da equação dada multiplicada por
5. Assim, podemos usar a equação dada para achar o polinômio, fazendo a seguinte
transformação:
𝑦
𝑦 = 5𝑥 ⇒ 𝑥 =
5
Assim, temos:
𝑦 3 𝑦 2 𝑦
𝑃 (𝑦 ) = 5 ( ) − 4 ( ) + 7 ( ) − 2
5 5 5
3 2
𝑦 4𝑦 7𝑦
𝑃 (𝑦 ) = − + −2
25 25 5
A soma dos coeficientes é dada por:
1 4 7 1 − 4 + 35 − 50 18
𝑃 (1 ) = − + −2= =−
25 25 5 25 25
Gabarito: A
39. (EN/2022)

Considere o sistema abaixo:


𝒚𝟐 + 𝒖𝟐 + 𝒗𝟐 + 𝒘𝟐 = 𝟒𝒙 − 𝟏
𝒙𝟐 + 𝒖𝟐 + 𝒗𝟐 + 𝒘𝟐 = 𝟒𝒚 − 𝟏
𝒙𝟐 + 𝒚𝟐 + 𝒗𝟐 + 𝒘𝟐 = 𝟒𝒖 − 𝟏
𝒙𝟐 + 𝒚𝟐 + 𝒖𝟐 + 𝒘𝟐 = 𝟒𝒗 − 𝟏
{𝒙𝟐 + 𝒚𝟐 + 𝒖𝟐 + 𝒗𝟐 = 𝟒𝒘 − 𝟏
Se 𝒙 = 𝒂, 𝒚 = 𝒃, 𝒖 = 𝒄, 𝒗 = 𝒅 e 𝒘 = 𝒆 constituem a solução do sistema, assinale a opção que
apresenta a soma 𝒂 + 𝒃 + 𝒄 + 𝒅 + 𝒆.
𝟓
a) 𝟐
𝟐
b) 𝟕
𝟏
c) 𝟕
𝟏
d) 𝟒
𝟐
e) 𝟑

Comentários
Substituindo a solução no sistema:

AULA 07 – EQUAÇÕES ALGÉBRICAS 75


Prof. Victor So

𝑏2 + 𝑐 2 + 𝑑 2 + 𝑒 2 = 4𝑎 − 1
𝑎2 + 𝑐 2 + 𝑑 2 + 𝑒 2 = 4𝑏 − 1
𝑎2 + 𝑏2 + 𝑑 2 + 𝑒 2 = 4𝑐 − 1
𝑎2 + 𝑏2 + 𝑐 2 + 𝑒 2 = 4𝑑 − 1
{𝑎2 + 𝑏2 + 𝑐 2 + 𝑑 2 = 4𝑒 − 1
Note que 𝑎, 𝑏, 𝑐, 𝑑, 𝑒 são positivos, pois a expressão da esquerda é a soma de números reais
elevados ao quadrado. Assim, temos 𝑎, 𝑏, 𝑐, 𝑑, 𝑒 > 0. Devido à simetria do problema, vamos
somar todas as equações:
4(𝑎 2 + 𝑏 2 + 𝑐 2 + 𝑑 2 + 𝑒 2 ) = 4(𝑎 + 𝑏 + 𝑐 + 𝑑 + 𝑒 ) − 5
5
⇒ 𝑎 2 + 𝑏 2 + 𝑐 2 + 𝑑 2 + 𝑒 2 = (𝑎 + 𝑏 + 𝑐 + 𝑑 + 𝑒 ) −
(𝐼 )
4
Usando a primeira equação em (𝐼) e fazendo 𝑎 + 𝑏 + 𝑐 + 𝑑 + 𝑒 = 𝑆
5
𝑎2 + 4𝑎 − 1 = 𝑆 −
4
Completando o quadrado para 𝑎:
5
𝑎2 + 4𝑎 − 1 + 5 = 𝑆 − +5
4
15
𝑎2 + 4𝑎 + 4 = 𝑆 +
4
15
(𝑎 + 2) 2 = 𝑆 +
4
Como 𝑎 > 0, devemos ter:

15
⇒ 𝑎 + 2 = √𝑆 +
4
Analogamente, obtemos:

15
𝑏 + 2 = √𝑆 +
4

15
𝑐 + 2 = √𝑆 +
4

15
𝑑 + 2 = √𝑆 +
4

15
𝑒 + 2 = √𝑆 +
4

AULA 07 – EQUAÇÕES ALGÉBRICAS 76


Prof. Victor So

Somando todas elas:

15
𝑎 + 𝑏 + 𝑐 + 𝑑 + 𝑒 + 10 = 5√𝑆 +
4

15
𝑆 + 10 = 5√𝑆 +
4
Elevando ao quadrado e resolvendo:
15
𝑆 2 + 20𝑆 + 100 = 25 (𝑆 + )
4
25
𝑆 2 − 5𝑆 + =0
4
4𝑆 2 − 20𝑆 + 25 = 0
20 ± √400 − 400 20 5
𝑆= = =
8 8 2
Gabarito: A

9. QUESTÕES NÍVEL 3
40. (ITA/2019)

Seja 𝒑(𝒙) = 𝒙𝟑 + 𝒂𝒙𝟐 + 𝒃𝒙 um polinômio cujas raízes são não negativas e estão em progressão
aritmética. Sabendo que a soma de seus coeficientes é igual a 10, podemos afirmar que a soma das
raízes de 𝒑(𝒙) é igual a

a) 𝟗

b) 𝟖
c) 𝟑
𝟗
d) 𝟐

e) 𝟏𝟎

41. (ITA/2019)

Considere as seguintes afirmações:

I. Se 𝒙𝟏 , 𝒙𝟐 𝐞 𝒙𝟑 são as raízes da equação 𝒙𝟑 − 𝟐𝒙𝟐 + 𝒙 + 𝟐 = 𝟎, então 𝒚𝟏 = 𝒙𝟐 𝒙𝟑 , 𝒚𝟐 =


𝒙𝟏 𝒙𝟑 𝐞 𝒚𝟑 = 𝒙𝟏 𝒙𝟐 são as raízes da equação 𝒚𝟑 − 𝒚𝟐 − 𝟒𝒚 − 𝟒 = 𝟎.

AULA 07 – EQUAÇÕES ALGÉBRICAS 77


Prof. Victor So

II. A soma dos cubos de três números inteiros consecutivos é divisível por 9.
𝟑+√𝟓 𝟏+√𝟓
III. √ =
𝟐 𝟐

É(são) VERDADEIRA(S)

a) Apenas I.

b) Apenas II.

c) Apenas III.

d) Apenas II e III.

e) Todas.

42. (ITA/2018)
1 𝑥 𝑥2 𝑥3
Considere a matriz [ 1 2 3 4 ], 𝑥 ∈ ℝ. Se o polinômio 𝑝(𝑥) é dado por 𝑝(𝑥) = 𝑑𝑒𝑡𝐴,
−1 3 4 5
−2 2 1 1
então o produto das raízes de 𝑝(𝑥) é
1
a)
2
1
b)
3
1
c)
5
1
d)
7
1
e)
11

43. (ITA/2016)
Seja 𝑝 o polinômio dado por 𝑝(𝑥) = 𝑥 8 + 𝑥 𝑚 − 2𝑥 𝑛 , em que os expoentes 8, 𝑚, 𝑛 formam,
nesta ordem, uma progressão geométrica cuja soma dos termos é igual a 14. Considere as
seguintes afirmações:
I. 𝑥 = 0 é uma raiz dupla de 𝑝.
II. 𝑥 = 1 é uma raiz dupla de 𝑝.
III. 𝑝 tem quatro raízes com parte imaginária não nula.
Destas, é (são) verdadeira(s)
a) Apenas I.

AULA 07 – EQUAÇÕES ALGÉBRICAS 78


Prof. Victor So

b) Apenas I e II.
c) Apenas I e III.
d) Apenas II e III.
e) I, II e III.

44. (ITA/2016)
Considere o polinômio 𝑝 com coeficientes complexos definido por
𝑝 (𝑧 ) = 𝑧 4 + (2 + 𝑖 ) 𝑧 3 + ( 2 + 𝑖 ) 𝑧 2 + (2 + 𝑖 ) 𝑧 + (1 + 𝑖 ) .
Podemos afirmar que
a) Nenhuma das raízes de 𝑝 é real.
b) Não existem raízes de 𝑝 que sejam complexas conjugadas.
c) A soma dos módulos de todas as raízes de 𝑝 é igual a 2 + √2.
d) O produto dos módulos de todas as raízes de 𝑝 é igual a 2√2.
e) O módulo de uma das raízes de 𝑝 é igual a √2.

45. (ITA/2014)
Considere os polinômios em 𝑥 ∈ ℝ da forma 𝑝(𝑥) = 𝑥 5 + 𝑎3 𝑥 3 + 𝑎2 𝑥 2 + 𝑎1 𝑥. As raízes de
1
𝑝(𝑥) = 0 constituem uma progressão aritmética de razão quando (𝑎1 , 𝑎2 , 𝑎3 ) é igual a
2
1 5
a) ( , 0, ).
4 4
1 5
b) ( , 1, ).
4 4
1 5
c) ( , 0, − ).
4 4
5 1
d) ( , 0, ).
4 4
1 1
e) ( , −1, − ).
4 4

46. (ITA/2012)
As raízes 𝑥1 , 𝑥2 𝑒 𝑥3 do polinômio 𝑝(𝑥) = 16 + 𝑎𝑥 − (4 + √2)𝑥 2 + 𝑥 3 estão relacionadas
pelas equações:
𝑥3
𝑥1 + 2𝑥2 + = 2 𝑒 𝑥1 − 2𝑥2 − √2𝑥3 = 0
2
Então, o coeficiente 𝑎 é igual a

AULA 07 – EQUAÇÕES ALGÉBRICAS 79


Prof. Victor So

a) 2(1 − √2).
b) √2 − 4.
c) 2(2 + √2).
d) 4 + √2.
e) 4(√2 − 1).

47. (ITA/2010)
Sabe-se que o polinômio 𝑝(𝑥) = 𝑥 5 − 𝑎𝑥 3 + 𝑎𝑥 2 − 1, 𝑎 ∈ ℝ, admite a raiz −𝑖. Considere
as seguintes afirmações sobre as raízes de 𝑝:
I. Quatro das raízes são imaginárias puras.
II. Uma das raízes tem multiplicidade dois.
III. Apenas uma das raízes é real.
Destas, é (são) verdadeira(s) apenas
a) I.
b) II.
c) III.
d) I e III.
e) II e III.

48. (ITA/2010)
Um polinômio real 𝑝(𝑥) = ∑5𝑛=0 𝑎𝑛 𝑥 𝑛 , com 𝑎5 = 4, tem três raízes reais distintas, 𝑎, 𝑏 𝑒 𝑐,
que satisfazem o sistema
𝑎 + 2𝑏 + 5𝑐 = 0
{ 𝑎 + 4𝑏 + 2𝑐 = 6
2𝑎 + 2𝑏 + 2𝑐 = 5
Sabendo que a maior das raízes é simples e as demais tem multiplicidade dois, pode-se
afirmar que 𝑝(1) é igual a
a) −4.
b) −2.
c) 2.
d) 4.
e) 6.

AULA 07 – EQUAÇÕES ALGÉBRICAS 80


Prof. Victor So

49. (ITA/2010)
Considere o polinômio 𝑝(𝑥) = ∑6𝑛=0 𝑎𝑛 𝑥 𝑛 , com coeficientes reais, sendo 𝑎0 ≠ 0 e 𝑎6 = 1.
Sabe-se que se 𝑟 é raiz de 𝑝, −𝑟 também é raiz de 𝑝. Analise a veracidade ou falsidade das
afirmações:
I. Se 𝑟1 𝑒 𝑟2 , |𝑟1 | ≠ |𝑟2 |, são raízes reais e 𝑟3 é raiz não real de 𝑝, então 𝑟3 é imaginário
puro.
II. Se 𝑟 é raiz dupla de 𝑝, então 𝑟 é real ou imaginário puro.
III. 𝑎0 < 0.

50. (ITA/2009)
Considere as funções 𝑓(𝑥) = 𝑥 4 + 2𝑥 3 − 2𝑥 − 1 e 𝑔(𝑥) = 𝑥 2 − 2𝑥 + 1. A multiplicidade
das raízes não reais da função composta 𝑓 ∘ 𝑔 é igual a
a) 1.
b) 2.
c) 3.
d) 4.
e) 5.

51. (ITA/2009)
O polinômio de grau 4
(𝑎 + 2𝑏 + 𝑐 )𝑥 4 + (𝑎 + 𝑏 + 𝑐 )𝑥 3 − (𝑎 − 𝑏)𝑥 2 + (2𝑎 − 𝑏 + 𝑐 )𝑥 + 2(𝑎 + 𝑐 ),
Com 𝑎, 𝑏, 𝑐 ∈ ℝ, é uma função par. Então, a soma dos módulos de suas raízes é igual a
a) 3 + √3.
b) 2 + 3√3.
c) 2 + √2.
d) 1 + 2√2.
e) 2 + 2√2.

52. (ITA/2006)
Sobre o polinômio 𝑝(𝑥) = 𝑥 5 − 5𝑥 3 + 4𝑥 2 − 3𝑥 − 2 podemos afirmar que
a) 𝑥 = 2 não é raiz de 𝑝.

AULA 07 – EQUAÇÕES ALGÉBRICAS 81


Prof. Victor So

b) 𝑝 só admite raízes reais, sendo uma delas inteira, duas racionais e duas irracionais.
c) 𝑝 admite uma única raiz real, sendo ela uma raiz inteira.
d) 𝑝 só admite raízes reais, sendo duas delas inteiras.
e) 𝑝 admite somente 3 raízes reais, sendo uma delas inteira e duas irracionais.

53. (ITA/2006)
Seja 𝑝 um polinômio com coeficientes reais, de grau 7, que admite 1 − 𝑖 como raiz de
multiplicidade 2. Sabe-se que a soma e o produto de todas as raízes de 𝑝 são,
respectivamente, 10 e −40. Sendo afirmado que três raízes de 𝑝 são reais e distintas e
formam uma progressão aritmética, então, tais raízes são
3 193 3 193
a) √ ,3, + √ .
2 6 2 6

b) 2 − 4√13, 2, 2 + 4√13.
c) −4, 2, 8.
d) −2, 3, 8.
e) −1, 2, 5.

54. (ITA/2005)
O número complexo 2 + 𝑖 é raiz do polinômio 𝑓 (𝑥) = 𝑥 4 + 𝑥 3 + 𝑝𝑥 2 + 𝑥 + 𝑞, com 𝑝, 𝑞 ∈
ℝ. Então, a alternativa que mais se aproxima da soma das raízes reais de 𝑓 é
a) 4.
b) −4.
c) 6.
d) 5.
e) −5.

55. (ITA/2004)
Para algum número real 𝑟, o polinômio 8𝑥 3 − 4𝑥 2 − 42𝑥 + 45 é divisível por (𝑥 − 𝑟)2 . Qual
dos números abaixo está mais próximo de 𝑟?
a) 1,62.
b) 1,52.
c) 1,42.
d) 1,32.

AULA 07 – EQUAÇÕES ALGÉBRICAS 82


Prof. Victor So

e) 1,22.

56. (ITA/2001)
O valor da soma 𝑎 + 𝑏 para que as raízes do polinômio 4𝑥 4 − 20𝑥 3 + 𝑎𝑥 2 − 25𝑥 + 𝑏
estejam em progressão aritmética de razão 1/2 é:
a) 36.
b) 41.
c) 26.
d) -27.
e) -20.

57. (ITA/2001)
O polinômio com coeficientes reais 𝑃(𝑥) = 𝑥 5 + 𝑎4 𝑥 4 + 𝑎3 𝑥 3 + 𝑎2 𝑥 2 + 𝑎1 𝑥 + 𝑎0 tem duas
raízes distintas, cada uma delas com multiplicidade 2, e duas de suas raízes são 2 e 𝑖. Então
a soma dos coeficientes é igual a:
a) −4.
b) −6.
c) −1.
d) 1.
e) 4.

58. (ITA/1998)
Seja 𝑎 um número real tal que o polinômio 𝑝(𝑥) = 𝑥 6 + 2𝑥 5 + 𝑎𝑥 4 − 𝑎𝑥 2 − 2𝑥 − 1 admite
apenas raízes reais. Então:
a) 𝑎 ∈ [2, ∞[.
b) 𝑎 ∈ [−1,1].
c) 𝑎 ∈ ] − ∞, −7].
d) 𝑎 ∈ [−2, −1[.
e) 𝑎 ∈ ]1,2[.

59. (ITA/1996)
Considere o polinômio 𝑝(𝑧) = 𝑧 6 + 2𝑧 5 + 6𝑧 4 + 12𝑧 3 + 8𝑧 2 + 16𝑧.

AULA 07 – EQUAÇÕES ALGÉBRICAS 83


Prof. Victor So

Sobre as raízes da equação 𝑝(𝑧) = 0, podemos afirmar que:


a) Apenas uma é real.
b) Apenas duas raízes são reais e distintas.
c) Apenas duas raízes são reais e iguais.
d) Quatro raízes são reais, sendo duas a duas distintas.
e) Quatro raízes são reais, sendo apenas duas iguais.

60. (ITA/1994)
Seja 𝑃(𝑥) um polinômio de grau 5, com coeficientes reais, admitindo 2 e 𝑖 como raízes. Se
𝑃(1)𝑃(−1) < 0, então o número de raízes reais de 𝑃(𝑥) pertencentes ao intervalo ] − 1,1[
é:
a) 0.
b) 1.
c) 2.
d) 3.
e) 4.

61. (IME/2019)

Seja a inequação:

𝟔𝒙𝟒 − 𝟓𝒙𝟑 − 𝟐𝟗𝒙𝟐 + 𝟏𝟎𝒙 < 𝟎


Seja (𝒂, 𝒃) um intervalo contido no conjunto solução dessa inequação. O maior valor possível para
𝒃 − 𝒂 é:

a) 𝟐

b) 𝟏𝟑/𝟔

c) 𝟏/𝟑

d) 𝟓/𝟐

e) 𝟖/𝟑

62. (IME/2019)

Sejam 𝒙𝟏 , 𝒙𝟐 e 𝒙𝟑 raízes da equação 𝒙𝟑 − 𝒂𝒙 − 𝟏𝟔 = 𝟎. Sendo 𝒂 um número real, o valor de 𝒙𝟑𝟏 +


𝒙𝟑𝟐 + 𝒙𝟑𝟑 é igual a:

AULA 07 – EQUAÇÕES ALGÉBRICAS 84


Prof. Victor So

a) 𝟑𝟐 − 𝒂

b) 𝟒𝟖 − 𝟐𝒂

c) 𝟒𝟖

d) 𝟒𝟖 + 𝟐𝒂

e) 𝟑𝟐 + 𝒂

GABARITO
40. a
41. e
42. d
43. 4
44. c
45. e
46. c
47. e
48. c
49. a
50. c
51. e
52. e
53. e
54. e
55. b
56. b
57. a
58. c
59. b
60. b
61. b
62. c

RESOLUÇÃO
40. (ITA/2019)

AULA 07 – EQUAÇÕES ALGÉBRICAS 85


Prof. Victor So

Seja 𝒑(𝒙) = 𝒙𝟑 + 𝒂𝒙𝟐 + 𝒃𝒙 um polinômio cujas raízes são não negativas e estão em progressão
aritmética. Sabendo que a soma de seus coeficientes é igual a 10, podemos afirmar que a soma das
raízes de 𝒑(𝒙) é igual a

a) 𝟗

b) 𝟖

c) 𝟑
𝟗
d) 𝟐

e) 𝟏𝟎
Comentários
Analisando o polinômio, podemos descobrir uma das raízes. Vamos colocar 𝑥 em
evidência:
𝑝(𝑥) = 𝑥 3 + 𝑎𝑥 2 + 𝑏𝑥 = 𝑥 (𝑥 2 + 𝑎𝑥 + 𝑏) = 0
Logo, 𝑥 = 0 é raiz.
Como as raízes são não negativas e elas estão em progressão aritmética, a PA deve ser do
tipo:
(0, 𝑟, 2𝑟), com 𝑟 ≥ 0
Então, temos que encontrar o valor de 𝑟.
Ainda, do enunciado, temos que a soma dos coeficientes é igual a 10:
1 + 𝑎 + 𝑏 = 10 ⇒ 𝑎 + 𝑏 = 9 (𝐼)
Vamos usar as relações de Girard na seguinte equação:
𝑥 2 + 𝑎𝑥 + 𝑏 = 0 (𝐼𝐼)
Desse modo:
𝑥1 + 𝑥2 = 𝑟 + 2𝑟 = −𝑎 ⇒ 𝑎 = −3𝑟
De (𝐼), podemos encontrar o valor de 𝑏:
𝑎 + 𝑏 = 9 ⇒ 𝑏 = 9 + 3𝑟
Substituindo os valores encontrados em (𝐼𝐼) e fazendo 𝑥 = 𝑟:
𝑟 2 + (−3𝑟)𝑟 + 9 + 3𝑟 = 0
3
−2𝑟 2 + 3𝑟 + 9 = 0 ⇒ 𝑟1 = 3 𝑜𝑢 𝑟2 = −
2
Sendo as raízes não negativas, devemos ter 𝑟 = 3.
Portanto, a soma das raízes é dada por:
0 + 𝑟 + 2𝑟 = 3𝑟 = 9

AULA 07 – EQUAÇÕES ALGÉBRICAS 86


Prof. Victor So

Gabarito: “a”.
41. (ITA/2019)

Considere as seguintes afirmações:

I. Se 𝒙𝟏 , 𝒙𝟐 𝐞 𝒙𝟑 são as raízes da equação 𝒙𝟑 − 𝟐𝒙𝟐 + 𝒙 + 𝟐 = 𝟎, então 𝒚𝟏 = 𝒙𝟐 𝒙𝟑 , 𝒚𝟐 =


𝒙𝟏 𝒙𝟑 𝐞 𝒚𝟑 = 𝒙𝟏 𝒙𝟐 são as raízes da equação 𝒚𝟑 − 𝒚𝟐 − 𝟒𝒚 − 𝟒 = 𝟎.

II. A soma dos cubos de três números inteiros consecutivos é divisível por 9.
𝟑+√𝟓 𝟏+√𝟓
III. √ =
𝟐 𝟐

É(são) VERDADEIRA(S)

a) Apenas I.

b) Apenas II.

c) Apenas III.

d) Apenas II e III.

e) Todas.
Comentários
I. Verdadeira.
Utilizando as relações de Girard nas equações, temos:
𝑥1 + 𝑥2 + 𝑥3 = 2
3 2
𝑥 − 2𝑥 + 𝑥 + 2 = 0 ⇒ { 1 2 + 𝑥2 𝑥3 + 𝑥1 𝑥3 = 1
𝑥 𝑥
𝑥1 𝑥2 𝑥3 = −2
𝑦1 + 𝑦2 + 𝑦3 = 1
3 2
𝑦 − 𝑦 − 4𝑦 − 4 = 0 ⇒ {𝑦1 𝑦2 + 𝑦2 𝑦3 + 𝑦1 𝑦3 = −4
𝑦1 𝑦2 𝑦3 = 4
Vamos verificar se as relações em 𝑦 são válidas, usando as relações em 𝑥. Para 𝑦1 =
𝑥2 𝑥3 , 𝑦2 = 𝑥1 𝑥3 e 𝑦3 = 𝑥1 𝑥2 :
𝑦1 + 𝑦2 + 𝑦3 = 𝑥2 𝑥3 + 𝑥1 𝑥3 + 𝑥1 𝑥2 = 1
𝑦1 𝑦2 + 𝑦2 𝑦3 + 𝑦1 𝑦3 = 𝑥2 𝑥3 𝑥1 𝑥3 + 𝑥1 𝑥3 𝑥1 𝑥2 + 𝑥2 𝑥3 𝑥1 𝑥2
= 𝑥1 𝑥2 𝑥3 (𝑥1 + 𝑥2 + 𝑥3 ) = (−2) ⋅ 2 = −4
𝑦1 𝑦2 𝑦3 = (𝑥2 𝑥3 )(𝑥1 𝑥3 )(𝑥1 𝑥2 ) = (𝑥1 𝑥2 𝑥3 )2 = (−2)2 = 4
Portanto, todas as relações são válidas, logo, elas são raízes da equação.

II. Verdadeira.

AULA 07 – EQUAÇÕES ALGÉBRICAS 87


Prof. Victor So

Seja 𝑎 ∈ ℤ. Vamos tomar os inteiros consecutivos (𝑎 − 1, 𝑎, 𝑎 + 1), a soma dos seus cubos
resulta:
(𝑎 − 1)3 + 𝑎 3 + (𝑎 + 1)3
= 𝑎3 − 3𝑎2 + 3𝑎 − 1 + 𝑎3 + 𝑎3 + 3𝑎2 + 3𝑎 + 1
= 3𝑎3 + 6𝑎
= 3𝑎(𝑎2 + 2)
Devemos provar que, para qualquer 𝑎 ∈ ℤ, a expressão 3𝑎(𝑎2 + 2) é divisível por 9.
Vamos dividir o problema em todos os casos possíveis:
a) 𝑎 = 3𝑘, 𝑘 ∈ ℤ valores (0, ±3, ±6, ±9, … , ±3𝑘)
3𝑎(𝑎2 + 2) = 3(3𝑘 )((3𝑘 )2 + 2) = 9𝑘(9𝑘 2 + 2)
b) 𝑎 = 3𝑘 + 1 valores (1, 4, 7, 10, … ,3𝑘 + 1) e (−2, −5, −8, … , −3𝑘 + 1)
3𝑎(𝑎2 + 2) = 3(3𝑘 + 1)((3𝑘 + 1)2 + 2) = 3(3𝑘 + 1)(9𝑘 2 + 6𝑘 + 1 + 2)
= 9(3𝑘 + 1)(3𝑘 2 + 2𝑘 + 1)
c) 𝑎 = 3𝑘 + 2 valores (2, 5, 8, … ,3𝑘 + 1) e (−1, −4, −7, … , −3𝑘 + 1)
3𝑎(𝑎2 + 2) = 3(3𝑘 + 2)((3𝑘 + 2)2 + 2) = 3(3𝑘 + 2)(9𝑘 2 + 12𝑘 + 4 + 2)
= 9(3𝑘 + 2)(3𝑘 2 + 4𝑘 + 2)
Todos esses casos possuem o fator 9 na expressão, logo, para 𝑎 ∈ ℤ, 3𝑎(𝑎2 + 2) é divisível
por 9.

III. Verdadeira.
2
1 + √5 1 + √5 1 + 5 + 2√5 √6 + 2√5 √3 + √5
= √( ) =√ = =
2 2 4 4 2
Gabarito: “e”.
42. (ITA/2018)
1 𝑥 𝑥2 𝑥3
Considere a matriz [ 1 2 3 4 ], 𝑥 ∈ ℝ. Se o polinômio 𝑝(𝑥) é dado por 𝑝(𝑥) = 𝑑𝑒𝑡𝐴,
−1 3 4 5
−2 2 1 1
então o produto das raízes de 𝑝(𝑥) é
1
a)
2
1
b)
3
1
c)
5

AULA 07 – EQUAÇÕES ALGÉBRICAS 88


Prof. Victor So

1
d)
7
1
e)
11

Comentários
O polinômio 𝑝(𝑥) é dado por:
1 𝑥 𝑥2 𝑥3
𝑝 (𝑥 ) = | 1 2 3 4|
−1 3 4 5
−2 2 1 1
Vamos escrever 𝑝(𝑥) da seguinte forma:
𝑝(𝑥) = 𝑎3 𝑥 3 + 𝑎2 𝑥 2 + 𝑎1 𝑥 + 𝑎0
Aplicando Laplace à primeira linha do determinante, temos:
2 3 4
𝑎0 = (−1)1+1 |3 4 5| = −1
2 1 1
1 3 4
𝑎1 = (−1)1+2 |−1 4 5| = −1 ∙ 0 = 0
−2 1 1
1 2 4
1+3
𝑎2 = (−1) |−1 3 5| = −9
−2 2 1
1 2 3
1+4
𝑎3 = (−1) |−1 3 4| = −1 ∙ (−7) = 6
−2 2 1
Ou seja, 𝑝(𝑥) é dado por:
𝑝(𝑥) = 7𝑥 3 − 9𝑥 2 − 1
Das relações de Girard, sabemos que o produto das raízes de um polinômio do 3º grau é
dado por:
𝑎0 1 1
− = − (− ) =
𝑎3 7 7
Gabarito: “d”.
43. (ITA/2018)
Seja 𝑝(𝑥) um polinômio não nulo. Se 𝑥 3 − 4𝑥 2 + 5𝑥 − 2 e 𝑥 3 − 5𝑥 2 + 8𝑥 − 4 são divisores
de 𝑝(𝑥), determine o menor grau possível de 𝑝(𝑥).
Comentário
Se os polinômios dados dividem 𝑝(𝑥), podemos escrever:
𝑝(𝑥) = 𝑞1 (𝑥)(𝑥 3 − 4𝑥 2 + 5𝑥 − 2)

AULA 07 – EQUAÇÕES ALGÉBRICAS 89


Prof. Victor So

𝑝(𝑥) = 𝑞2 (𝑥)(𝑥 3 − 5𝑥 2 + 8𝑥 − 4 )
Ou seja, as raízes dos polinômios dados são raízes de 𝑝(𝑥). Então, vamos descobrir as raízes
dos polinômios dados. Para facilitar, vamos dar nomes aos polinômios:
𝑔1 (𝑥) = 𝑥 3 − 4𝑥 2 + 5𝑥 − 2
𝑔2 (𝑥) = 𝑥 3 − 5𝑥 2 + 8𝑥 − 4
Em polinômios de grau maior que 2 é sempre útil chutar que 1 é uma de suas raízes. Então,
vamos verificar:
𝑔1 (1) = 1 − 4 + 5 − 2 = 0
𝑔2 (1) = 1 − 5 + 8 − 4 = 0
Ou seja, 1 é raiz de 𝑔1 (𝑥) e de 𝑔2 (𝑥). Vamos usar o dispositivo de Briot-Ruffini para reduzir
o grau desses polinômios.
Para 𝑔1 (𝑥), temos:

1 1 −4 5 −2

1 −3 2 0

Ou seja:
𝑔1 (𝑥) = (𝑥 − 1)(𝑥 2 − 3𝑥 + 2)
Logo, as outras raízes de 𝑔1 (𝑥) são as raízes de (𝑥 2 − 3𝑥 + 2), que são 1 e 2.
De outra forma:
𝑔1 (𝑥) = (𝑥 − 1)2 (𝑥 − 2)
Para 𝑔2 (𝑥), temos:

1 1 −5 8 −4

1 −4 4 0

Ou seja:
𝑔2 (𝑥) = (𝑥 − 1)(𝑥 2 − 4𝑥 + 4)
Logo, as outras raízes de 𝑔2 (𝑥) são as raízes de (𝑥 2 − 4𝑥 + 4) = (𝑥 − 2)2 , ou seja, 2 com
multiplicidade 2.
De outra forma:
𝑔2 (𝑥) = (𝑥 − 1)(𝑥 − 2)2
Conclusão:

AULA 07 – EQUAÇÕES ALGÉBRICAS 90


Prof. Victor So

O polinômio 𝑝(𝑥) deve possuir 1 como raiz como multiplicidade 2, de 𝑔1 (𝑥), e deve possuir
2 com multiplicidade 2, de 𝑔2 (𝑥). Ou seja, para atender ao enunciado, o polinômio 𝑝(𝑥) deve ser
da forma:
𝑝(𝑥) = 𝑔(𝑥)(𝑥 − 1)2 (𝑥 − 2)2
Logo, o menor grau possível para 𝑝(𝑥) é 4, que é quando 𝑔(𝑥) = 𝑐, 𝑐 ∈ ℝ.
Gabarito: 4.
44. (ITA/2016)
Seja 𝑝 o polinômio dado por 𝑝(𝑥) = 𝑥 8 + 𝑥 𝑚 − 2𝑥 𝑛 , em que os expoentes 8, 𝑚, 𝑛 formam,
nesta ordem, uma progressão geométrica cuja soma dos termos é igual a 14. Considere as
seguintes afirmações:
I. 𝑥 = 0 é uma raiz dupla de 𝑝.
II. 𝑥 = 1 é uma raiz dupla de 𝑝.
III. 𝑝 tem quatro raízes com parte imaginária não nula.
Destas, é (são) verdadeira(s)
a) Apenas I.
b) Apenas I e II.
c) Apenas I e III.
d) Apenas II e III.
e) I, II e III.
Comentários
O primeiro passo nessa questão é encontrar os valores de 𝑚 e 𝑛 usando o fato de que eles
pertencem a uma P.G. de primeiro termo 8. Seja 𝑞 a razão dessa progressão, podemos
representar essa progressão pela terna ordenada:
(8, 8𝑞, 8𝑞2 )
Sua soma é 14, do que temos:
8 + 8𝑞 + 8𝑞2 = 14 ⇒ 4𝑞2 + 4𝑞 − 3 = 0
Resolvendo para 𝑞, temos:
3 1
𝑞=−𝑜𝑢 𝑞 =
2 2
Como se trata de um polinômio, o expoente de 𝑥 deve ser, necessariamente, um número
inteiro não negativo.
3
Suponha que 𝑞 = − . Disso, teríamos que:
2

AULA 07 – EQUAÇÕES ALGÉBRICAS 91


Prof. Victor So

3
𝑚 = 8 ∙ (− ) = −12
2
Ou seja, 𝑚 seria negativo, o que não convém.
1
A conclusão é que 𝑞 = e a terna ordenada fica:
2

(8, 4, 2)
Do que segue que:
𝑝(𝑥) = 𝑥 8 + 𝑥 4 − 2𝑥 2
De posse do polinômio, podemos julgar as afirmações.
I. Verdadeira: Veja que 𝑝(𝑥) = 𝑥 8 + 𝑥 4 − 2𝑥 2 = 𝑥 2 (𝑥 6 + 𝑥 2 − 2), ou seja, 𝑝(𝑥)
apresenta o fator (𝑥 − 0) duas vezes, de onde temos que 0 é uma raiz dupla.
II. Falsa: Observe que
𝑥 6 + 𝑥 2 − 2 = 𝑥 6 − 1 + 𝑥 2 − 1 = (𝑥 2 − 1)(𝑥 4 + 𝑥 2 + 1) + (𝑥 2 − 1)
Ou ainda
𝑝(𝑥) = (𝑥 2 − 1)(𝑥 4 + 𝑥 2 + 2)
Disso, temos que 1 é raiz com multiplicidade 1, pois 𝑝(𝑥) somente apresenta uma
vez o fator (𝑥 − 1).
III. Verdadeira: Vamos mostrar que as raízes de (𝑥 4 + 𝑥 2 + 2) são complexas não
1 1 7 1 2 7
reais. Para isso, veja que 𝑥 4 + 𝑥 2 + 2 = (𝑥 2 )2 + 2 ∙ 𝑥 2 + + = (𝑥 2 + ) + .
2 4 4 2 4
2 1 2 7 7
Mas isso obedece (𝑥 + ) + ≥ para todo 𝑥 real. Logo, essa equação não
2 4 4
apresenta raízes reais.
Gabarito: “c”.
45. (ITA/2016)
Considere o polinômio 𝑝 com coeficientes complexos definido por
𝑝 (𝑧 ) = 𝑧 4 + (2 + 𝑖 ) 𝑧 3 + ( 2 + 𝑖 ) 𝑧 2 + (2 + 𝑖 ) 𝑧 + (1 + 𝑖 ) .
Podemos afirmar que
a) Nenhuma das raízes de 𝑝 é real.
b) Não existem raízes de 𝑝 que sejam complexas conjugadas.
c) A soma dos módulos de todas as raízes de 𝑝 é igual a 2 + √2.
d) O produto dos módulos de todas as raízes de 𝑝 é igual a 2√2.
e) O módulo de uma das raízes de 𝑝 é igual a √2.
Comentários
O polinômio fornecido assusta à primeira vista. Mas se você observar com calma, vai
perceber que:

AULA 07 – EQUAÇÕES ALGÉBRICAS 92


Prof. Victor So

𝑝 (𝑧 ) = 𝑧 4 + (2 + 𝑖 ) 𝑧 3 + (2 + 𝑖 ) 𝑧 2 + (2 + 𝑖 ) 𝑧 + (1 + 𝑖 ) =
= 𝑧 4 − 1 + 1 + (2 + 𝑖 ) 𝑧 2 + (2 + 𝑖 ) 𝑧 + (1 + 𝑖 ) =
= 𝑧 4 − 1 + (2 + 𝑖)(𝑧 3 + 𝑧 2 + 𝑧 + 1)
Isso é muito bom, pois 𝑝(1) ≠ 0, ou seja, para 𝑝(𝑧) = 0, podemos escrever:

4
(2 + 𝑖)(𝑧 4 − 1) 2+𝑖
𝑝 (𝑧 ) = 𝑧 − 1 + = (𝑧 4 − 1) (1 + )=0
𝑧−1 𝑧−1
Já que 𝑧 ≠ 1. Disso, temos que:
𝑧 4 − 1 = 0 ⇒ (𝑧 2 − 1)(𝑧 2 + 1) = 0
Resolvendo para 𝑧, vem:
𝑧 = ±1 𝑜𝑢 𝑧 = ±𝑖
Mas 𝑧 ≠ 1, do que segue que 1 não convém.
Ainda temos que:
2+𝑖 𝑧+1+𝑖
1+ =0⇒ = 0 ⇒ 𝑧 = −1 − 𝑖
𝑧−1 𝑧−1
Note que o módulo dessa última raiz de 𝑝(𝑧) é |𝑧| = √(−1)2 + (−1)2 = √2, do que segue
que a alternativa “e” é a correta.
Gabarito: “e”.
46. (ITA/2014)
Considere os polinômios em 𝑥 ∈ ℝ da forma 𝑝(𝑥) = 𝑥 5 + 𝑎3 𝑥 3 + 𝑎2 𝑥 2 + 𝑎1 𝑥. As raízes de
1
𝑝(𝑥) = 0 constituem uma progressão aritmética de razão quando (𝑎1 , 𝑎2 , 𝑎3 ) é igual a
2
1 5
a) ( , 0, ).
4 4
1 5
b) ( , 1, ).
4 4
1 5
c) ( , 0, − ).
4 4
5 1
d) ( , 0, ).
4 4
1 1
e) ( , −1, − ).
4 4

Comentários
Note que o polinômio dado possui zero como uma de suas raízes, pois:
𝑝(0) = 05 + 𝑎3 03 + 𝑎2 02 + 𝑎1 0 = 0
Seja 𝑟 a “raiz central”, ou seja, tal que:
1 1
(𝑟 − 1, 𝑟 − , 𝑟, 𝑟 + , 𝑟 + 1)
2 2

AULA 07 – EQUAÇÕES ALGÉBRICAS 93


Prof. Victor So

Sejam as soluções de 𝑝(𝑥) = 0. Das relações de Girard, podemos escrever que:


1 1 𝑎4 0
𝑟−1+𝑟− +𝑟+𝑟+ +𝑟+1=− =− =0
2 2 𝑎5 1
5𝑟 = 0 ⇒ 𝑟 = 0
Logo, as raízes de 𝑝(𝑥) são:
1 1
(−1, − , 0, , 1 )
2 2
Ainda das relações de Girard, podemos escrever:
𝑎3
= 𝑎3
𝑎5
1 1 1 1 1 1 1
= (−1) ∙ (− ) + (−1) ∙ 0 + (−1) ∙ + (−1) ∙ 1 + (− ) ∙ 0 + (− ) ∙ ( ) + (− ) ∙ 1 + 0 ∙
2 2 2 2 2 2 2
1
+0∙1+ ∙1
2
Ou seja:
5
𝑎3 = −
4
Além disso:
5 1
𝑝(−1) = −1 + + 𝑎2 − 𝑎1 = 0 ⇒ 𝑎2 − 𝑎1 = −
4 4
5 1
𝑝(1) = 1 − + 𝑎2 + 𝑎1 = 0 ⇒ 𝑎2 + 𝑎1 =
4 4
Resolvendo o sistema acima, temos:
1
𝑎2 = 0 𝑒 𝑎1 =
4
Gabarito: “c”.
47. (ITA/2012)
As raízes 𝑥1 , 𝑥2 𝑒 𝑥3 do polinômio 𝑝(𝑥) = 16 + 𝑎𝑥 − (4 + √2)𝑥 2 + 𝑥 3 estão relacionadas
pelas equações:
𝑥3
𝑥1 + 2𝑥2 + = 2 𝑒 𝑥1 − 2𝑥2 − √2𝑥3 = 0
2
Então, o coeficiente 𝑎 é igual a
a) 2(1 − √2).
b) √2 − 4.
c) 2(2 + √2).

AULA 07 – EQUAÇÕES ALGÉBRICAS 94


Prof. Victor So

d) 4 + √2.
e) 4(√2 − 1).
Comentários
Observe que temos três incógnitas, mas o enunciado só nos fornece duas equações.
Podemos obter mais uma equação usando as relações de Girard, pois a soma das raízes é
dada por:
𝑥1 + 𝑥2 + 𝑥3 = 4 + √2
Assim, temos o sistema de equações:
𝑥3
𝑥1 + 2𝑥2 + =2
2
𝑥1 + 𝑥2 + 𝑥3 = 4 + √2
{ 𝑥1 − 2𝑥2 − √2𝑥3 = 0
Somando a primeira e a terceira equação, obtemos:
𝑥3
𝑥1 + 2𝑥2 + + (𝑥1 − 2𝑥2 − √2𝑥3 ) = 2 + 0
2
1 √2 1
2𝑥1 + ( − √2) 𝑥3 = 2 ⇒ 𝑥1 = ( − ) 𝑥3 + 1
2 2 4
Substituindo na segunda equação, vem:
√2 1 √2 3
( − ) 𝑥3 + 1 + 𝑥2 + 𝑥3 = 4 + √2 ⇒ 𝑥2 = 3 + √2 − ( + ) 𝑥3
2 4 2 4
Por fim, substituindo 𝑥2 e 𝑥1 na terceira equação:
√2 1 √2 3
( − ) 𝑥3 + 1 − 2 [3 + √2 − ( + ) 𝑥3 ] − √2𝑥3 = 0 ⇒ 𝑥3 = 4
2 4 2 4
Assim, como 4 é raiz do polinômio, podemos escrever:
16 + 4𝑎 − 16(4 + √2) + 64 = 0 ⇒ 𝑎 = 4(√2 − 1)
Gabarito: “e”.
48. (ITA/2010)
Sabe-se que o polinômio 𝑝(𝑥) = 𝑥 5 − 𝑎𝑥 3 + 𝑎𝑥 2 − 1, 𝑎 ∈ ℝ, admite a raiz −𝑖. Considere
as seguintes afirmações sobre as raízes de 𝑝:
I. Quatro das raízes são imaginárias puras.
II. Uma das raízes tem multiplicidade dois.
III. Apenas uma das raízes é real.

AULA 07 – EQUAÇÕES ALGÉBRICAS 95


Prof. Victor So

Destas, é (são) verdadeira(s) apenas


a) I.
b) II.
c) III.
d) I e III.
e) II e III.
Comentários
Observe que 𝑝(𝑥) possui todos os seus coeficientes reais. Assim, se −𝑖 é raiz de 𝑝(𝑥), 𝑖
também é.
Vamos encontrar o valor de 𝑎 usando que 𝑝(𝑖) = 0:
𝑖 5 − 𝑎(𝑖)3 + 𝑎(𝑖)2 − 1 = 0 ⇒ 𝑎 = −1
Então, o polinômio fica:
𝑝 (𝑥 ) = 𝑥 5 + 𝑥 3 − 𝑥 2 − 1
Note que 𝑝(1) = 0:
𝑝 (1) = 1 + 1 − 1 − 1 = 0
Vamos dividir 𝑝(𝑥), sucessivamente por (𝑥 − 1), (𝑥 − 𝑖) e (𝑥 + 𝑖) usando Briot-Rufffini:

1 1 0 1 −1 0 −1

𝑖 1 1 2 1 1 0

−𝑖 1 1+𝑖 1+𝑖 𝑖 0

1 1 1 0

Ou seja:
𝑝(𝑥) = (𝑥 − 1)(𝑥 − 𝑖)(𝑥 + 𝑖)(𝑥 2 + 𝑥 + 1)
−1±𝑖 √3
As raízes de 𝑥 2 + 𝑥 + 1 são 𝑥 = , pertencentes aos complexos.
2

Vamos julgar item por item:


Item I:
Falsa, pois apenas duas de suas raízes são imaginárias puras.
Item II:
Falsa, pois nenhuma de suas raízes possui multiplicidade 2.
Item III:
Verdadeira, pois apenas 1 ∈ ℝ.

AULA 07 – EQUAÇÕES ALGÉBRICAS 96


Prof. Victor So

Gabarito: “c”.
49. (ITA/2010)
Um polinômio real 𝑝(𝑥) = ∑5𝑛=0 𝑎𝑛 𝑥 𝑛 , com 𝑎5 = 4, tem três raízes reais distintas, 𝑎, 𝑏 𝑒 𝑐,
que satisfazem o sistema
𝑎 + 2𝑏 + 5𝑐 = 0
{ 𝑎 + 4𝑏 + 2𝑐 = 6
2𝑎 + 2𝑏 + 2𝑐 = 5
Sabendo que a maior das raízes é simples e as demais tem multiplicidade dois, pode-se
afirmar que 𝑝(1) é igual a
a) −4.
b) −2.
c) 2.
d) 4.
e) 6.
Comentários
O primeiro passo nessa questão é encontrar as raízes resolvendo o sistema linear
fornecido.
Subtraindo a primeira equação da segunda equação:
𝑎 + 4𝑏 + 2𝑐 − (𝑎 + 2𝑏 + 5𝑐 ) = 6 − 0 ⇒ 2𝑏 = 3𝑐 + 6
Substituindo 2𝑏 na primeira equação:
𝑎 + 3𝑐 + 6 + 5𝑐 = 0 ⇒ 𝑎 = −8𝑐 − 6
Substituindo 𝑎 e 2𝑏 na terceira equação, vem:
−16𝑐 − 12 + 3𝑐 + 6 + 2𝑐 = 5 ⇒ 𝑐 = −1
Assim, temos que:
3
2𝑏 = −3 + 6 = 3 ⇒ 𝑏 = 𝑒𝑎 =8−6=2
2
3
A maior das raízes encontradas é 2, do que segue que −1 e possuem multiplicidade 2.
2
Como 𝑎5 = 4, podemos escrever:
3 2
𝑝(𝑥) = 4(𝑥 − 2)(𝑥 + 1 )2 (𝑥 − )
2
Do que segue que 𝑝(1) é dado por:
3 2
𝑝(1) = 4(1 − 2)(1 + 1 )2 (1 − ) = −4
2
Gabarito: “a”.

AULA 07 – EQUAÇÕES ALGÉBRICAS 97


Prof. Victor So

50. (ITA/2009)
Considere as funções 𝑓(𝑥) = 𝑥 4 + 2𝑥 3 − 2𝑥 − 1 e 𝑔(𝑥) = 𝑥 2 − 2𝑥 + 1. A multiplicidade
das raízes não reais da função composta 𝑓 ∘ 𝑔 é igual a
a) 1.
b) 2.
c) 3.
d) 4.
e) 5.
Comentários
Antes de escrevermos a função composta, vamos fazer algumas simplificações:
𝑔(𝑥) = 𝑥 2 − 2𝑥 + 1 = (𝑥 − 1)2
Vamos fatorar 𝑓(𝑥). Note primeiramente que 1 é raiz:
𝑓 (1) = 1 + 2 − 2 − 1 = 0
Vamos usar Briot-Ruffini para fatorar 𝑓(𝑥):

1 1 2 0 −2 −1

1 3 3 1 0

Ou seja:
𝑓(𝑥) = (𝑥 − 1)(𝑥 3 + 3𝑥 2 + 3𝑥 + 1)
Observe que o segundo fator de 𝑓(𝑥) é uma expressão conhecida, se trata do cubo de (𝑥 +
1):
(𝑥 + 1)3 = 𝑥 3 + 3𝑥 2 + 3𝑥 + 1
Assim:
𝑓 (𝑥) = (𝑥 − 1)(𝑥 + 1)3
Agora vamos escrever 𝑓(𝑔(𝑥)):
𝑓 (𝑔(𝑥)) = [(𝑥 − 1)2 − 1][(𝑥 − 1)2 + 1]3
Queremos os valores de 𝑥 para 𝑓(𝑔(𝑥)) = 0, ou seja:
[(𝑥 − 1)2 − 1][(𝑥 − 1)2 + 1]3 = 0 ⇒ (𝑥 − 1)2 − 1 = 0 𝑜𝑢 [(𝑥 − 1)2 + 1]3 = 0
Do que segue que:
(𝑥 − 1)2 − 1 = 0 ⇒ 𝑥 − 1 = ±1 ⇒ 𝑥 = 2 𝑜𝑢 𝑥 = 0
E ainda:
(𝑥 − 1)2 + 1 = 0 ⇒ 𝑥 − 1 = ±𝑖 ⇒ 𝑥 = 1 ± 𝑖

AULA 07 – EQUAÇÕES ALGÉBRICAS 98


Prof. Victor So

Perceba que o fator (𝑥 − 1)2 + 1 se repete três vezes no polinômio 𝑓(𝑔(𝑥)), ou seja, suas
raízes têm multiplicidade 3 no polinômio composto.
Gabarito: “c”.
51. (ITA/2009)
O polinômio de grau 4
(𝑎 + 2𝑏 + 𝑐 )𝑥 4 + (𝑎 + 𝑏 + 𝑐 )𝑥 3 − (𝑎 − 𝑏)𝑥 2 + (2𝑎 − 𝑏 + 𝑐 )𝑥 + 2(𝑎 + 𝑐 ),
Com 𝑎, 𝑏, 𝑐 ∈ ℝ, é uma função par. Então, a soma dos módulos de suas raízes é igual a
a) 3 + √3.
b) 2 + 3√3.
c) 2 + √2.
d) 1 + 2√2.
e) 2 + 2√2.
Comentários
Seja 𝑝(𝑥) o polinômio dado. Se ele é uma função par, então temos que:
𝑝(𝑥) = 𝑝(−𝑥)
Ou seja:
(𝑎 + 2𝑏 + 𝑐 )𝑥 4 + (𝑎 + 𝑏 + 𝑐 )𝑥 3 − (𝑎 − 𝑏)𝑥 2 + (2𝑎 − 𝑏 + 𝑐 )𝑥 + 2(𝑎 + 𝑐 )
= (𝑎 + 2𝑏 + 𝑐 )𝑥 4 − (𝑎 + 𝑏 + 𝑐 )𝑥 3 − (𝑎 − 𝑏)𝑥 2 − (2𝑎 − 𝑏 + 𝑐 )𝑥 + 2(𝑎 + 𝑐 )
Que implica:
2(𝑎 + 𝑏 + 𝑐 )𝑥 3 + 2(2𝑎 − 𝑏 + 𝑐 )𝑥 = 0𝑥 3 + 0𝑥 = 0
Como vale para todo 𝑥, a igualdade polinomial nos permite escrever que:
(𝑎 + 𝑏 + 𝑐 ) = 0 𝑒 (2𝑎 − 𝑏 + 𝑐 ) = 0
Isolando 𝑎 𝑒 𝑐 em função de 𝑏, temos:
𝑎 = 2𝑏 𝑒 𝑐 = −3𝑏
Assim, o polinômio fica:
𝑝(𝑥) = 𝑏𝑥 4 − 𝑏𝑥 2 − 2𝑏 = 𝑏(𝑥 4 − 𝑥 2 − 2)
Queremos as soluções de 𝑥 4 − 𝑥 2 − 2 = 0. Faça 𝑥 2 = 𝑦. Do que temos:
𝑦2 − 𝑦 − 2 = 0
Resolvendo para 𝑦, obtemos 𝑦 = 2 ou 𝑦 = −1. Ou seja:
𝑥 2 = 2 ⇒ 𝑥 = ±√2
𝑥 2 = −1 ⇒ 𝑥 = ±𝑖

AULA 07 – EQUAÇÕES ALGÉBRICAS 99


Prof. Victor So

Seu conjunto solução é, portanto:


𝑆 = {±√2, ±𝑖}
A soma do módulo de suas raízes é:
|√2| + |−√2| + |𝑖| + |−𝑖| = 2√2 + 2 = 2(√2 + 1)
Gabarito: “e”.
52. (ITA/2006)
Sobre o polinômio 𝑝(𝑥) = 𝑥 5 − 5𝑥 3 + 4𝑥 2 − 3𝑥 − 2 podemos afirmar que
a) 𝑥 = 2 não é raiz de 𝑝.
b) 𝑝 só admite raízes reais, sendo uma delas inteira, duas racionais e duas irracionais.
c) 𝑝 admite uma única raiz real, sendo ela uma raiz inteira.
d) 𝑝 só admite raízes reais, sendo duas delas inteiras.
e) 𝑝 admite somente 3 raízes reais, sendo uma delas inteira e duas irracionais.
Comentários
Vamos verificar cada alternativa.
Alternativa A:
Vamos calcular 𝑝(2):
𝑝(2) = 25 − 5 ∙ 23 + 4 ∙ 22 − 3 ∙ 2 − 2 = 32 − 40 + 16 − 6 − 2 = 48 − 48 = 0
∴ 𝑝 (2) = 0
Logo, a alternativa é falsa, pois 2 é raiz do polinômio.
Antes de continuar, vamos fatorar o polinômio usando Briot-Ruffini:

2 1 0 −5 4 −3 −2

1 2 −1 2 1 0

Do que temos:
𝑝(𝑥) = (𝑥 − 2)(𝑥 4 + 2𝑥 3 − 𝑥 2 + 2𝑥 + 1)
Para investigar 𝑝(𝑥) = 0, devemos estudar 𝑥 4 + 2𝑥 3 − 𝑥 2 + 2𝑥 + 1 = 0.
Essa equação é do tipo:
𝑎𝑥 4 + 𝑏𝑥 3 + 𝑐𝑥 2 + 𝑏𝑥 + 𝑎
Existe um truque conhecido para resolvê-la. Divida a equação por 𝑥 2 , ou seja, o termo
“central”:
1 1
𝑎𝑥 2 + 𝑏𝑥 + 𝑐 + 𝑏 ( ) + 𝑎 ( 2 ) = 0
𝑥 𝑥

AULA 07 – EQUAÇÕES ALGÉBRICAS 100


Prof. Victor So

Agrupando os termos:
1 1
𝑎 (𝑥 2 +
) + 𝑏 (𝑥 + )+𝑐 =0
𝑥2 𝑥
Agora, faça a seguinte substituição de variável:
1
𝑦=𝑥+
𝑥
Perceba que:
1 2 1 1
𝑦 = (𝑥 + ) = 𝑥 2 + 2 + 2 ⇒ 𝑦 2 − 2 = 𝑥 2 + 2
2
𝑥 𝑥 𝑥
Substituindo na equação, vem:
𝑎(𝑦 2 − 2) + 𝑏𝑦 + 𝑐 = 0 𝑜𝑢 𝑎𝑦 2 + 𝑏𝑦 + 𝑐 − 2𝑎 = 0
No nosso caso, 𝑎 = 1, 𝑏 = 2 e 𝑐 = −1.
Ou seja:
𝑦 2 + 2𝑦 − 3 = 0
Resolvendo para 𝑦, temos 𝑦 = 1 𝑜𝑢 𝑦 = −3.
Assim:
1
= 1 ⇒ 𝑥2 − 𝑥 + 1 = 0
𝑥+
𝑥
Que resolvendo para 𝑥, temos:
1 ± 𝑖√3
𝑥=
2
E também temos que:
1
= −3 ⇒ 𝑥 2 + 3𝑥 + 1 = 0
𝑥+
𝑥
Que resolvendo para 𝑥, temos:
−3 ± √5
𝑥=
2
Dessa forma, observe que 𝑝(𝑥) admite três raízes reais, sendo uma delas o 2, inteira, e as
−3±√5
outras duas, , irracionais.
2

Gabarito: “e”.
53. (ITA/2006)
Seja 𝑝 um polinômio com coeficientes reais, de grau 7, que admite 1 − 𝑖 como raiz de
multiplicidade 2. Sabe-se que a soma e o produto de todas as raízes de 𝑝 são,

AULA 07 – EQUAÇÕES ALGÉBRICAS 101


Prof. Victor So

respectivamente, 10 e −40. Sendo afirmado que três raízes de 𝑝 são reais e distintas e
formam uma progressão aritmética, então, tais raízes são
3 193 3 193
a) √ ,3, + √ .
2 6 2 6

b) 2 − 4√13, 2, 2 + 4√13.
c) −4, 2, 8.
d) −2, 3, 8.
e) −1, 2, 5.
Comentários
Primeiramente, perceba que 𝑝(𝑥) possui coeficientes reais. Isso implica que, se 1 − 𝑖 é
uma de suas raízes, então seu conjugado, 1 + 𝑖, também é.
Além disso, 1 − 𝑖 possui multiplicidade 2, do que segue que 1 + 𝑖 também possui
multiplicidade 2.
Dessa forma, já temos 4 raízes do polinômio.
Sejam 𝑟1 , 𝑟2 𝑒 𝑟3 as outras raízes de 𝑝(𝑥), já que seu grau é 7. O enunciado nos diz que elas
estão em P.A. e são reais, do que podemos representá-las pela terna ordenada:
(𝑟2 − 𝑟, 𝑟2 , 𝑟2 + 𝑟)
Temos mais duas informações úteis para usar:
Informação 1: A soma de todas as raízes de 𝑝 é 10.
Ou seja:
(1 − 𝑖) + (1 − 𝑖) + (1 + 𝑖) + (1 + 𝑖) + 𝑟2 − 𝑟 + 𝑟2 + 𝑟2 + 𝑟 = 10
Que implica:
4 + 3𝑟2 = 10 ⇒ 𝑟2 = 2
Informação 2: O produto de todas as raízes de 𝑝 é −40.
Ou seja:
(1 − 𝑖)2 (1 + 𝑖)2 (2 − 𝑟)2(2 + 𝑟) = −40
Antes de continuar, veja que:
(1 − 𝑖)2 (1 + 𝑖)2 = [(1 − 𝑖)(1 + 𝑖)]2 = [1 + 𝑖 − 𝑖 + 1]2 = 4
Assim:
4(2 − 𝑟)2(2 + 𝑟) = −40 ⇒ (4 − 𝑟 2 ) = −5 ⇒ 𝑟 = ±3
Tomando 𝑟 = 3, temos:
(−1,2,5)

AULA 07 – EQUAÇÕES ALGÉBRICAS 102


Prof. Victor So

Tomando 𝑟 = −3, temos:


(5,2, −1)
Gabarito: “e”.
54. (ITA/2005)
O número complexo 2 + 𝑖 é raiz do polinômio 𝑓 (𝑥) = 𝑥 4 + 𝑥 3 + 𝑝𝑥 2 + 𝑥 + 𝑞, com 𝑝, 𝑞 ∈
ℝ. Então, a alternativa que mais se aproxima da soma das raízes reais de 𝑓 é
a) 4.
b) −4.
c) 6.
d) 5.
e) −5.
Comentários
Se 𝑝 𝑒 𝑞 são reais, então 𝑓(𝑥) é um polinômio com coeficientes reais.
Disso, temos que se 2 + 𝑖 é uma raiz de 𝑓(𝑥), então 2 − 𝑖 também é raiz.
Observe que 𝑓(𝑥) possui grau 4, isto é, restam duas raízes 𝑟1 𝑒 𝑟2 a determinar. Se uma
delas for complexa, a outra também deve ser, pois seu conjugado também deverá ser raiz. Dessa
forma, se 𝑓(𝑥) admite uma raiz real, nesse caso ele deverá admitir duas.
Das relações de Girard:
2 + 𝑖 + 2 − 𝑖 + 𝑟1 + 𝑟2 = −1 ⇒ 𝑟1 + 𝑟2 = −5
Gabarito: “e”.
55. (ITA/2004)
Para algum número real 𝑟, o polinômio 8𝑥 3 − 4𝑥 2 − 42𝑥 + 45 é divisível por (𝑥 − 𝑟)2 . Qual
dos números abaixo está mais próximo de 𝑟?
a) 1,62.
b) 1,52.
c) 1,42.
d) 1,32.
e) 1,22.
Comentários
Primeiramente, observe que 𝑝(𝑥) pode ser escrito como:
𝑝(𝑥) = (2𝑥)3 − (2𝑥)2 − 21(2𝑥) + 45
Isso sugere a seguinte substituição de variável:

AULA 07 – EQUAÇÕES ALGÉBRICAS 103


Prof. Victor So

𝑦 = 2𝑥
Ou seja:
𝑝(𝑦) = 𝑦 3 − 𝑦 2 − 21𝑦 + 45
Uma ferramenta muito importante em questões onde não sabemos por onde começar a
resolução do polinômio é testar se ele possui raízes racionais.
𝑝
Lembre-se que se 𝑝(𝑦) possui raízes 𝑦 = , racionais, devemos ter:
𝑞

𝑝|45 𝑒 𝑞|1
Fatorando 45, vem:
45 = 1 ∙ 32 ∙ 5
Ou seja:
𝑝 ∈ {±1, ± 3, ±9, ± 5, ± 15, ± 45}
𝑞 ∈ {±1}
Do que temos que:
𝑝
∈ {±1, ± 3, ±9, ± 5, ± 15, ± 45}
𝑞
Vamos testar da menor positiva para a maior positiva:
𝑝
= 1 ⇒ 𝑝(1) = 1 − 1 − 21 + 45 = 24 ≠ 0
𝑞
𝑝
= 3 ⇒ 𝑝(3) = 33 − 32 − 21 ∙ 3 + 45 = 0
𝑞
Temos então que 𝑦 = 3 é raiz de 𝑝(𝑦). Vamos usar Brio-Ruffini para reduzir o grau do
polinômio:

3 1 −1 −21 45

1 2 −15 0

Dessa forma, podemos escrever:


𝑝(𝑦) = (𝑦 − 3)(𝑦 2 + 2𝑦 − 15)
Resolvendo a equação do segundo grau, temos que 𝑦 = 3 𝑜𝑢 𝑦 = −5. Voltando ao
polinômio inicial, temos:

3 2 2(
3 2
( ) ( ) )
𝑝 𝑥 = 8𝑥 − 4𝑥 − 42 + 45 = 2𝑥 − 3 2𝑥 + 5 = 4 (𝑥 − ) (2𝑥 + 5)
2
3
Ou seja, 𝑟 = .
2

AULA 07 – EQUAÇÕES ALGÉBRICAS 104


Prof. Victor So

Gabarito: “b”.
56. (ITA/2001)
O valor da soma 𝑎 + 𝑏 para que as raízes do polinômio 4𝑥 4 − 20𝑥 3 + 𝑎𝑥 2 − 25𝑥 + 𝑏
estejam em progressão aritmética de razão 1/2 é:
a) 36.
b) 41.
c) 26.
d) -27.
e) -20.
Comentários
Esse polinômio possui grau 4, logo, possui 4 raízes. Como elas estão em progressão
1
aritmética de razão , podemos escrever suas raízes como sendo a 4-tupla ordenada:
2
1 3
(𝑟, 𝑟 + , 𝑟 + 1, 𝑟 + )
2 2
Das relações de Girard, temos que:
1 3 −20 1
𝑟+𝑟+ + 𝑟+1+ 𝑟+ =− = 5 ⇒ 4𝑟 + 3 = 5 ⇒ 𝑟 =
2 2 4 2
Ou seja, suas raízes são:
1 3
( , 1, , 2)
2 2
Seja 𝑝(𝑥) = 4𝑥 4 − 20𝑥 3 + 𝑎𝑥 2 − 25𝑥 + 𝑏. Temos que:
𝑝(1) = 0 ⇒ 4 − 20 + 𝑎 − 25 + 𝑏 = 0 ⇒ 𝑎 + 𝑏 = 41
Gabarito: “b”.
57. (ITA/2001)
O polinômio com coeficientes reais 𝑃(𝑥) = 𝑥 5 + 𝑎4 𝑥 4 + 𝑎3 𝑥 3 + 𝑎2 𝑥 2 + 𝑎1 𝑥 + 𝑎0 tem duas
raízes distintas, cada uma delas com multiplicidade 2, e duas de suas raízes são 2 e 𝑖. Então
a soma dos coeficientes é igual a:
a) −4.
b) −6.
c) −1.
d) 1.
e) 4.
Comentários

AULA 07 – EQUAÇÕES ALGÉBRICAS 105


Prof. Victor So

Se 𝑃(𝑥) possui coeficientes reais, temos que −𝑖 também é raiz de 𝑃(𝑥).


Como o polinômio possui grau 5, temos ainda que restam 3 raízes a determinar.
Sejam então 𝑟1 , 𝑟2 , 𝑟3 , 𝑖, −𝑖 as raízes de 𝑃(𝑥). Seja, sem perda de generalidade, 𝑟1 = 2.
Suponha que 2 seja uma raiz com multiplicidade 2. Disso, teríamos que, por exemplo, 𝑟1 =
𝑟2 = 2, e restaria 𝑟3 a determinar. Lembre-se que ainda temos que obedecer a condição que “tem
duas raízes distintas, cada uma delas com multiplicidade 2”. Mas observe que, se 𝑟3 ≠ 2, então
deveríamos ter mais duas raízes, já que 𝑟3 teria multiplicidade 2, o que é absurdo, dado que o
grau de 𝑃(𝑥) é 5.
Suponha então que 𝑖 possua multiplicidade 2. Faça, para isso, 𝑟2 = 𝑖. Como as raízes
complexas vem aos pares, teríamos que ter 𝑟3 = −𝑖. Observe que, nesse caso, temos duas raízes
distintas, 𝑖 𝑒 − 𝑖 com multiplicidade 2, o que atende à condição do enunciado.
Como o coeficiente de 𝑥 5 do polinômio é 1, podemos escrevê-lo como:
𝑃(𝑥) = (𝑥 − 2)(𝑥 − 𝑖)2 (𝑥 + 𝑖)2 = (𝑥 − 2)(𝑥 2 + 1)2
A soma dos coeficientes de um polinômio pode ser obtida calculando-se seu valor para 𝑥 =
1, logo:
𝑃(1) = (1 − 2)(1 + 1)2 = −4
Gabarito: “a”.
58. (ITA/1998)
Seja 𝑎 um número real tal que o polinômio 𝑝(𝑥) = 𝑥 6 + 2𝑥 5 + 𝑎𝑥 4 − 𝑎𝑥 2 − 2𝑥 − 1 admite
apenas raízes reais. Então:
a) 𝑎 ∈ [2, ∞[.
b) 𝑎 ∈ [−1,1].
c) 𝑎 ∈ ] − ∞, −7].
d) 𝑎 ∈ [−2, −1[.
e) 𝑎 ∈ ]1,2[.
Comentários
Primeiramente, devemos perceber que 𝑝(1) = 0, veja:
𝑝 (1) = 0 ⇒ 1 + 2 + 𝑎 − 𝑎 − 2 − 1 = 0
Podemos então dividir o polinômio por 𝑥 − 1 usando Briot-Ruffini:

1 1 2 𝑎 0 −𝑎 −2 −1

1 3 3+𝑎 3+𝑎 3 1 0

Ou seja:

AULA 07 – EQUAÇÕES ALGÉBRICAS 106


Prof. Victor So

𝑝(𝑥) = (𝑥 − 1)(𝑥 5 + 3𝑥 4 + (3 + 𝑎)𝑥 3 + (3 + 𝑎)𝑥 2 + 3𝑥 + 1)


Observe ainda que:
𝑝(−1) = (−1 − 1)(−1 + 3 − (3 + 𝑎) + (3 + 𝑎) − 3 + 1) = 0
Ou seja, 𝑝(−1) = 0. Vamos então continuar dividindo 𝑝(𝑥), mas agora por 𝑥 + 1:

1 1 2 𝑎 0 −𝑎 −2 −1

−1 1 3 3+𝑎 3+𝑎 3 1 0

1 2 𝑎+1 2 1 0

Disso, resulta que:


𝑝(𝑥) = (𝑥 + 1)(𝑥 − 1)(𝑥 4 + 2𝑥 3 + (𝑎 + 1)𝑥 2 + 2𝑥 + 1)
Vamos agora olhar para o fator 𝑥 4 + 2𝑥 3 + (𝑎 + 1)𝑥 2 + 2𝑥 + 1 e dividir a equação por
𝑥2:
1 1
𝑥 4 + 2𝑥 3 + (𝑎 + 1)𝑥 2 + 2𝑥 + 1 = 0 ⇒ 𝑥 2 + 2𝑥 + (𝑎 + 1) + 2 ( ) + 2
𝑥 𝑥
1 1
Faça 𝑦 = 𝑥 + , temos que 𝑦 2 − 2 = 𝑥 2 + . Logo:
𝑥 𝑥2

𝑦 2 − 2 + 2𝑦 + (𝑎 + 1) = 0 ⇒ 𝑦 2 + 2𝑦 + 𝑎 − 1 = 0 ⇒ 𝑦 2 + 2𝑦 + 1 + 𝑎 − 2 = 0
Ou seja:
(𝑦 + 1)2 + 𝑎 − 2 = 0 ⇒ 𝑦 = −1 ± √2 − 𝑎
1
Olhe agora para 𝑦 = 𝑥 + ⇒ 𝑥 2 − 𝑦𝑥 + 1 = 0. Seu discriminante é:
𝑥

Δ = 𝑦2 − 4
Queremos que suas raízes sejam reais, logo, Δ ≥ 0.
Ou ainda:
𝑦 2 − 4 ⇒ |𝑦| > 2 ⇒ 𝑦 ≥ 2 𝑜𝑢 𝑦 ≤ −2
Temos que 𝑦 = −1 − √2 − 𝑎 < 0, que gera a possibilidade:
−1 − √2 − 𝑎 ≤ −2 ⇒ √2 − 𝑎 ≥ 1 ⇒ 2 − 𝑎 ≥ 1 ⇒ 1 ≥ 𝑎
Note que isso implica:
−1 + √2 − 𝑎 ≥ 0
Que gera a possibilidade:
−1 + √2 − 𝑎 ≥ 2 ⇒ −7 ≥ 𝑎
Além disso, note que 2 − 𝑎 ≥ 0, pois 𝑦 ∈ ℝ.
Queremos então que:

AULA 07 – EQUAÇÕES ALGÉBRICAS 107


Prof. Victor So

𝑎 ≤ 1 𝑒 𝑎 ≤ 2 𝑒 𝑎 ≤ −7
Ou seja:
𝑎 ≤ −7
Gabarito: “c”.
59. (ITA/1996)
Considere o polinômio 𝑝(𝑧) = 𝑧 6 + 2𝑧 5 + 6𝑧 4 + 12𝑧 3 + 8𝑧 2 + 16𝑧.
Sobre as raízes da equação 𝑝(𝑧) = 0, podemos afirmar que:
a) Apenas uma é real.
b) Apenas duas raízes são reais e distintas.
c) Apenas duas raízes são reais e iguais.
d) Quatro raízes são reais, sendo duas a duas distintas.
e) Quatro raízes são reais, sendo apenas duas iguais.
Comentários
Veja, de início, que 𝑧 = 0 é raiz do polinômio.
Ou seja:
𝑝(𝑧) = 𝑧(𝑧 5 + 2𝑧 4 + 6𝑧 3 + 12𝑧 2 + 8𝑧 + 16)
Olhe para o fator 𝑧 5 + 2𝑧 4 + 6𝑧 3 + 12𝑧 2 + 8𝑧 + 16 e veja que:
𝑧 5 + 2𝑧 4 + 6𝑧 3 + 12𝑧 2 + 8𝑧 + 16 = 𝑧 4 (𝑧 + 2) + 6𝑧 2 (𝑧 + 2) + 8(𝑧 + 2)
Ou ainda:
(𝑧 + 2)(𝑧 4 + 6𝑧 2 + 8)
Do que resulta:
𝑝(𝑧) = 𝑧(𝑧 + 2)(𝑧 4 + 6𝑧 2 + 8)
Então perceba que:
𝑝(−2) = −2 ∙ (−2 + 2)((−2)4 + 6(−2)2 + 8) = 0
Ou seja, −2 é raiz de 𝑝(𝑧).
Por fim, queremos saber os valores de 𝑧 tais que:
𝑧 4 + 6𝑧 2 + 8 = 0
Faça 𝑧 2 = 𝑤:
𝑤 2 + 6𝑤 + 8 = 0
Que resolvendo para 𝑤, resulta em:
𝑤 = −2 𝑒 𝑤 = −4

AULA 07 – EQUAÇÕES ALGÉBRICAS 108


Prof. Victor So

Ou ainda:
𝑧 2 = −2 ⇒ 𝑧 = ±𝑖√2
𝑧 2 = −4 ⇒ 𝑧 = ±2𝑖
Suas raízes são:
0,2, ±𝑖√2, ±2𝑖
Gabarito: “b”.
60. (ITA/1994)
Seja 𝑃(𝑥) um polinômio de grau 5, com coeficientes reais, admitindo 2 e 𝑖 como raízes. Se
𝑃(1)𝑃(−1) < 0, então o número de raízes reais de 𝑃(𝑥) pertencentes ao intervalo ] − 1,1[
é:
a) 0.
b) 1.
c) 2.
d) 3.
e) 4.
Comentários
Se 𝑃(𝑥) possui coeficientes reais, podemos afirmar que −𝑖 também é raiz de 𝑃(𝑥). Temos
então 3 raízes de 𝑃(𝑥): 2, ±𝑖.
Somente nos restam duas raízes a determinar.
Suponha que ambas pertençam ao intervalo ] − 1,1[. Dessa forma, o gráfico de 𝑃(𝑥) seria
algo do tipo:

AULA 07 – EQUAÇÕES ALGÉBRICAS 109


Prof. Victor So

Ou seja, 𝑃(1) e 𝑃(−1) teriam o mesmo sinal, de forma que:


𝑃(1)𝑃(−1) > 0
Absurdo!
Observação: O gráfico acima foi construído supondo-se raízes quaisquer no intervalo ] −
𝟏, 𝟏[. Note que poderíamos ter, igualmente, 𝑷(𝟏) > 𝟎 𝒆 𝑷(−𝟏) > 𝟎.
Como 𝑃(1)𝑃(−1) < 0, pelo Teorema de Bolzano, podemos afirmar que pelo menos uma
raiz está nesse intervalo. Mas como visto acima, faltam apenas duas a determinar e ambas não
podem estar simultaneamente nesse intervalo, então exatamente uma está nesse intervalo.
Gabarito: “b”.
61. (IME/2019)

Seja a inequação:

𝟔𝒙𝟒 − 𝟓𝒙𝟑 − 𝟐𝟗𝒙𝟐 + 𝟏𝟎𝒙 < 𝟎


Seja (𝒂, 𝒃) um intervalo contido no conjunto solução dessa inequação. O maior valor possível para
𝒃 − 𝒂 é:

a) 𝟐

b) 𝟏𝟑/𝟔

c) 𝟏/𝟑

d) 𝟓/𝟐
e) 𝟖/𝟑

AULA 07 – EQUAÇÕES ALGÉBRICAS 110


Prof. Victor So

Comentários
Inicialmente, devemos fatorar a expressão do polinômio:
𝑝(𝑥) = 6𝑥 4 − 5𝑥 3 − 29𝑥 2 + 10𝑥 = 𝑥 (6𝑥 3 − 5𝑥 2 − 29𝑥 + 10)
Para simplificar a expressão do terceiro grau podemos aplicar o teorema das raízes
racionais. Os números divisores do coeficiente 𝑎0 = 10 são {±1; ±2; ±5; ±10}, vamos verificar
se há alguma raiz inteira:
Para 𝑥 = 1:
6(1)3 − 5(1)2 − 29(1) + 10 = 6 − 5 − 29 + 10 = −18
Para 𝑥 = −1:
6(−1)3 − 5(−1)2 − 29(−1) + 10 = −6 − 5 + 29 + 10 = 28
Para 𝑥 = 2:
6(2)3 − 5(2)2 − 29(2) + 10 = 48 − 20 − 58 + 10 = −20
Para 𝑥 = −2:
6(−2)3 − 5(−2)2 − 29(−2) + 10 = −48 − 20 + 58 + 10 = 0
Portanto, 𝑥 = −2 é raiz. Podemos aplicar o algoritmo de Briot-Ruffini:
−2 6 −5 −29 10
6 −17 5 0
Assim, encontramos:
𝑝(𝑥) = 𝑥 (𝑥 + 2)(6𝑥 2 − 17𝑥 + 5)
Para simplificar a expressão do segundo grau, basta encontrar suas raízes:
17 ± √169 17 ± 13 5 1
6𝑥 2 − 17𝑥 + 5 = 0 ⇒ 𝑥 = = = 𝑜𝑢
12 12 2 3
5 1
𝑝(𝑥) = 6𝑥 (𝑥 + 2) (𝑥 − ) (𝑥 − )
2 3
Vamos fazer o estudo do sinal:

AULA 07 – EQUAÇÕES ALGÉBRICAS 111


Prof. Victor So

Desse modo, para 𝑝(𝑥) < 0, devemos ter:


1 5
𝑥 ∈ (−2, 0) ∪ ( , )
3 2
Como (𝑎, 𝑏) está contido no conjunto solução e queremos que 𝑏 − 𝑎 seja máximo, então:
Se (𝑎, 𝑏) = (−2, 0):
𝑏 − 𝑎 = 0 − (−2) = 2
1 5
Se (𝑎, 𝑏) = ( , ):
3 2
5 1 13
𝑏−𝑎 = − = >2
2 3 6
Portanto:
13
𝑏−𝑎 =
2
Gabarito: “b”.
62. (IME/2019)

Sejam 𝒙𝟏 , 𝒙𝟐 e 𝒙𝟑 raízes da equação 𝒙𝟑 − 𝒂𝒙 − 𝟏𝟔 = 𝟎. Sendo 𝒂 um número real, o valor de 𝒙𝟑𝟏 +


𝒙𝟑𝟐 + 𝒙𝟑𝟑 é igual a:

a) 𝟑𝟐 − 𝒂
b) 𝟒𝟖 − 𝟐𝒂

AULA 07 – EQUAÇÕES ALGÉBRICAS 112


Prof. Victor So

c) 𝟒𝟖

d) 𝟒𝟖 + 𝟐𝒂

e) 𝟑𝟐 + 𝒂
Comentários
A questão pede para calcular o valor da expressão 𝑥13 + 𝑥23 + 𝑥33 , sendo 𝑥1 , 𝑥2 , 𝑥3 raízes da
equação 𝑥 3 − 𝑎𝑥 − 16 = 0. Notando a presença do termo cúbico na equação, podemos
substituir as raízes nessa equação e encontrar:
𝑥13 − 𝑎𝑥1 − 16 = 0
𝑥23 − 𝑎𝑥2 − 16 = 0
𝑥33 − 𝑎𝑥3 − 16 = 0
Somando as equações:
(𝑥13 + 𝑥23 + 𝑥33 ) − 𝑎(𝑥1 + 𝑥2 + 𝑥3 ) − 48 = 0
𝑥13 + 𝑥23 + 𝑥33 = 𝑎(𝑥1 + 𝑥2 + 𝑥3 ) + 48
Pelas relações de Girard, obtemos:
𝑥1 + 𝑥2 + 𝑥3 = 0
𝑥13 + 𝑥23 + 𝑥33 = 𝑎 ⏟
(𝑥1 + 𝑥2 + 𝑥3 ) + 48
0

Portanto:
𝑥13 + 𝑥23 + 𝑥33 = 48
Gabarito: “c”.

AULA 07 – EQUAÇÕES ALGÉBRICAS 113


Prof. Victor So

10. CONSIDERAÇÕES FINAIS DA AULA


Chegamos ao final da última aula de álgebra para a prova. Nesta aula, estudamos como
resolver equações algébricas e diversos teoremas que nos ajudam a encontrar suas raízes. Para
internalizar esses teoremas, o melhor jeito é resolver muitos exercícios. Assim, quando você se
deparar com uma questão sobre equações algébricas, você já estará craque e terá grandes
chances de acerto!
O tópico mais importante para a prova é, sem dúvidas, as relações de Girard. Então, se
você não entendeu direito esse tópico, recomendo retornar à teoria e refazer as questões desse
tema.
Fico feliz que tenha chegado até aqui! A preparação para concursos de nível elevado como
esse é uma maratona. Muitas pessoas desistem no meio do caminho, mas você está aqui
batalhando pela sua aprovação. E isso já o torna diferente dos outros! Continue se esforçando,
que o resultado virá! Para aqueles que estão sentindo dificuldade, não se sintam mal. Isso é muito
comum entre os estudantes que estão se preparando para esse nível de concurso. O segredo é
persistir!
Antes de me despedir, deixarei aqui uma frase de Sun Tzu:
“A vitória está reservada para aqueles que estão dispostos a pagar o preço.”

11. REFERÊNCIAS BIBLIOGRÁFICAS


[1] Iezzi, Gelson. Fundamentos de matemática elementar, 6: complexos, polinômios e equações.
8. ed. Atual, 2013. 250p.
[2] Morgado, Augusto. Wagner, Eduardo. Carvalho, Paulo. Lima, Elon. A Matemática do Ensino
Médio volume 3. 7 ed. SBM, 2016. 198p.
[3] Guimarães, Caio. Matemática em nível IME/ITA Volume 1: Números Complexos e Polinômios.
1 ed. Vestseller, 2008. 330p.
[4] Andreescu, Titu. 101 Problems in Algebra. AMT Publishing, 2001. 139p.
[5] Andreescu, Titu. Gelca, Răzvan. Putnam and Beyond. Springer, 2007. 798p.

AULA 07 – EQUAÇÕES ALGÉBRICAS 114

Você também pode gostar